Sunteți pe pagina 1din 102

Anul XIX, Nr.

1 Ianuarie – Iunie 2017

RECREAŢII
MATEMATICE
REVISTĂ DE MATEMATICĂ PENTRU ELEVI ŞI PROFESORI

e i   1

Asociaţia „Recreaţii Matematice”


IAŞI – 2017
i
Semnificaţia formulei de pe copertă. Într-o formă concisă, formula e  1 leagă
cele patru ramuri fundamentale ale matematicii: ARITMETICA – reprezentată de 1; GEOMETRIA –
reprezentată de  ; ALGEBRA – reprezentată de i ; ANALIZA MATEMATICĂ – reprezentată de e.

Membri onorifici :
Acad. Constantin CORDUNEANU Acad. Radu MIRON
Prof.univ. Vasile OPROIU Cercet.pr. Dan TIBA

Redactor şef : Temistocle BÎRSAN

Redactori principali : Gabriel POPA, Gheorghe IUREA,


Petru ASAFTEI, Maria RACU

Comitetul de redacţie :
Mihai ANASTASIEI Mihai CRĂCIUN (Paşcani) Marian TETIVA (Bârlad)
Cornelia-Livia BEJAN Paraschiva GALIA Lucian TUŢESCU (Craiova)
Sânziana CARAMAN Paul GEORGESCU Adrian ZANOSCHI
Alexandru CĂRĂUŞU Dan POPESCU (Suceava) Titu ZVONARU (Comăneşti)
Constantin CHIRILĂ Neculai ROMAN (Mirceşti)
Adrian CORDUNEANU Ioan ŞERDEAN (Orăştie)

Materialele vor fi trimise la una dintre adresele: t_birsan@yahoo.com, profgpopa@yahoo.co.uk

COPYRIGHT © 2008, ASOCIAŢIA “RECREAŢII MATEMATICE”


Toate drepturile aparţin Asociaţiei “Recreaţii Matematice”. Reproducerea integrală sau parţială a
textului sau a ilustraţiilor din această revistă este posibilă numai cu acordul prealabil scris al acesteia.

TIPĂRITĂ LA BLUE SIM&Co IAŞI


Str. Petru Movilă, nr. 59
Tel. 0332 111021, 0721 571705; e-mail: simonaslf@yahoo.com

ISSN 1582 – 1765


Anul XIX, Nr. 1 Ianuarie – Iunie 2017

RECREAŢII
MATEMATICE
REVISTĂ DE MATEMATICĂ PENTRU ELEVI ŞI PROFESORI

e i  1

Revistă cu apariţie semestrială

EDITURA „RECREAŢII MATEMATICE”

IAŞI – 2017
Doctor PAUL TANCO - personalitate a epocii sale
La nici 30 de ani lui Paul Tanco i se acordase, la 14
decembrie 1872, DIPLOMA DE DOCTOR ÎN MATEMA-
TICĂ a Universităţii din Graz. În istoria matematicii el
se ı̂nscrie ca primul român care a obţinut titlul de doctor1 .
Şi-a făcut din profesorat un ideal ı̂nchinat neamului său.
Alegerea lui, ı̂n toamna anului 1873, ı̂n corpul profesoral
al liceului avea să fie legământul sufletesc şi profesional
cu şcoala năsăudeană pentru ı̂ntrega sa viaţă. Din 30 de
ani (1873-1903) de carieră didactică, 28 a avut funcţia de
director sau prodirector. Concomitent cu activitatea di-
dactică este un perseverent cercetător, scriind şi publicând
ı̂n diverse reviste. Propagă ştiinţele fizico-matematice şi
contribuie la răspândirea lor de la catedră, prin conferinţe şi prin articole de presă. A
ı̂mbinat ı̂n mod constant activitatea didactică şi ştiinţifică cu preocupări legate de alte
domenii şi probleme ale comunităţii. Se ştie că a fost unul dintre fondatori şi mulţi
ani director la ,,Aurora”, Societate de Împrumut şi Păstrare, prima de acest fel din
România. A făcut parte mulţi ani din comitetul ,,Fondul Central Scolastic” şi a iniţiat
măsuri pentru stimularea elevilor eminenţi, trimiţându-i la şcoli superioare. A fost
preşedinte al societăţii ,,Vasile Naşcu” ce acorda ajutoare elevilor lipsiţi de mijloace
materiale. A stat mult timp ı̂n fruntea ,,ASTREI” locale, una dintre cele mai active
filiale din Transilvania. A sprijinit activitatea societăţii de lectură ,,Virtus Romana
Rediviva” a elevilor cursului superior şi care avea organ publicistic revista ,,Muza
Someşană”. Acest sprijin este remarcabil având ı̂n vedere că ı̂n acea perioadă a dua-
lismului a trebuit să se confrunte cu o serie de măsuri care urmăreau deznaţionalizarea
românilor şi inhibarea spiritului de afirmare a elitelor româneşti. Perioada de direc-
torat a coincis şi cu anii grei şi importanţi ı̂n care s-a construit clădirea de azi a
Colegiului Naţional ,,George Coşbuc”. La 4 octombrie 1888 Paul Tanco a primit ı̂n
numele Gimnaziului Superior şi ı̂n calitate de director cheile noii clădiri din mâna
patronului făuritor - Fondurile Grănicereşti. Merită să reţinem din cuvântul rostit
următoarele: ,,O icoană vrednică de admirat ne ı̂nfăţişează toate acestea, dar totodată
atâta mărinimie, atâtea binefaceri, atâtea ı̂ngrijiri şi interesări, atâtea rezultate ale
muncii ı̂n acest răstimp de 25 de ani; şi câtă dorinţă impun asupra mea şi a colegilor
profesori! Grea răspundere ne aşteaptă. Când primesc această cheie mi-am dat seama
de această răspundere şi fiind dedicaţi carierei de profesori ı̂n acest moment solemn
nu putem să zicem mai mult decât - mulţumind din adâncul sufletului patronilor şi
susţinătorilor gimnaziului pentru numeroasele binefaceri de până acum şi rugându-i
de binevoitorul lor ajutor şi sprijin şi ı̂n viitor - cu tot respectul să facem promisiunea
solemnă, căci cu ajutorul lui Dumnezeu vom năzui şi ı̂n viitor să facem destul cu toată
conştiinciozitatea şi scrupulozitatea aşteptărilor şi dorinţelor ce avem ca educatori”2 .
În condiţiile politice grele de la finele secolului al XIX-lea şi ı̂nceputul secolu-
1 Vezi George Şt. Andonie, Istoria matematicii ı̂n România, Vol. I, Editura Ştiinţifică, Bucureşti,

1965, p. 249.
2 Virgil Şotropa - Nicolae Drăganu, Istoria Şcoalelor Năsăudene, Năsăud, Tipografia G. Matheiu,

Bistriţa, 1913, p. 182-183.

1
lui al XX-lea, Paul Tanco, ı̂n calitate de prodirector a militat, ı̂n ciuda opoziţiei
reprezentanţilor Dualismului, pentru evidenţierea tradiţiilor şi aspiraţiilor gimnaziu-
lui românesc năsăudean ı̂n lumina drepturilor naţionale şi istorice. Considerat eveni-
ment ı̂n viaţa gimnaziului, pensionarea matematicianului prilejuieşte profesorului Ioan
Păcurariu ı̂n discursul la aniversarea celor 40 de ani ai instituţiei, ı̂n 4 octombrie 1903,
o scurtă caracterizare: ,,Cu fine anului şcolar 1902/1903 dr. Paul Tanco, vrednicul
şi de toţi veneratul profesor de la acest gimnaziu, a ı̂mplinit 30 de ani de servit fără
ı̂ntrerupere, cu abnegaţie şi zel neobosit, cu adevărat tact pedagogic şi cu rezultate
neı̂ntrecute ı̂n acest gimnaziu. Cine odată a servit la un gimnaziu 30 de ani fără
ı̂ntrerupere cu zel şi succesele unui dr. Paul Tanco, acela fără ı̂ndoială şi-a câştigat
titlul de a i se recunoaşte ı̂n gradul suprem meritele de crescătoriu şi de a se duce
cu deplină satisfacţiune sufletească ı̂n pensiunea binemeritată. (...). Când ı̂nsă zelul,
abnegaţia şi interesul d-lui dr. Paul Tanco faţă de gimnaziu şi cum după activitatea
ı̂ncordată de 30 de ani sunt atât de mari, ı̂ncât preponderează faţă de orice reflexiuni
la o viaţă liniştită şi ticnită şi-l ı̂nduplecă, să mai rămână ı̂n serviciul activ şi mai
departe la gimnaziu, numai ca interesele şi necesităţile pedagogico-didactice ale aces-
tuia să fie satisfăcute, - o! atunci d-l dr. Paul Tanco ı̂n zelul şi interesul său faţă de
gimnaziu apare mult mai sus şi mai demn de a fi sı̂rbătorit ” 3
Paul Tanco s-a stins din viaţă la 11 septembrie 1916 ı̂n Năsăud, la vârsta de 73 de
ani. Pentru familie va rămâne un titlu de mândrie ı̂ndreptăţită, profundă admiraţie
şi exemplul ı̂nălţimii nebănuite la care se poate ridica un apostol al ştiintei şi moralei.

stră- şi stră-strănepoţii4

100 de ani de la moartea lui PAUL TANCO - cronologie

• S-a născut la 24 iulie 1843 ı̂n Monor, comună din ţinutul Năsăudului, pe atunci
District militar grăniceresc.
• Părinţii săi, Ion şi Anisia, erau ţărani români. În timpul Revoluţiei din 1848,
ca urmare a evenimentelor sângeroase abătute asupra satului, rămâne orfan de tată.
Nu trec nici trei ani şi ı̂şi pierde şi mama.
• A ı̂nceput să ı̂nveţe carte la şcoala trivială 1 din Monor.
• În 1853, unchiul Petru Tanco, fratele mamei sale, locotenent pensionar (fost
combatant ı̂n Regimentul II al grănicerilor români) ı̂l duce, dimpreună cu vărul său
Teodor Tanco, orfan şi acesta, la şcoala primară superioară Norma2 din Năsăud.
3 Ioan Păcurariu, discurs rostit la 4 octombrie 1903..., ı̂n Raportul al XLI-lea despre Gimnaziul

Superior Fundaţional din Năsăud, Bistriţa, Tipografia G. Matheiu, 1904, p. 61-62.


4 Acest material a fost trimis redacţiei de către d-na prof. Adriana Lupoaie, strănepoată şi direc-

toare ı̂ntr-o anumită perioadă a Colegiului Naţional ,,George Coşbuc”.


1 Şcoală primară cu trei obicte principale de studiu: citirea, scrierea, calcularea.
2 Şcoală centrală din Năsăud ce funcţiona cu patru clase, cu predare pe obiecte ı̂n clasele mari (a

III-a şi a IV-a).

2
• Urmează apoi cursul inferior la Gimnaziul german din Bistriţa, aproximativ ı̂n
perioada 1856-1860.
• După o ı̂ntrerupere de doi ani a studiilor, Paul Tanco este trimis să urmeze
cursul superior la Gimnaziul greco-catolic românesc din Blaj (1862-1866). În iulie
1866 trece examenul de maturitate (bacalaureatul).
• Ca elev al liceului blăjean, participă activ la apariţia revistei şcolare literar-
ştiinţifică ,,Buchetul” (ianuarie-iunie 1864), ı̂n care debutează cu studiul Viaţa şi
caracterul lui Socrate.
• În toamna anului 1866, cu sprijinul material acordat de Fondurile grănicereşti,
cât şi cu ajutorul primit de la unchii săi, Paul Tanco se ı̂nscrie la Facultatea de ştiinţe
şi filosofie a Universităţii din Viena. A absolvit cursurile ı̂n anul 1870.
• Complementar cu perfecţionarea ı̂n domeniul matematicii, este atras de pro-
blemele social-naţionale ale studenţilor români. Este secretar al Societăţii literar-
sociale ,,România”, societate precursoare a ,,României June”. În aceste ı̂mprejurări
a cunoscut şi a avut legături de prietenie cu M. Eminescu, I. Slavici şi alţi tineri,
viitori iluştri intelectuali români.
• În anii 1870-1872 continuă studiile la Graz, important centru matematic şi as-
tronomic din Imperiul Habsburgic. Aici, la 14 decembrie 1872, i se acordă diploma
,,Carola Francisca Universitas Graecensis” de doctor ı̂n filozofie (matematica fiind
considerată o secţiune a filozofiei). Paul Tanco este astfel primul român cu doctoratul
ı̂n matematici.
• La 31 aug. 1873 a fost numit profesor de matematică şi fizică la Gimnaziul
Grăniceresc din Năsăud, iar la 19 aug. 1874 a fost ales director al şcolii. A avut
atribuţii de conducere 12 ani ca director şi 16 ani ca prodirector. S-a dovedit a fi
,,un om al şcolii”, excelent pedagog şi bun gospodar. A susţinut activitatea societăţii
de lectură ,,Virtus Romana Rediviva” a elevilor cursului superior şi a publicaţiei sale
,,Musa Someşană”. Printre elevii săi se numără George Coşbuc şi Liviu Rebreanu. A
ieşit la pensie ı̂n anul 1903, dar funcţionează ca provizor şi ı̂n anul şcolar 1903/1904.
• Un eveniment important petrecut la data de 4 oct. 1888, ı̂n timpul celui de-al
doilea directorat al lui Paul Tanco, este inaugurarea noului local al gimnaziului - un
palat şcolar ı̂n care astăzi funcţionează Colegiul Naţional ,,George Coşbuc”.
• Paul Tanco avea o vastă cultură de specialitate şi umanistă. Publică un număr
de lucrări originale ı̂n domeniile matematicilor elementare, astronomie, cosmogonie
ş.a. ı̂n Anuarul Gimnaziului din Năsăud, Revista Ştiinţifică (Bucureşti) şi Recreaţii
Ştiinţifice (Iaşi).
• Publică ı̂n 1892, la Năsăud, culegerea Probleme algebrice, vol.I, ce cuprinde peste
3000 de probleme şi care s-a dovedit foarte utilă.
• Paul Tanco s-a implicat ı̂n variate activităţi de interes public ce aveau ca scop
progresul economic al ţinutului, ţinerea vie a dragostei de neam, ı̂mplinirea idealurilor
naţionale: a fost director al Despărţământului Năsăud al societăţii ASTRA (1901-
1905), se numără printre fondatorii Societăţii de ı̂mprumut şi păstrare ,,Aurora” din
Năsăud şi director din 1903 până ı̂n 1916, a fost preşedinte al Comisiei Administra-
toare a Fondurilor grănicereşti ş.a.
• Paul Tanco a ı̂ncetat din viaţă la 11 sept. 1916, fără să fi avut fericirea de a vedea
Marea Unire. Locul odihnei sale veşnice se află ı̂n cimitirul Comoara din Năsăud.

3
Profesorul VASILE OPROIU
75 de ani de viaţă şi activitate
La 8 septembrie ı̂n anul 1941 a văzut lumina zilei
Vasile Oproiu, geometru cu contribuţii remarcabile ce
i-au adus recunoaşterea şi consacrarea ca personalitate
ştiinţifică de către comunitatea matematică.
Din satul natal, Vulcana-Pandele, din apropierea
confluenţei pârâului Vulcana cu Ialomiţa, a plecat la Pu-
cioasa pentru studiile secundare. Îşi aminteşte cu plăcere
de anii petrecuţi ı̂n liceu, ani ı̂n care interesul pentru
matematică s-a transformat ı̂ntr-o mare pasiune şi a rodit
mai târziu ı̂ntr-o carieră matematică de excepţie. Evocă
cu afecţiune şi multă recunoştinţă figura profesorului său
de matematică, despre care ţine să repete cu orice prilej că
este un caz de ,,om care sfinţeşte locul”. În acest context,
a fost inevitabilă ı̂ntâlnirea sa cu Gazeta Matematică şi participarea la olimpiadele de
matematică. Numele său a figurat ı̂n mod curent ı̂n listele rezolvitorilor de probleme
la Gazetă, dar şi ı̂n listele restrânse ale rezolvitorilor evidenţiaţi.
Studiile superioare le-a făcut ı̂n Iaşi, la Facultatea de matematică a Universităţii
,,Alexandru Ioan Cuza”. A fost un student eminent al unei serii de excepţie a facultăţii
- promoţia 1964 -, din care s-a ridicat un număr impresionant de mare de cercetători
şi cadre universitare: Viorel Barbu (viitorul academicean), Octavian Nanu, Ioan Pop,
Mirela Ştefănescu, Dan Brânzei şi mulţi alţii.
După absolvire, prin repartiţie ocupă un post de cercetător la Institutul de matem-
atică ,,Octav Mayer” al Academiei Române, filiala Iaşi. Aici, ı̂n perioada 1964-1970,
parcurge o etapă fecundă ı̂n acumulări şi rezultate proprii, culminând cu obţinerea
titlului de doctor ı̂n matematici ı̂n anul 1969 cu teza Conexiuni compatibile cu diverse
G-structuri şi structuri de ordinul superior (sub conducerea prof. Gh. Gheorghiev ).
O ı̂ntâmplare semnificativă este legată de savantul Alexandru Myller, creatorul
şcolii de geometrie ieşene. Acesta, ajuns la o vârstă venerabilă şi neputând să mai fie
ı̂n mijlocul unor activităţi publice, a cerut facultăţii să-i recomande un tânăr pentru
a-l ı̂ndruma ı̂n tainele geometriei. Alegerea pentru această onorantă favoare s-a oprit
la tânărul matematician Vasile Oproiu. În fapt, ı̂ntâlnirea nu a putut avea loc, pentru
că la scurt timp, la 4 iulie 1965, ilustrul matematician român a părăsit această lume.
În anul 1970 se transferă la Catedra de geometrie şi topologie a Universităţii ieşene,
devine profesor titular ı̂n 1990 şi se pensionează ı̂n 2006. Din octombrie 2008 este
Profesor Emeritus al Universităţii din Iaşi. De-a lungul anilor a predat variate cursuri
de geometrie: Geometrie diferenţială, Geometrie computaţională, Teoria relativităţii,
Varietăţi complexe, G-structuri şi integrabilitate etc. Ca dascăl, a câştigat aprecierea
şi respectul seriilor de studenţi cărora le-a fost profesor, prin rigoarea şi conţinutul
bogat al cursurilor predate la catedră sau al celor scrise.
Nu putem face decât o sumară referire la rezultatele şi contribuţia ştiinţifică a
Profesorului Vasile Oproiu, revista Recreaţii Matematice adresându-se unui pu-
blic preocupat de matematicile elementare. Domnia sa şi-a desfăşurat activitatea de
cercetare ı̂n domeniile geometriei diferenţiale şi topologiei, domenii ı̂n care a obţinut

4
rezultate de valoare care l-au făcut cunoscut ı̂n ţară şi străinătate; enumerăm doar o
parte dintre contribuţiile sale: geometria diferenţială a G-structurilor, geometria di-
ferenţială a fibratelor tangent şi cotangent, geometria diferenţială a fibratelor de ordin
superior, geometria spaţiilor Lagrange legată de geometria fibratului tangent etc.
Este conducător de doctorat şi creator de şcoală matematică; s-a format ı̂n jurul
său un grup de talentaţi cercetători, unii cu teza susţinută sub ı̂ndrumarea sa, care
au adus contribuţii de valoare ı̂n domeniul geometriei diferenţiale.
Profesorul Vasile Oproiu este autorul a opt cărţi - tratate, monografii, cursuri
pentru uzul studenţilor -, dintre care cităm: Varietăţi diferenţiabile finit şi infinit
dimensionale (Ed. Acad. Române, Buc., vol.I-1976, vol.II-1979; ı̂n colaborare cu Gh.
Gheorghiev), Geometrie diferenţială (Ed. Univ. ,,Al.I. Cuza”, Iaşi, 2002), Geometria
computaţională a curbelor şi suprafeţelor (Ed. Univ. ,,Al.I. Cuza”, Iaşi, 2003).
A participat activ la bunul mers al structurilor organizatorice ale facultăţii şi
universităţii: membru al unor comisii metodice, de cercetare ştiinţifică, de avizare a
distincţiilor pentru teze de doctorat etc., la nivelul Consiliului facultăţii sau Senatului.
Este şeful Catedrei de geometrie ı̂n perioada 2004-2005. Începând cu anul 2006 este
director al Seminarului Matematic ,,Al. Myller” din Iaşi.
A luat parte la multe manifestări ştiinţifice pe plan local sau naţional. A fost invi-
tat să facă expuneri ı̂n plen la importante conferinţe internaţionale ı̂n domeniul geome-
triei diferenţiale (Berlin, Bologna, Bucureşti). A fost invitat să ţină conferinţe ı̂n Polo-
nia (Lublin, Wroclaw), Italia (Cagliari, Bari, Torino), Germania (Berlin, Freiburg).
A fost profesor vizitator ı̂n mari centre universitare din Italia, Polonia şi Germania.
Enumerăm, fără putinţa de a epuiza şi de a detalia, şi alte coordonate ale bogatei
sale activităţii: director al unor granturi de cercetare, referent ı̂n multe comisii de doc-
torat, referent la reviste de informare (Mathematical Reviews, Zentralblatt MATH),
membru al unor societaţi (American Mathematical Society, European Mathematical
Society, Unione Matematica Italiana, Tensor Society, S.S.M.R., Asociaţia ,,Recreaţii
Matematice”), membru al comitetelor de redacţie al unor reviste de specialitate.
Profesorul V. Oproiu a primit ı̂n 1976 premiul ,,Gh. Ţiţeica” acordat de
Academia Română, iar ı̂n 2006 premiul de excelenţă ,,D. Mangeron” pentru ı̂ntreaga
sa activitate.
Pentru cititorii acestei reviste menţionăm faptul că Profesorul Vasile Oproiu
este membru de onoare al Asociaţiei ,,Recreaţii Matematice” şi s-a ataşat de revista
Recreaţii Matematice cu un interes permanent pentru problemele legate de apariţia
sa. A contribuit la creşterea prestigiului revistei publicând ı̂n paginile sale peste
zece articole abordând chestiuni fundamentale ale geometriei sau omagiale. În anul
2008, cu prilejul simpozionului dedicat ı̂mplinirii a 125 de ani de la apariţia revistei
Recreaţii Ştiinţifice (1883 - 1888) - străbuna prezentei reviste -, prof. V. Oproiu a
făcut o expunere amplă asupra părţii sale de geometrie.
Stimate şi Iubite Domnule Profesor Vasile Oproiu, ı̂n numule redacţiei revistei
,,Recreaţii Matematice” vă urez, la ı̂mplinirea a trei sferturi de veac de viaţă, un sincer
şi călduros ,,LA MULŢI ANI ! ”, pentru a vă bucura de roadele muncii voastre şi a
le spori ı̂n anii ce urmează.

Prof.univ.dr. Temistocle BÎRSAN

5
IOAN V. MAFTEI (1941–2016)
Profesorul Ioan V. Maftei, un harnic şi devotat slujitor al ı̂nvăţământului şi
culturii româneşti, a părăsit pentru totdeauna această lume ı̂n ziua de 30 august
2016. Rămâne pentru noi un exemplu de competenţă, muncă, dăruire şi implicare ı̂n
rezolvarea variatelor probleme ale ı̂nvăţământului şi societăţii.
Originar din Buhăieşti, unde s-a născut la data de 20 iulie 1941, a urmat Liceul
,,Mihail Kogălniceanu” din Vaslui şi apoi Facultatea de Matematică a Universităţii
,,Alexandru Ioan Cuza” din Iaşi.
După absolvirea facultăţii, ı̂n anul 1964, a ocupat prin repartiţie ministerială
un post de profesor de matematică ı̂n Bucureşti, la Liceul ,,Electronica”, unde a
funcţionat până ı̂n anul 1980. Din acest moment şi până la pensionare, ı̂n anul 2008,
a profesat la renumitul aşezământ şcolar ,,Sfântul Sava”. A fost un profesor devotat,
cu deosebite calităţi didactice, cu pasiune pentru matematică şi cu darul comunuicării
clare şi convingătoare.
Între anii 1966-1969 a fost redactor la revista Gazeta Matematică (seria A), redac-
torul şef fiind A. Hollinger.
În perioada 1979-1986 a fost inspector general de matematică ı̂n Ministerul Educa-
ţiei şi Învăţământului, postură din care a contribuit la organizarea şi desfăşurarea exa-
menelor de bacalaureat şi a olimpiadelor de matematică pentru elevi. Remarcabilă
este şi contribuţia profesorului I.V. Maftei ca secretar al Comisiei de matematică din
cadrul ministerului, comisie creată cu scopul reformării manualelor de matematică
(preşedinte - acad. Gheorghe Mihoc, vicepreşedinte - prof. dr. Octavian Stănăşilă).
Este de menţionat şi meritul profesorului I.V. Maftei de a fi printre iniţiatorii
Olimpiadei Balcanice de Matematică.
A fost membru activ al mai multor societăţi sau asociaţii cu profil matematic;
la Societatea de Ştiinţe Matematice timp de mai multe decenii, iar la Asociaţia
,,Recreaţii Matematice” din anul 2005 - anul ı̂nfiinţării acesteia.
A desfăşurat o intensă activitate publicistică, concretizată ı̂n cărţi, manuale şcolare
şi culegeri de probleme (Exerciţii şi probleme de analiză matematică pentru clasele
a XI-a şi a XII-a (1981), Teme pentru cercurile şi concursurile de matematică ale
elevilor (1983), Olimpiadele naţionale de matematică pentru liceu, 1954-2003 (2004),
Inegalităţi alese ı̂n matematică (2005) etc. etc.), de preferinţă scrise ı̂n colaborare.
A contribuit cu articole, note şi probleme originale adresate elevilor şi profesorilor la
revistele Gazeta Matematică (seria B), RMT, Recreaţii Matematice, Arhimede ş.a.
În pragul ieşirii la pensie, profesorul I.V. Maftei devine activ şi prolific şi ı̂n dome-
niul complementar al literelor; publică volume de poezie, teatru, epigrame şi devine
membru al Uniunii Scriitorilor din România (filiala Iaşi), al Uniunii Epigramiştilor din
România, al Asociaţiei Literare ,,Păstorel” Iaşi şi al altor societăţi literar-artistice.
S-a simţit sufleteşte legat de locurile natale, de Iaşul studenţiei sale, de colegii de
facultate, printre care sunt unul din cei mai apropiaţi. Şi-a revăzut colegii de facultate
ı̂n anul 2014, cu prilejul celebrării semicentenarului promoţiei.
Dumnezeu să-l odihnească ı̂n pace!

Mihai HAIVAS

6
A new method to solve inequalities
Marius DRĂGAN1 , Neculai STANCIU2

Abstract. In this paper we present a new method to find the best constant k for the inequality
F (s, R, r) ≥ k and for its reverse.
Keywords: fundamental triangle inequality, best constant.
MSC 2010: 51M16.

The purpose of this article is to give a new and simple method to find the best
constant k for the inequality

(1) F (s, R, r) ≥ k,

as well as for its reverse

(2) F (s, R, r) ≤ k,

where F is a homogeneous function of degree zero and monotone with respect to s.


Theorem (fundamental triangle inequality or Blundon′ s inequality). For
any triangle ABC the inequalities

(3) s1 ≤ s ≤ s2

hold, where s1 and s2 are the semiperimeters of two isosceles triangles A1 B1 C1 and
A2 B2 C2 that have the same circumradius R and inradius r as the triangle ABC and
their sides are given as follows:
» »
a1 = 2 (R + r − d)(R − r + d), b1 = c1 = 2R(R + r − d),
» »
a2 = 2 (R + r + d)(R − r − d), b2 = c2 = 2R(R + r + d),

where d = R2 − 2Rr.

We treat the inequality (1) only. The inequality (2) can be treated similarly.
If F is decreasing in the argument s, it follows that F (s, R, r) ≥ F (s2 , R, r), hence
k is the best constant in (1) if and only if it is the best constant in the inequality

(4) F (s2 , R, r) ≥ k.

We denote
s√ √
» 2R − R − r − d
(5) t = (R + r + d)(R − r − d), x = √ √ ∈ (0, 1).
2R + R − r − d
1 Profesor, Colegiul Tehnic ,,Mircea cel Bătrân”, Bucureşti; marius.dragan2005@yahoo.com
2 Profesor, Şcoala Gimnazială ,,George Emil Palade”, Buzău; stanciuneculai@yahoo.com

7
A straightforward computation yields

(1 + x2 )t 2t (1 + x2 )2 t
(6) a2 = 2t, b2 = c2 = , s 2 = , r2 = r = xt, R2 = R = .
1 − x2 1 − x2 4x(1 − x2 )

Then the inequality (4) takes the form


Å ã
2 (1 + x2 )2
F , , x ≥ k, ∀x ∈ (0, 1).
1 − x2 4x(1 − x2 )

Consequently, the best constant k will be the inf x∈(0,1) f (x), where
Å ã
2 (1 + x2 )2
(7) f (x) = F , , x , x ∈ (0, 1).
1 − x2 4x(1 − x2 )

In the case that F is increasing in the argument s, it follows that F (s, R, r) ≥


F (s1 , R, r), hence k is the best constant in (1) if and only if it is the best constant in
the inequality

(8) F (s1 , R, r) ≥ k.

We denote
s√ √
» 2R − R − r − d
(9) τ = (R + r − d)(R − r + d), x = √ √ ∈ (0, 1),
2R + R − r − d

and by a straightforward computation one obtains

(1 + x2 )τ 2τ (1 + x2 )2 τ
(10) a1 = 2τ, b1 = c1 = , s 1 = , r1 = r = xτ, R1 = R = .
1 − x2 1 − x2 4x(1 − x2 )

Then the inequality (8) can be written in the form


Å ã
2 (1 + x2 )2
F , , x = f (x) ≥ k, ∀x ∈ (0, 1),
1 − x2 4x(1 − x2 )

so the best constant k is the inf f (x).


x∈(0,1)
In conclusion, whether F increases or decreases with respect to s, the best constant
k in the inequality (1) is the infimum of the function f defined by (7).

Applications
I. Find the best constant k for which the inequality

(x2 + y 2 + z 2 )(x2 + y 2 + z 2 − xy − yz − zx) ≥


(11) ≥ k(x + y + z)(x2 y + xy 2 + y 2 z + yz 2 + z 2 x + zx2 − 6xyz)

is true for any x, y, z ≥ 0.

8
In [1], Marian Tetiva has solved this problem using the algebraic norming method.
We present a new solution.
Solution. Let
x y z
(12) u= √ , v= √ , w=√ .
Σxy Σxy Σxy
Then, the inequality (11) is equivalent to

(13) (u2 + v 2 + w2 )(u2 + v 2 + w2 − 1) ≥ k(u + v + w)(u + v + w − 9uvw),

where uv + vw + wu = 1.
Since the function tg x2 is bijective on (0, π), it follows that there exists a triangle
ABC such that
A B C
u = tg , v = tg , w = tg .
2 2 2
It is well-known that
X A 4R + r Y A r X 2A (4R + r)2
(14) tg = , tg = , tg = − 2.
2 s 2 s 2 s2
Thus the inequality (13) becomes
ï ò
(4R + r)2 (4R + r)2 − 3s2
(15) 2
− 2 . ≥ k.
s (4R + r)(4R − 8r)

This is an inequality of the form (1). It is obvious that F (s, R, r) – the left hand side
of (15) – is an homogeneous of degree zero and decreasing function in s (is product
of two decreasing functions in s). By a tedious computation one obtains that the
9x4 − 2x2 + 1
function f associated to it as in (7) is f (x) = and that its derivative
12x4 + 4x2
4 2
15x − 6x − 1
is f ′ (x) = .
2x3 (3x2 + 1)2   …
′ 1 2 2
The roots of the equation f (x) = 0 are x1,2 = ± + , x1 > x2 . So,
√ √ 9 9 3
k = inf f (x) = f (x1 ) = 6 − 2, i.e. k = 6 − 2.
x∈(0,1)

II. Find the best constant k such that

(16) (x + y)2 (y + z)2 (z + x)2 − 64x2 y 2 z 2 ≥ k[xyz(x + y + z)3 − 27x2 y 2 z 2 ], ∀x, y, z > 0.

This problem was proposed and left as an exercise by Marian Tetiva in [1]. Here
we give our solution.
Solution. We proceed as in the previous problem to bring (16) to a geometric
inequality of the form (1). With the notations (12) the inequality (16) becomes
X X Y Y îX Y ó
u. uv − 64( u)2 ≥ k u ( u)3 − 27 u ,

9
and this transforms by (14) to

16R2 − 64r2
3 ≥ k,
r[ (4R+r)
s2 − 27r]

that is an inequality of the form (1) with F increasing with respect to s. According
to (7), for f and its derivative we find

4(7x4 − 10x2 − 1) 32x(13x4 + 4x3 − 1)


f (x) = 4 2
, f ′ (x) = .
15x − 14x − 1 (15x4 − 14x2 − 1)2

The equation f ′ (x) = 0 has the real roots


 √
17 2 1
x0 = 0, x1,2 = ± − ≃ 0.404, x1 > x0 , x41 = (1 − 4x21 ).
13 13 13
Therefore,

4(79x21 + 3) 17 + 23
k = inf f (x) = f (x1 ) = = ≃ 3.3903,
x∈(0,1) 121x21 − 1 8

17+23
that is k = 8 is the best constant for the inequality (16).

III. Find the best constants


√ for which the inequalities
(i) a2 + b2 + c2 ≤ 4 √3S + k[(a − b)2 + (b − c)2 + (c − a)2 ],
(ii) a2 + b2 + c2 ≥ 4 3S + k[(a − b)2 + (b − c)2 + (c − a)2 ]
are true in any triangle with area S.
It is well-known that the best constant in (i) is k = 3 and in (ii) this is k = 1 i.e.
when (ii) reduces to Hadwiger-Finsler inequality [2]. We prove these results using our
method.
P P
Solution. (i) Using the identities ab = s2 + r2 + 4Rr, a2 = 2(s2 − r2 −
4Rr), S = rs the inequality (i) can be written in the form

s2 − 2 3rs − r2 − 4Rr
F (s, R, r) = ≤k
s2 − 3r2 − 12Rr
or √
8Rr + 2r2 − 2 3rs
F (s, R, r) = 1 + 2 ≤ k.
s − 3r2 − 12Rr
√ √
We have 8Rr + 2r2 − 2 3rs ≥ 0 because of Finsler′ s inequality 3s ≤ 4R + r.
From Gerretsen′ s inequality, s2 ≥ 16rR − 5r2 , we get s2 ≥ 3r2 + 12Rr. Therefore, F
is a decreasing function in s since it is a product of two positive decreasing functions.
By a direct calculation one finds
Ç √ å2 √
x+ 3 4(x + 3)
f (x) = √ , f ′ (x) = − √ , x ∈ (0, 1).
3x + 1 ( 3x + 1)3

10
So, f (x) is a decreasing function. Hence, k = sup f (x) = f (0) = 3.
x∈(0,1)

(ii) The best constant for (ii) is k = inf f (x) = f (1) = 1.


x∈(0,1)

In the following we shall use our method in order to obtain some refinements of
Gerretsen′ s inequality.

IV. Find the best positive constant k such that the inequality

r2
(17) a2 + b 2 + c2 + k [(a − b)2 + (b − c)2 + (c − a)2 ] ≤ 8R2 + 4r2
s2
is true in any triangle ABC.
Solution. First, the inequality (17) can be written in the form (1) as follows:

4R2 + 3r2 + 4Rr − s2 s2


F (s, R, r) = . 2 ≥ k.
s2 − 3r2 − 12Rr r
We consider the function h : (0, ∞) 7→ R given by

(4R2 + 3r2 + 4Rr)u − u2


h(u) = ,
u − (3r2 + 12Rr)2

whose derivative is
−u2 + 2(3r2 + 12Rr)u − (4R2 + 3r2 + 4Rr)(3r2 + 12Rr)
h′ (u) = .
(u − 3r2 − 12Rr)2

The equation of degree two in u from the numerator has the discriminant ∆ =
−16(3r2 +12Rr)(R−2r) and it is obvious that ∆ ≤ 0. Hence h′ (u) ≤ 0. Consequently,
F is a decreasing function in s.
The best constant k is the infimum of the function
Å ã
2 (1 + x2 )2 1
f (x) = F , , x = 4.
1 − x2 4x(1 − x2 ) x
1
Thus k = inf = 1.
x4
x∈(0,1)
Concluding, the inequality

r2
(18) a2 + b 2 + c2 + [(a − b)2 + (b − c)2 + (c − a)2 ] ≤ 8R2 + 4r2
s2
is the best inequality of the type (17). It is obvious that (18) is a refinement for the
right side of the Gerretsen inequality.

V. Find the best positive constant k such that the inequalities


r2
(i) a2 + b2 + c2 + k · 2 [(a − b)2 + (b − c)2 + (c − a)2 ] ≤ 8R2 + 4r2 and
R

11
r
(ii) a2 + b2 + c2 ≥ 24Rr − 12r2 + k. [(a − b)2 + (b − c)2 + (c − a)2 ],
R
are true in any triangle ABC.
Solution. (i). We write (i) in the form

ï ò 2
4R2 + 3r2 + 4Rr − s2 R2 4R(R − 2r) R
k≤ 2 2
. 2
= 2 2
− 1 . 2 = F (s, R, r).
s − 3r − 12Rr r s − 3r − 12Rr r

Observe that F is a decreasing function in s. The best constant is k = inf f (x),


x∈(0,1)
(1 + x4 )4 (x2 − 1)(x2 + 1)3
where f (x) = 4
, x ∈ (0, 1). Since f ′ (x) = < 0, it follows
4x x5
that k = f (1) = 4. Concluding, the inequality

r2
(19) a2 + b 2 + c2 + 4 [(a − b)2 + (b − c)2 + (c − a)2 ] ≤ 8R2 + 4r2
R2
is valid in any triangle. It is another refinement of the right side of the Gerretsen
inequality.
(ii) The inequality (ii) takes the following form:
ï ò
s2 − 16Rr + 5r2 R 4R(R − 2r) R
k≤ 2 2
. = 1 − 2 2
. = F (s, R, r),
s − 3r − 12Rr r s − 3r − 12Rr r

with F decreasing in s. By a direct computation one finds:

(1 + x2 )2 ′ x(x2 + 1)(3 − x2 )
f (x) = , f (x) = , x ∈ (0, 1).
4(1 − x2 ) 2(1 − x2 )2
1
Therefore, f is an increasing function on (0, 1). We get k = inf f (x) = f (0) = .
x∈(0,1) 4
Finally, in any triangle ABC we have the inequality
1r
(20) a2 + b2 + c2 ≥ 24Rr − 12r2 + [(a − b)2 + (b − c)2 + (c − a)2 ],
4R
which represents a refinement for the left side of the Gerretsen inequality.

References

1. M. Tetiva – The best inequality of this type... (in Romanian), Recreaţii matematice,
VIII (2006), 120-123.
2. A. Cipu – Optimal reverse Finsler-Hadwiger inequalities, Gazeta Matematică (Bu-
charest), s.A., 3-4/2012, 61-68.

12
An inequality on the medians of a triangle
Vasile JIGLĂU1
Abstract. In this Note a proof of the geometric inequality (1) below is provided.
Keywords: median, circumradius, inradius, Brocard′ s angle.
MSC 2010: 51M04.

In this Note we shall give a proof of the following geometric inequality:


m2a + m2b + m2c R
(1) ≤
2(ma mb + mb mc + cc ma ) − (m2a + m2b + m2c ) 2r
with the usual notations in a triangle.
1. Preliminaries. First, we establish some results to be used later.
Let ABC be a triangle. Denote by ω its Brocard angle. The following properties
of the angle ω are well-known:
a2 + b 2 + c2 2S π
(2) (i) ctg ω = , (ii) sin ω = √ , (iii) ω ≤ ,
4S a b + b 2 c2 + c2 a2
2 2 6
π
and ω = for the equilateral triangles. For details we refer to the Chapter XVII in
6
[2].
Lemma 1. In any triangle we have
P 2 »
a
(3) P P 2 = t + t(t + 1),
2 ab − a
where
R
t = ctg2 ω/(4 + 1).
r

Proof. Taking into account (2(i)) as well as the formulas


X X
a2 = 2(p2 − r2 − 4Rr), ab = p2 + r2 + 4Rr

one obtains that (3) is successively equivalent to:


P 2 P P 2  P
a ( a2 )2 r a r ( a2 )2 r
= 2
. + ( 2
+ 1),
4r(4R + r) 16S 4R + r 4S 4R + r 16S 4R + r
P 2  
p a r (p2 − r2 − 4Rr)2
= + + ,
4R + r 4p(4R + r) 4R + r (4R + r)2 .4p2
X »
4p2 = a2 + 2 4rp2 (4R + r) + (p2 − r2 − 4Rr)2 ,
X X »
( a2 )2 − a2 = 2 (p2 + r2 + 4Rr)2 ,
X
ab = p2 + r2 + 4Rr.
1 jiglau.vasile@yahoo.com

13
Hence (3) holds.
Lemma 2. In any triangle the following inequalities hold:

1 4R2 − 3Rr + 6r2 R2 3R 3


(4) 2 ≤ 2
= 2
− + ,
sin ω 4r r 4r 2
2 2 2
4R − 3Rr + 2r R 3 R 1
(5) ctg2 ω ≤ = 2 − + .
4r2 r 4r 2
Proof. Taking into account that
X X X
a2 b 2 = ( ab)2 − 2abc a = (p2 + r2 + 4Rr)2 − 16Rrp2

one gets

1 (p2 + r2 + 4Rr)2 − 16rp2


(6) 2 = .
sin ω 4r2 p2
It is clear that (4) follows from the double inequality that will be proved below.

1 p2 + 3r2 − 7Rr 4R2 − 3Rr + 6r2


(7) ≤ ≤ .
sin2 ω 4r2 4r2
Indeed, according to (6), the first inequality (7) reduces to (p2 + r2 + 4Rr)2 −
16rp2 ≤ p2 (p2 + 3r2 − 7Rr) and this in turn becomes r3 + 16R2r + 8Rr2 ≤ p2 (r + R).
Since by the second Gerretsen′ s inequality 16Rr − 5r2 ≤ p2 [3], it suffices to show that
r3 + 16R2 r + 8Rr2 ≤ (r + R)(16Rr − 5r2 ). By some algebra this inequality reduces
to the Euler inequality 2r ≤ R. Thus the first inequality (7) holds.
The second inequality (7) easily follows by using the first Gerretsen′ s inequality:
p ≤ 4R2 + 4Rr + 3r2 [3]. Indeed, p2 + 3r2 − 7Rr ≤ (4R2 + 4Rr + 3r2 ) + 3r2 − 7Rr =
2

4R2 − 3Rr + 6r2 . Concluding, the proof of the inequality (4) is complete.
1
The identity ctg2 ω = − 1 help us to easily derive the inequality (5) from
sin2 ω
(4).

Remark. A remarkable consequence of the preceding results is the well-known


inequalities
1 R
(8) 2≤ ≤ .
sin ω r
(the left-side inequality follows from (2(iii)) and the right-side inequality follows from
1 4R2 − 3r(R − 2r) 4R2
(4): 2 ≤ 2
≤ etc.). In [1] one finds a refinement of the
sin ω 4r 4r2
right-side inequality (8).

2. Proof of the inequality (1). Let Am Bm Cm be the triangle whose sides


have the lengths respectively equal to the lengths ma , mb , mc of the medians of the
triangle ABC and let Rm , rm , Sm , ωm be entities easy to understand associated to it.

14
A well-known and veryP useful property says that ω = ωm . Indeed, by (2(i)) we
P 2 3
ma a2
have ctg ωm = = 4 3 = ctg ω and (2(iii)) implies ω = ωm .
4Sm 4. 4 S
Applying the Lemma 1 to the triangle Am Bm Cm one obtains
P 2 »
ma
P P 2 = tm + tm (tm + 1),
2 ma mb − ma
where
Rm Rm
tm = ctg2 ωm /(4 + 1) = ctg2 ω/(4 + 1).
rm rm
Therefore, the inequality (1) reduces to
» R
(9) tm (tm + 1) ≤ − tm .
2r
Å ã
R 2 R Rm
We need to show that − tm ≥ 0 or, equivalently, ctg ω ≤ 4 + 1 or
2r 2r rm
1 R Rm 1R
2 ≤2 . + + 1.
sin ω r rm 2 r
R 1 Rm 1
The last inequality is true since from (8) it follows that ≥ and ≥
r sin ω rm sin ω
1 1 1 1 1
and it is obvious that we have 2 ≤2 + + 1.
sin ω sin ω sin ω 2 sin ω
Squaring, (9) can be written in the following equivalent forms:
Å ã
2 1 R2 R 2 R 1 R2
tm + tm ≤ − t m + t m ⇔ + 1 t m ≤ .
4 r2 r r 4 r2
Thus the inequality (1) is equivalent to
Å ãï 2 Å ãò
2 1 Rm R R
(10) ctg ω ≤ 4 +1 / +1 .
4 rm r2 r
R Rm
Case 1. Assume that ≥ . Obviously, we have also
r rm
Å ã Å ã
R2 R Rm2
Rm
/ +1 ≥ 2 / +1 .
r2 r rm rm
Thus we can write
Å ãï 2 Å ãò Å ãï 2 Å ãò
1 Rm R R 1 Rm Rm Rm
(11) 4 +1 / + 1 ≥ 4 + 1 / + 1 .
4 rm r2 r 4 rm 2
rm rm
On the other hand, according to (5), we have
2
Rm 3 Rm 1
(12) ctg2 ω = ctg2 ωm ≤ 2
− + .
rm 4 rm 2

15
By (10),(11) and (12) it follows that the inequality (10) is true if the following in-
equality holds:
2 Å ãï 2 Å ãò
Rm 3 Rm 1 1 Rm Rm Rm
(13) 2
− + ≤ 4 +1 2
/ +1 .
rm 4 rm 2 4 rm rm rm

Rm
Setting = u, (13) takes the form
rm
Å ã
2 3 1 1
(14) u − u+ (u + 1) ≤ (4u + 1)u2 .
4 2 4

Next, (14) reduces to u ≥ 2, that is Rm ≥ 2rm (Euler). Thus, the inequality (1) is
true in the considered case.
R Rm
Case 2. Assume that ≤ . Then
r rm
Å ãï 2 Å ãò Å ãï 2 Å ãò
1 Rm R R 1 R R R
4 +1 / + 1 ≥ 4 + 1 / + 1
4 rm r2 r 4 r r2 r

and from (5) it follows that

R2 3R 1
ctg2 ω ≤ 2
− + .
r 4r 2
The inequality (10), hence and (1) will be true if we show that
ï 2 Å ãò
R2 3R 1 1 R R R
(15) − + ≤ (4 + 1) 2 / +1 .
r2 4r 2 4 r r r

We notice that (15) is just the inequality (13) written for the triangle ABC. Thus
it holds good and the inequality (1) is completely proved.

References

1. V. Jiglău – An inequality involving medians and altitudes (in Romanian), Recreaţii


matematice, nr.2/2016, 112-114 (in Romanian).
2. T. Lalescu – Geometry of the Triangle (in Romanian), Editura Tineretului,
Bucharest, 1958.
3. D.S. Mitrinović, J.E. Pec̆arić, V. Volenec – Recent Advances in Geometric
Inequalities, Kluwer Academic Publishers, Dordrecht/Boston/London, 1989.

16
An application of Pell′ s equation
Gheorghe IUREA1

Abstract. In this Note it is shown that the number 121 is the only square number with the
property that the product of its figures is a prime number.
Keywords: prime number, square number, Pell equation.
MSC 2010: 11D09.

The main purpose of this Note is to solve the following problem as well as a
problem related to it. The notions and the results to be used below can be found in
[1], [2], [3], [4].
Problem 1. Determine the square numbers with the property that the product of
their figures is a prime number.
Solution. If the number a1 a2 ...an is a square number and the product a1 ·a2 ·...·an
of its figures is a prime number then only one of its figures can be 2, 3, 5 or 7 and the
other ones are 1.

The following fact is well-known and easy to be checked.


Lemma. If the number a1 a2 ...ak is square number and ak is an odd figure, then
ak−1 is an even figure.

Therefore, it does not exist square numbers whose all figures to be odd. The
figures of the number a1 a2 ...an can be 1 and only one equal to 2. Observe that the
case an = 2 is not possible. Thus, our problem reduces to determine all squares
having the form 11...121 (n figures).
If d = 11...121 is a square number, then the number 9d = 100...089 = 10n + 89 is
also a square number. Let us determine the natural numbers n, m such that

(1) 10n + 89 = m2 .

If n = 2t the equation (1) can be written in the form (m − 10t )(m + 10t ) = 89
and it follows that m − 10t = 1 and m + 10t = 89, a contradiction. Hence n is odd,
n = 2t + 1. In this case we write the equation (1) in the form
2
(2) m2 − 10 · 10t = 89.

Therefore, we may regard the pair (m, 10t ) as solution of the Diophantine equation

(3) a2 − 10b2 = 89.

We seek to express the solutions of the equation (3) as functions of the solutions
of the Pell equation:

(4) x2 − 10y 2 = 1.
1 Profesor, Liceul Teoretic ,,Dimitrie Cantemir”, Iaşi; iurea g@yahoo.com

17
It is known that the natural solutions of the equation (4) are the pairs (xk , yk ),
where
1 hÄ √ äk Ä √ äk i
xk = 19 + 6 10 + 19 − 6 10 ,
2
(5)
1 hÄ √ äk Ä √ äk i
yk = √ 19 + 6 10 − 19 − 6 10 , k ∈ N.
2 10

Therefore, we have:

(6) xk = Ck0 19k + Ck2 19k−2 62 10 + ...


(7) yk = Ck1 19k−1 6 + Ck3 19k−3 63 10 + ...

Let (a, b), a, b > 0, a solution of the equation (3). Then


√ Ä √ äÄ √ äp
(8) Ap + Bp 10 = a + b 10 19 − 6 10 , p ∈ N

is a strictly decreasing sequence that converges to zero. By a direct computation it


comes out that for√every p ∈ N√the pair (Ap , Bp ) is a solution of the equation (3).
Since A0 + B0 10 = a + b 10 > 1, there exists an unique k ∈ N such that
√ √
Ak+1 + Bk+1 10 < 1 ≤ Ak + Bk 10.
√ Ä √ äÄ √ ä
From (8) it follows: Ak+1 + Bk+1 10 = Ak + Bk 10 19 − 6 10 < 1. Hence
√ √
Ak + Bk 10 < 19 + 6 10 < 38.
On the other hand, since (Ak , Bk ) is a solution of (3), we have:
Ä √ äÄ √ ä
Ak + Bk 10 Ak − Bk 10 = 89.

From here we infer that 2 < Ak − Bk 10 ≤ 89. By adding side by side the last two
inequalities one obtains 3 < 2Ak < 127, hence Ak ∈ {2, 3, ..., 63} .
Taking into account the equation

A2k − 89
Bk2 = ,
10
we conclude that the solutions (Ak , Bk ) are only the pairs (27, −8) and (33, −10).
Using (8) we get
√ Ä √ äÄ √ äk
(9) a + b 10 = 27 − 8 10 19 + 6 10

or
√ Ä √ äÄ √ äk
(10) a + b 10 = 33 − 10 10 19 + 6 10 .

18
Ä √ äk √
As from (5) we infer that 19 + 6 10 = xk +yk 10, it follows that the solutions
(a, b) of the equation (3) are given by

(11) a = 27xk − 80yk , b = 27yk − 8xk

or

(12) a = 33xk − 100yk , b = 33yk − 10xk .

Thus, there exists k ∈ N such that

(13) 10t = 27yk − 8xk

or

(14) 10t = 33yk − 10xk .

From (6) it follows that xk ≡ 1 (mod 3) , and from (14) it follows xk ≡ −1 (mod 3) ,
hence 1 ≡ −1 (mod 3) , a contradiction. Therefore, (14) does not hold true.
.
For t ≥ 3, from (13) it follows that yk ..8. Taking into account (7), we infer
. .
Ck1 19k−1 6..8, hence k ..4, i.e. k = 4q, q ∈ N.
.
For q even, x is odd and y ..16. Using (13), for t ≥ 4 we arrive at a contradiction.
k k
Thus q is odd.
k
Taking xk , yk modulo 19, from (6) and (7) we obtain: xk ≡ Ckk 6k 10 2 ≡ 64q 102q ≡
3602q ≡ (−1)2q ≡ 1(mod 19) and yk ≡ 0(mod 19). From (13), it follows that

(15) 10t ≡ 11(mod 19).

Calculating the powers of 10 modulo 19, from (15) we find that t = 18s + 6, s ∈ N.
The initial equation (1) reduces to

(16) 1036s+13 + 89 = m2 .

According to Fermat′ s Little Theorem we have 1012 ≡ 1(mod 13). Thus, from (16) we
get m2 ≡ 10 + 89(mod 13), that is m2 ≡ 8(mod 13), an equation that has no solutions.
Therefore, the equation (16) has no solutions for t ≥ 4.
It remains to consider t ≤ 3. It comes out that only t = 1 is convenient, hence
n = 3, and the square number searched for is 121.
Concluding, the Problem 1 is solved.

The next problem, related to the Problem 1 is much easier.


Problem 2. Determine square numbers that have n figures (n ≥ 2) equal to 1
and one equal to a figure a.
Solution. According to Lemma above, do not exist square numbers having only
odd figures. Thus, we have no solutions for a odd. If a is 0, 4 or 8, then a can not

19
be the last figure. Contrary, the searched numbers are divisible by 2 but are not
divisible by 4. Again by Lemma above, the figure a will be the penultimate figure.
As 11...1a1 = 11...1000 + 1a1 = M8 + 5 and the square of an odd number is M8 + 1,
it follows that there are no solutions for a ∈ {0, 4, 8} .
For a = 6, we have to analyse the numbers 11...16 and 11...161. We have: 11...16 =
4 · 277...79 = 4 · (M4 + 3) hence it is not a square number and 11...161 is M11 + 6 or
M11 + 7 and also it is not a square number since it has no the form M11 + r with
r ∈ {0, 1, 3, 4, 5, 9}.
For a = 2, reasoning as in the preceding case we find the square number 121.
Concluding, there exists only a square number with the property of the statement
of the Problem 2.

References

1. I. Cucurezeanu – Squares and Cubes of Integer Numbers (in Romanian), GIL,


Zalău, 2007.
2. A. Engel – Problem-Solving Strategies, Springer-Verlag, New York Inc., 1998.
3. L. Panaitopol, D. Şerbănescu – Problems of Number Theory and Combinatorics
for Junior (in Romanian), GIL, Zalău, 2003.
4. L. Panaitopol, A. Gica – Problems of Arithmetics and Number Theory, Ideas and
Solution Methods (in Romanian), GIL, Zalău, 2006.

ERATĂ
I. Recreaţii Matematice, nr. 1/2016. La pp. 39-40, ı̂n articolul O remarcă asupra
verificării rădăcinilor unor ecuaţii iraţionale – A. Vernescu, numerotarea formulelor
s-a făcut greşit, cu numerele de la 3 la 9. Se va face modificarea n −→ n − 2, n ∈ 3, 9,
pentru ca trimiterile din text să fie adresate corect formulelor.
II. Recreaţii Matematice, nr. 2/2016. La pp. 124-127, ı̂n articolul Obţinerea
relaţiei de recurenţă pentru un anumit şir de integrale definite– A. Vernescu, nu-
merotarea formulelor s-a făcut greşit cu numerele de la 19 la 30. Corectarea se face
prin modificarea n −→ n − 18, n ∈ 19, 30.

La 27 ianuarie 2017, o persoană respectabilă a fost ı̂ntrebată ce vârstă are.


– M-am născut ı̂ntr-un an pătrat perfect şi vârsta mea este un pătrat perfect.
– Mulţi ani fericiţi!
Aflaţi anul naşterii şi vârsta persoanei.
Neculai Stanciu, Titu Zvonaru
(Răspuns la p. 37 )

20
În legătură cu o identitate ı̂n triunghi
din revista Curierul Matematic (1925-1928)
Marin CHIRCIU1

Abstract. In this Note, by use of the identity (∗) as a starting point, the author establishes a
number of geometric inequalities.
Keywords: altitude, perimeter, inradius, circumradius, Gerretsen′ s inequality, Hölder′ s in-
equality.
MSC 2010: 51M04.

În Curierul Matematic, C. I. Ionescu-Bujor [1] propune următoarea identitate


geometrică valabilă ı̂n orice triunghi ABC (notaţii uzuale):

a b c
(∗) · ctg A + · ctg B + · ctg C = 2.
ha hb hc
P a P a cos A 1 P 2 2R RP
Soluţie. · ctg A = a · · = a · · cos A = a cos A =
ha 2S sin A 2S a S
R 2rp
· = 2.
S R

În acest articol sunt discutate identităţi şi inegalităţi pentru sume asemănătoare
cu cele din (∗). Considerăm că sunt cunoscute următoarele inegalităţi geometrice
2
ı̂n triunghi, frecvent utilizate mai jos: 1) E: R ≥ 2r (Euler), √ 2) G: 16Rr − 5r ≤
√ 3 3
p2 ≤ 4R2 + 4Rr + 3r2 (Gerretsen), 3) M: 3 3r ≤ p ≤ R (Mitrinović), 4) D:
√ 2
√ P1 3
4R + r ≥ 3p,. 5) L: ≥ (Leuenberger); menţionăm că ı̂n oricare dintre
a R
ele avem egalitate dacă şi numai dacă triunghiul este echilateral. De asemenea, vor
fi utilizate şi inegalităţile algebrice: 1) CBS - Cauchy-Buniakovski-Schwarz, 2) C -
Åk=n ãm+1 Åk=n ãm
P xm+1
k=n
k P P
Cebâşev, 3) H - Hölder, 4) R: m ≥ xk / yk , xk , yk , m ∈ R+
k=1 y k k=1 k=1
(Radon).
P a
a) Sume de forma · f (A), unde f notează aici, ca şi peste tot ı̂n
ha
articol, una dintre funcţiile trigonometrice.
P a
1. · tg A ≥ 6, ı̂n orice triunghi ascuţitunghic. Într-adevăr, avem:
ha
X a X a2 sin A 1 X a3 H 1 (a + b + c)
3
· tg A = · = ≥ P =
ha 2S cos A 4RS cos A 4RS 3 cos A

8p3 R 2p2 G 2 16Rr − 5r2 2 (16R − 5r) E
· = ≥ = ≥ 6.
12Rrp R + r 3r (R + r) 3r (R + r) 3 (R + r)
1 Profesor, Colegiul Naţional ,,Zinca Golescu”, Piteşti; marin.chirciu@yahoo.com

21
P a
2. · sin A ≥ 3, ı̂n orice triunghi. Avem:
ha
X a 1 X 3 p2 − 3r2 − 6Rr G 5R − 4r E
· sin A = a = ≥ ≥ 3.
ha 4RS 2Rr R

P a 9R2
3. · cos A ≤ , ı̂n orice triunghi ascuţitunghic.
ha 4rp
X a 1 X 2 C 1 1 X 2X 1 1 X 2 9R2
· cos A = a cos A ≤ · a cos A ≤ · a ≤ .
ha 2S 2rp 3 2rp 2 4rp
P a √
4. · sec A ≥ 4 3, ı̂n orice triunghi ascuţitunghic.
ha
P 2 √
X a 1 X a2 CBS 1 ( a) 2p R M 6 3R E √
· sec A = ≥ ·P = · ≥ ≥ 4 3.
ha 2S cos A 2rp cos A r R+r R+r
P a 2R
5. ·cosec A = ≥ 4, ı̂n orice triunghi. Se verifică printr-un calcul simplu.
ha r
P a
b) Sume de forma ·f (B)f (C) . Următoarele şase inegalităţi sunt consecinţe
ha
P a P a
imediate ale celor anterioare, deoarece · f (B)f (C) = f (A) f (B)f (C) · ·
ha ha
1
.
f (A)
P a Q
6. · ctg B ctg C ≥ 6 ctg A, ı̂n orice triunghi ascuţitunghic.
ha
P a Q √
7. · tg B tg C = 2 tg A ≥ 6 3, ı̂n orice triunghi ascuţitunghic.
ha √
P a p 3 3
8. · sin B sin C = ≤ , ı̂n orice triunghi.
ha R 2 √
P a √ Q 3î 2
ó
9. · cos B cos C ≥ 4 3 cos A = 2 p2 − (2R + r) , ı̂n orice triunghi
ha R
ascuţitunghic.
P a 9R4
10. · sec B sec C ≤ î 2
ó , ı̂n orice triunghi ascuţitunghic.
ha rp p2 − (2R + r)
P a 6R2
11. · cosec B cosec C ≥ , ı̂n orice triunghi.
ha rp

P ha
c) Sume de forma · f (A).
a
P ha 3
12. · ctg A ≥ , ı̂n orice triunghi.
a 2
X ha X 1 cos A X 1 b 2 + c2 − a2
· ctg A = 2S · = 4RS · =
a a2 sin A a3 2bc

22
X b 2 + c2 − a2 Å ã Å ã
1 X a2 + b 2 + c2 1 X 2 X 1 3
= − 2 = a · − 6 ≥ .
2a2 2 a 2 2 a 2 2
P ha 9
13. · tg A ≥ , ı̂n orice triunghi ascuţitunghic.
a 2
X ha X 1 sin A SX 1 S 9 S 9R 9
· tg A = 2S 2
· = ≥ ·P = · = .
a a cos A R a cos A R a cos A R 2S 2

P ha p2 + r2 + 4Rr  r 2
14. · sin A = 2
≥9 , ı̂n orice triunghi. Avem:
a 4R R
X ha X 1 S X1 S p2 + r2 + 4Rr
· sin A = 2S 2
· sin A = · = · =
a a R a R 4Rrp

p2 + r2 + 4Rr G 5Rr − r2 E  r 2
≥ ≥9 ,
4R2 R2 R
sau rezolvarea:

X ha S X 1 L rp 3 M  r 2
· sin A = · ≥ · ≥9 .
a R a R R R

P ha p4 − 8Rrp2 − r2 (4R + r)2 3 3 r
15. · cos A = ≥ · , ı̂n orice triunghi.
a 8R2 rp 2 R
X ha Å ã
S X b 2 + c2 − a2 1 X a2 + b 2 + c2
· cos A = = − 2a =
a abc a 4R a
Å ã
1 X 2 X1
= a · − 4p =
4R a
ï ò
1 2 2
 p2 + r2 + 4Rr
2 p − r − 4Rr · − 4p =
4R 4Rrp
2
p4 − 8Rrp2 − r2 (4R + r) G
= ≥
8R2 rp
2  2
16Rr − 5r2 − 8Rr 16Rr − 5r2 − r2 (4R + r)
=
8R2 rp
 √
r 28R2 − 32Rr + 6r2 (+) 3 3 r
= ≥ · ,
2R2 p 2 R
√ D
deoarece (+) ⇔ 28R2 −32Rr+6r2 ≥ 3R·p 3 ⇐ 28R2 −32Rr+6r2 ≥ 3R (4R + r) ⇔
E
(R − 2r) (16R − 3r) ≥ 0.
P ha 6rp
16. · sec A ≥ , ı̂n orice triunghi ascuţitunghic.
a R (R + r)
P 1 2 Ç √ å2
X ha X 1 1 CBS L 3 R 6rp
a
·sec A = 2S · ≥ 2S · P ≥ 2S · = .
a a2 cos A cos A R R+r R (R + r)

23
P ha 6r
17. · cosec A ≥ , ı̂n orice triunghi. Avem:
a R
X ha X 1 R (1 + 1 + 1)4 1 M 6r
· cosec A = 4RS 3
≥ 4rpR · 3 = 81Rr · 2 ≥ .
a a (a + b + c) 2p R

P ha
d) Sume de forma · f (B)f (C). Următoarele inegalităţi sunt consecinţe
a
ale celor de la numerele 12-17. √
P ha 3Q 9 3
18. · tg B tg C ≥ tg A ≥ , ı̂n orice triunghi ascuţitunghic.
a 2 2
P ha 9Q
19. · ctg B ctg C ≥ ctg A, ı̂n orice triunghi ascuţitunghic.
a 2
P ha 6r Q 3r2 p
20. · sin B sin C ≥ sin A = , ı̂n orice triunghi.
a R R3 î ó
P ha 6rp Q 3rp p2 − (2R + r)2
21. · cos B cos C ≥ cos A = , ı̂n orice
a R (R + r) 2R3 (R + r)
triunghi. √ √
P ha 3 3 r Q 3 3 r
22. · sec B sec C ≥ · · sec A ≥ · , ı̂n orice triunghi.
a 2 R 16 R
P ha 2 2
p + r + 4Rr √
23. · cosec B cosec C = ≥ 2 3, ı̂n orice triunghi.
a 2rp
X ha Y X ha 14 2R2 p2 + r2 + 4Rr
· cosec B cosec C = cosec A · · sin A = · =
a a rp 4R2

p2 + r2 + 4Rr (++) √
≥ 2 3,
2rp
√ D
deoarece avem: (++) ⇔ p2 + r2 + 4Rr ≥ 4r · p 3 ⇐ p2 + r2 + 4Rr ≥ 4r · (4R + r) ⇔
G
p2 ≥ 12Rr + 3r2 ⇐ p2 ≥ 16Rr − 5r2 ≥ 12Rr + 3r2 (ultima inegalitate fiind, după
calcul, R ≥ 2r).

Observaţie. În toate inegalităţile de mai sus avem egalitate dacă şi numai dacă
triunghiul este echilateral.

Bibliografie

1. C.I. Ionescu-Bujor – Problema 62, Curierul Matematic, vol. I, 8/1925.


2. O. Bottema, R.Ž. Djordjević, R.R. Janić, D.S. Mitrinović, P.M. Vasić –
Geometric Inequalities, Groningen, 1969.
3. D. Brânzei, G. Popa, M. Chirciu, Gh. Iurea, J. Grigoraş – Geometrie şi
Trigonometrie, Editura Tiparg, Piteşti, 2004.
4. M. Chirciu – Inegalităţi geometrice, de la iniţiere la performanţă, Editura Paralela
45, Piteşti, 2016.

24
Conice şi cubice ı̂n probleme elementare
de loc geometric
Ştefan DOMINTE1
Abstract. In this Note, a number of simple problems are presented to support the idea that
conic and cubic curves can frequently occur in elementary issues of Euclidean geometry.
Keywords: median, altitude, angle bisector, perpendicular bisector, locus.
MSC 2010: 51M04.

Este cunoscut faptul că problemele de loc geometric pot conduce la conice, cubice
etc., chiar şi atunci când figura de pornire este extrem de simplă. Vom ilustra această
idee considerând figuri formate doar din câteva drepte şi obţinând locuri geometrice
de regulă conice sau cubice şi doar accidental puncte, drepte sau cercuri.

Problema 1. Fie B şi C două puncte fixe ı̂n plan, ∆ o dreaptă fixă paralelă
cu BC, A un punct fix pe ∆ şi P intersecţia cevienelor BB ′ ( B ′ ∈ AC) şi CC ′
( C ′ ∈ AB). Să se determine locul geometric al punctului P , atunci când A este
mobil pe ∆, ştiind că ceviana BB ′ (cât şi, independent, ceviana CC ′ ) are una dintre
proprietăţile: este mediană (m, pe scurt), este ı̂nălţime (h) sau este bisectoare (b).

Se impun a fi studiate un număr de şase cazuri relativ la cuplul de ceviene


(BB ′ , CC ′ ): 1) m-m, 2) m-h, 3) m-b, 4) h-h, 5) h-b, 6) b-b. Cazurile h-m, b-m,
b-h sunt simetrice faţă de cazurile 2), 3), respectiv 4), locurile respective fiind curbe
simetrice ı̂n raport cu mediatoarea segmentului BC.
Notăm cu a lungimea segmentului BC şi cu d distanţa dintre dreptele BC şi ∆.
Locul punctului P depinde de parametrii a şi d, uneori fiind necesară o discuţie după
valorile lor.

1. Cazul mediană-mediană (m-m). Este un caz trivial. Dacă BB ′ şi CC ′


sunt mediane ı̂n triunghiul ABC, atunci P este centrul de greutate al lui şi va fi la
d
distanţa de dreapta BC. Ca urmare, locul punctului P este dreapta paralelă cu
3
d
BC, la distanţa de BC şi situată faţă de BC de partea ı̂n care se află ∆.
3
2. Cazul mediană-ı̂nălţime (m-h). Alegem un reper cartezian cu originea ı̂n
B ca ı̂n Fig.1. Notăm cu λ ∈ R abscisa punctului A mobil pe ∆. Avem: A(λ, d),
a+λ d
B(0, 0), C(a, 0) şi B ′ ( , ). Ecuaţiile medianei BB ′ şi ı̂nălţimii CC ′ sunt:
2 2
d λ
(BB ′ ) : y = x, (CC ′ ) : y = − (x − a).
a+λ d
1 Elev, cl. a XII-a, Liceul Internaţional de Informatică, Bucureşti; stef dominte@yahoo.com

25
dy
Din a doua ecuaţie avem că λ = − şi, prin ı̂nlocuire ı̂n prima ecuaţie, obţinem
x−a
dy
ecuaţia locului geometric: y(a − ) = dx sau
x−a
(1) d(x2 + y 2 ) − axy − adx + a2 y = 0.
Evident, (1) este ecuaţia unei conice. Se verifică direct că ea trece prin punctele
B şi C. Cum
d −a/2 a a
δ= = (d + )(d − ),
−a/2 d 2 2
conica este o elipsă, dacă δ > 0 ⇐⇒ a < 2d, o hiperbolă, dacă δ < 0 ⇐⇒ a > 2d sau
o parabolă, dacă a = 2d. În Fig.1, cazul a < 2d, locul geometric este o elipsă.

3. Cazul mediană-bisectoare (m-b). Alegem acelaşi sistem de axe (Fig.2).


d
Ca şi mai sus, ecuaţia medianei este (BB ′ ): y = x. Cum dreptele CB şi
a+λ
d
CA au ecuaţiile y = 0 şi y = (x − a), bisectoarea CC ′ are ecuaţia (CC ′ ):
λ−a
|dx − (λ − a)y − ad|
y= p .
d2 + (λ − a)2
Ecuaţia locului punctului P se obţine eliminând λ din sistemul ecuaţiilor
d |dx − (λ − a)y − ad|
y= x, y= p .
a+λ d2 + (λ − a)2
dx dx − 2ay
Din prima ecuaţie rezultă că a + λ = şi apoi λ − a = . Ridicând la pătrat
y y
a doua ecuaţie şi substituind λ − a cu expresia găsită, avem:
dx − 2ay 2
y 2 [d2 + ( ) ] = [dx − (dx − 2ay) − ad]2
y

26
şi, după calcule de rutină, obţinem ecuaţia locului geometric:

(2) d(x2 + y 2 ) − 4axy + 4a2 y − a2 d = 0,

o conică care
trece prin punctele C(a, 0) şi D(−a, 0). Pentru conica (2) avem δ =
d −2a
= (d + 2a)(d − 2a), deci avem elipsă, hiperbolă, sau parabolă după cum
−2a d
d > 2a, d < 2a, respectiv d = 2a.
Observăm că punctul P , din modul cum este definit, se află ,,deasupra” dreptei

BC; ca urmare, locul punctului P este arcul CD al conicei (2) situat ,,deasupra”

dreptei BC. În Fig.2, cazul d > 2a, locul lui P este arcul de elipsă CD. Dacă
considerăm şi punctul P ′ de intersecţie a medianei BB ′ cu bisectoarea exterioară a

unghiului C, “ constatăm că punctul P ′ parcurge arcul CD aflat ,,sub” BC (Fig. 2,
linia ı̂ntreruptă). Explicaţia vine de la faptul că prin ridicarea la pătrat efectuată mai
sus s-a pus ı̂n joc şi bisectoarea exterioară.

4. Cazul ı̂nălţime-ı̂nălţime (h-h). Faptul că intră ı̂n joc o singură linie im-
portantă a triunghiului face ca acest caz să prezinte o ,,simetrie” ı̂n raport cu laturile
AB şi AC, iar ı̂n final locul geometric să fie o curbă simetrică faţă de mediatoarea
segmentului BC. Ca urmare, de data aceasta este avantajos să alegem un reper cu
originea ı̂n mijlocul segmentului BC şi ca axe dreapta BC (cu sensul de la B la C) şi
a a
mediatoarea segmentului BC. Avem: B(− , 0), C( , 0), A(λ, d), iar ı̂nălţimile BB ′
2 2
şi CC ′ au ecuaţiile:
a a
λ− a λ+ a
(BB ′ ) : y = − 2
(x + ), (CC ′ ) : y = − 2
(x − ).
d 2 d 2
a dy a dy
Din prima ecuaţie rezultă că λ − =− sau λ + = a − şi, ı̂nlocuind
2 x + a2 2 x + a2
dy a
ı̂n a doua ecuaţie, obţinem că dy = −(a − )(x − ), iar după calcule, ecuaţia
x + a2 2
locului geometric:

1 a2
(3) y= ( − x2 ),
d 4
adică o parabolă cu axa de simetrie x = 0 (mediatoarea segmentului BC), cu conca-
a2
vitatea spre y-ii negativi, ce trece prin punctele B şi C şi are vârful (0, ).
4d

5. Cazul ı̂nălţime-bisectoare (h-b). Revenind la reperul considerat ı̂n cazurile


2 şi 3 (v. şi Fig.3), ı̂nălţimea BB ′ şi bisectoarea CC ′ au ecuaţiile:

λ−a |dx − (λ − a)y − ad|


(BB ′ ) : y = − x, (CC ′ ) : y = p .
d d2 + (λ − a)2

27
dy
Procedând ı̂n acelaşi mod, obţinem mai ı̂ntâi λ − a = − şi apoi ecuaţia locului
x
punctului P :
d2 y 2 dy 2
y 2 [d2 + ] = (dx + − ad)2 ⇐⇒
x2 x
x3 + xy 2 − 2ax2 − 2ay 2 + a2 x = 0 ⇐⇒ (2a − x)y 2 = x(x − a)2 .
Mutând originea sistemului ı̂n punctul C prin translaţia x = a + t, ultima ecuaţie se
scrie ı̂n forma (a − t)y 2 = (a + t)t2 sau, echivalent,

a+t
(4) y = ±t .
a−t
Aceasta este ecuaţia strofoidei cu nodul t = 0 - punctul C - şi asimptota t = a sau
x = 2a - dreapta perpendiculară pe BC ı̂n simetricul punctului B faţă de C.
Similar cu situaţia din Cazul 3, punctul P ′ de intersecţie a ı̂nălţimii BB ′ cu
“ parcurge ramura strofoidei figurată ı̂ntrerupt
bisectoarea exterioară a unghiului C,
(Fig. 3).

6. Cazul bisectoare-bisectoare (b-b). Din motive de simetrie, vom alege un


reper cu originea ı̂n mijlocul segmentului BC (Fig.4), aşa cum s-a procedat ı̂n Cazul
4 (h-h).
Cum ecuaţiile dreptelor BC, AB, AC sunt:
d a d a
(BC) : y = 0, (AB) : y = a (x + ), (AC) : y = a (x − ),
λ+ 2 2 λ− 2 2
bisectoarele BB ′ şi CC ′ au următoarele ecuaţii:
|dx − (λ + a2 )y + ad2 | |dx − (λ − a2 )y − ad2 |
(BB ′ ) : y = » , (CC ′ ) : y = » .
2 a 2 2 a 2
d + (λ + 2 ) d + (λ − 2 )

28
Prin ridicare la pătrat, din aceste ecuaţii obţinem:
a2 d a2 d
a d(x2 − y 2 ) + adx + 4 a d(x2 − y 2 ) − adx + 4
λ+ = , λ− = .
2 2xy + ay 2 2xy − ay
a a
Înlocuind aceste expresii ı̂n identitatea λ + = (λ − ) + a şi efectuând calcule de
2 2
rutină, vom obţine ecuaţia locului:

a2 d
(5) (4x2 − a2 )y = 2d(x2 + y 2 ) − .
2
Aceasta reprezintă o cubică ce trece prin punctele B şi C şi este simetrică ı̂n
raport cu mediatoarea segmentului BC. Forma ei, trasată cu programul Geogebra,
este redată ı̂n Fig.4, linia continuă; linia ı̂ntreruptă ce completează cubica corespunde
bisectoarelor exterioare ı̂n vârfurile B şi C.

Enunţul Problemei 1 poate fi cu uşurinţă modificat ı̂n scopul obţinerii de noi


probleme ı̂nrudite. În acest sens, indicăm două direcţii posibil de urmat: antrenarea şi
altor linii remarcabile ale triunghiului (mediatoarea, simediana etc.) şi/sau ı̂nlocuirea
dreptei fixe ∆ paralelă cu BC cu alte curbe (cerc prin punctele B şi C, parabolă etc.).
Este de aşteptat, ı̂nsă, ca forma curbelor obţinute ca loc geometric să fie mult mai
complicată. Vom vedea acest lucru ı̂n cazul mediatoarei, pe care o alăturăm celor trei
linii deja considerate ı̂n Problema 1.

Problema 2. În condiţiile existente ı̂n Problema 1, să se determine locul geome-
tric al punctului P de intersecţie a mediatoarei laturii AB cu: mediatoarea, mediana,
ı̂nălţimea sau bisectoarea corespunzătoare laturii AC.

7. Cazul mediatoare-mediatoare (M-M). Acest caz este simplu şi poate fi


tratat sintetic. Totul se sprijină pe faptul că mediatoarele segmentelor AB şi AC
(mobile) se intersectează pe mediatoarea segmentului BC (fixă). Dacă aceasta din
urmă intersectează dreapta ∆ ı̂n punctul A0 şi O este centrul cercului circumscris
triunghiului A0 BC, atunci locul cerut este semidreapta [OA0 ), ce este parcursă de
două ori.

8. Cazul mediatoare-mediană (M-m). Considerăm sistemul de axe cu ori-


λ d
ginea ı̂n B utilizat anterior. Coordonatele mijloacelor laturilor AB şi AC sunt ( , )
2 2
λ+a d
şi ( , ), iar ecuaţiile mediatoarei şi medianei sunt:
2 2
d λ λ d
(M ) : y − = − (x − ), (m) : y = x.
2 d 2 λ+a
Eliminând λ din aceste ecuaţii obţinem:

(6) 2dx2 y − 2axy 2 + 2dy 3 − d2 x2 + 2adxy − (a2 + d2 )y 2 = 0

29
(cu excluderea punctului (0, 0)), care reprezintă o cubică. Graficul locului geometric,
realizat cu programul Geogebra, este prezentat ı̂n Fig. 5.

9. Cazul mediatoare-ı̂nălţime (M-h). Procedând ca ı̂n cazul precedent,


obţinem cubica

(7) x2 (2ax − d2 ) = (dy − ax)2 ,

al cărei grafic este redat ı̂n Fig. 6.

10. Cazul mediatoare-bisectoare (M-b). Locul geometric are ecuaţia

(8) 4x2 y = d(x2 + y 2 ),

cubică cu graficul dat ı̂n Fig. 7. Cubica este simetrică faţă de axa y-lor (perpen-
diculara ı̂n B pe BC), ramura cu linie ı̂ntreruptă fiind generată de punctul P ′ (de

intersecţie a mediatoarei cu bisectoarea exterioară a unghiului B).

Încheiem cu observaţia că este de aşteptat să obţinem rezultate interesante dacă
ı̂n Problemele 1 şi 2 dreapta ∆ pe care se mişcă punctul mobil A se ı̂nlocuieşte cu un
cerc ce trece prin punctele fixe B şi C.

30
Aspecte geometrice ale unei rozete
asociate unui triunghi
Vlad TUCHILUŞ, Răzvan Andrei MORARIU,
Robert ANTOHI1

Abstract. In this Note, a rosette is associated to an arbitrary triangle and the triangles with
regular associated rosettes are determined.
Keywords: triangle, circle, median, hexagon, rosette.
MSC 2010: 51M04.

Date cercurile C0 , C1 , . . . , Cn , vom spune că ele formează o rozetă dacă au raze
egale şi centrele cercurilor C1 , C2 , . . . , Cn se află pe C0 . În acest caz, C0 este numit
cercul central al rozetei, cercurile C1 , C2 , . . . , Cn - cercurile-petale ale rozetei, iar cen-
trul cercului C0 - centrul rozetei. Dacă poligonul determinat de centrele cercurilor
C1 , C2 , . . . , Cn este regulat vom spune că rozeta este regulată.
Fie Oi centrul cercului Ci , i = 0, n, şi r raza acestor cercuri. Evident, cercurile
Ci , i = 1, n, trec prin centrul O0 . Pentru i = 1, n, notăm cu Ti punctul diametral
opus punctului O0 ı̂n cercul Ci . Avem O0 Ti = 2r şi O0 M < 2r, ∀M ∈ Ci , M 6= Ti . Ca
urmare, cercul de centru O0 şi rază 2r este tangent cercului Ci ı̂n punctul Ti , i = 1, n,
deci circumscrie rozeta.
În această Notă vom asocia unui triunghi o rozetă cu şase cercuri-petale şi vom
preciza condiţiile ce trebuie impuse triunghiului pentru ca rozeta să fie regulată.
1. Fie ABC un triunghi oarecare. Notăm cu G centrul de greutate al triunghiului
şi cu A′ , B ′ , C ′ mijloacele laturilor BC, CA, respectiv AB. Construim simetricele
punctului G ı̂n raport cu A′ , B ′ , C ′ şi notăm aceste puncte cu A1 , B1 , respectiv C1 .
Relativ la latura BC, considerăm triunghiurile GBA1 , GCA1 şi notăm cu O1a , O2a
centrele cercurilor circumscrise lor, iar cu C1a , C2a aceste cercuri. Similar se introduc
cercurile C1b , C2b , C1c , C2c .
Lemă. 1) G este centru de simetrie pentru următoarele perechi de triunghiuri:
△GBA1 şi △GB1 A, △GCA1 şi △GC1 A, △GCB1 şi △GC1 B.
2) Avem: △GBA1 ≡ △A1 CG ≡ △B1 GC ≡ △GB1 A ≡ △AC1 G ≡ △BGC1 .
Demonstraţie. 1) Punctele A1 , B1 , C1 sunt simetricele vârfurilor A, B, C ı̂n ra-
2 2 2
port cu G, căci GA = GA1 = ma , GB = GB1 = mb , GC = GC1 = mc , unde
3 3 3
ma , mb , mc sunt lungimile medianelor triunghiului ABC. Afirmaţia rezultă imediat
din această observaţie.
2) Ţinând seama de punctul 1) şi de faptul că triunghiurile simetrice faţă de un
punct sunt congruente, deducem că este suficient să arătăm că △GBA1 ≡ △A1 CG ≡
△B1 GC. Aceste congruenţe sunt implicate de egalităţile următoare: GB = A1 C =
2 2 2
B1 G = mb , GA1 = A1 G = B1 C = ma , A1 B = GC = CG = mc , care rezultă
3 3 3
din paralelogramele GBA1 C şi B1 GA1 C.
1 Elevi,
cl. a X-a, Colegiul Naţional ,,C. Negruzzi”, Iaşi; vlad.tuchilus@yahoo.com,
morariu.andrei41ram@yahoo.com, antohir@yahoo.com

31
Propoziţia 1. Cercurile Cia , Cib , Cic , i = 1, 2, au următoarele proprietăţi:
(i) razele lor sunt egale - fie ̺ valoarea comună -, iar centrele lor sunt conciclice,
aflate pe cercul cu centrul ı̂n G şi de rază ̺;
(ii) cercurile din perechile C1a şi C2b , C1b şi C2c , C1c şi C2a au centrele simetrice
faţă de G (ı̂n particular sunt tangente ı̂n G);
(iii) hexagonul (H) al centrelor celor şase cercuri are laturile opuse paralele şi
egale.
Demonstraţie. (i) Cercurile au aceeaşi rază, deoarece sunt circumscrise la tri-
unghiuri congruente (punctul 2) al Lemei). Cum aceste triunghiuri trec prn G, rezultă
că razele lor este dată de ρ = GO1a = GO2a = GO1b = GO2b = GO1c = GO2c . De
aici rezultă că Oia , Oib , Oic ∈ C(G, ρ), i = 1, 2.
(ii) Afirmaţia decurge din punctul 1) al Lemei.
(iii) Conform punctului precedent, simetrica laturii O1a O2a a lui (H) ı̂n raport
cu G este latura O2b O1c . Ca urmare, aceste laturi sunt paralele şi egale. La fel se
procedează cu celelalte două perechi de laturi opuse ale hexagonului, ceea ce ı̂ncheie
demonstraţia.
Observaţii. 1) Din punctul (i) al propoziţiei precedente rezultă că putem asocia
oricărui triunghi rozeta formată din cercurile C(G, ρ), Cia , Cib , Cic , (i = 1, 2).
2) Raza rozetei asociate este dată de formula:
2 ma mb mc
(1) ρ= · ,
9 S
unde S notează aria triunghiului ABC.
Într-adevăr, să calculăm ρ ca rază a cercului circumscris triunghiului GBA1 , uti-
abc 2 2 2
lizând formula R = . Cum GB = mb , GA1 = ma şi BA1 = GC = mc , iar
4S 3 3 3

32
1
SGBA1 = SGBC = S, obţinem:
3

GB · GA1 · BA1 ( 2 )3 ma mb mc 2 ma mb mc
ρ= = 3 = · .
4SGBA1 4 9 S
S
3

3) Cercul C(G, ρ) este determinat ca poziţie şi mărime ı̂n raport cu △ABC, dar
cele şase cercuri-petale ale rozetei sunt determinate numai ca mărime de formula (1)
pentru poziţia centrelor lor pe C(G, ρ) având doar restricţia impusă de punctul (iii)
al Propoziţiei 1.
4) Dacă hexagonul (H) este circumscriptibil, atunci el este regulat (se ţine seama
de proprietăţile de simetrie (ii) şi (iii)).

2. Să notăm cu (R) rozeta asociată triunghiului ABC. Să vedem ı̂n ce condiţii re-
lative la △ABC rozeta (R) este regulată. Sunt necesare câteva rezultate pregătitoare.

Propoziţia 2. Următoarele afirmaţii sunt echivalente:


(i) cercurile C1a şi C2a au centrele unul pe altul (Fig. 2);
’ dintre medianele BB ′ şi CC ′ este de 60◦ sau 120◦ ;
(ii) unghiul BGC
(iii) hexagonul (H) are laturile O1a O2a şi O2b O1c egale cu ̺.
Demonstraţie. Vom ı̂ncepe cu câteva observaţii. Deoarece cercurile C1a şi C2a
sunt egale, centrele lor O1a şi O2a sunt simetrice ı̂n raport cu mijlocul A′ al corzii
comune GA1 , iar patrulaterul GO1a A1 O2a este romb cu latura egală cu ρ.
÷1 ≡ GCA
Din faptul că GBA ÷1 rezultă că aceste unghiuri sunt simultan ascuţite
sau obtuze, ceea ce impune centrelor O1a şi O2a ordinea O1a − A′ − O2a , respectiv
O2a − A′ − O1a .
Cercurile C1a şi C2a au centrele unul pe altul dacă şi numai dacă O1a O2a = ρ, ceea
ce este echivalent cu faptul că rombul GO1a A1 O2a are m(GO ÿ ÿ
1a A1 ) = m(GO2a A1 ) =
◦ Ÿ ⁄
120 (sau m(O1a GO2a ) = m(O1a A1 O2a ) = 60 ). ◦

(i)=⇒(ii). Presupunem că O1a O2a = ρ. Dacă are loc ordinea O1a − A′ − O2a ,
÷1 este ascuţit (Fig. 2), atunci ı̂n cercul C1a avem: m(GBA
adică unghiul GBA ÷1 ) =
1 ÿ ’ = 180◦ − m(GBA ÷1 ) = 120◦.
m(GO1a A1 ) = 60◦ . Ca urmare, m(BGC)
2
Dacă avem O2a −A′ −O1a , adică unghiul GBA ÷1 este obtuz (Fig. 3), atunci ı̂n cercul
C1a avem: m(GBA÷1 ) = m(GO ÿ ◦ ’
2a A1 ) = 120 , de unde rezultă că m(BGC) = 60 .

(ii)=⇒(i). Dacă m(BGC)’ = 120 , rezultă că m(GBA1 ) = 60 , deci centrele


◦ ◦

cercurilor sunt ı̂n ordinea O1a − A′ − O2a (Fig. 2). Ca urmare, m(GO ÿ 1a A1 ) =
÷1 ) = 120◦ şi deducem că O1a O2a = ρ, adică este verificată (i).
2m(GBA
’ = 60◦ , obţinem că m(GBA
Dacă m(BGC) ÷1 ) = 120◦ şi, deci, avem ordinea O2a −
A′ − O1a (Fig. 3). Urmează că m(GO ÿ ◦
2a A1 ) = 120 , echivalent cu O1a O2a = ρ, adică
cu (i).
(iii)⇐⇒ (i). Rezultă din faptul că O1a O2a = O2b O1c (punctul (ii), Propoziţia 1).

33
Observaţie. În condiţiile Propoziţiei 2, avem:

2 3
(2) ρ= ma ,
√9
3
(3) S= mb mc ,
3
(4) 9a = 4(m2b + m2c ± mb mc ),
2

(5) 5a2 = b2 + c2 ± 4 3S,

unde semnele + şi − corespund cu m(BGC) ’ = 120◦, respectiv m(BGC)’ = 60◦ .


Într-adevăr, din faptul că triunghiul O1a GO2a este echilateral de latură ρ şi
1
ı̂nălţime GA′ = ma , cu teorema lui Pitagora obţinem formula (2). Egalând ex-
3
presiile lui ρ date de (1) şi (2), obţinem fomula (3).
Aplicând teorema cosinusului ı̂n △BGC, avem:

4 2 4 2 4 ’
a2 = mb + mc − 2 · mb mc cos(BGC),
9 9 9
de unde, ţinând cont de punctul (ii) al Propoziţiei 2, rezultă imediat (4). Cu teorema
medianei şi formula (3), din (4) deducem (5).
Propoziţia 2 se referă la următoarele perechi de cercuri ale rozetei (R): (C1a , C2a ),
(C1b , C2b ), (C1c , C2c ). Pentru perechile (C2b , C1c ), (C2c , C1a ), (C2a , C1b ), obţinute din
precedentele prin simetrie ı̂n raport cu G, este valabil următorul rezultat similar cu
Propoziţia 2:
Corolar. Afirmaţiile următoare sunt echivalente:
(i) Cercurile C2b şi C1c au centrele unul pe altul;
(ii) Unghiul B÷′ GC ′ dintre medianele BB ′ şi CC ′ este de 60◦ sau 120◦ .

34
Demonstraţie. Se aplică Propoziţia 2 cercurilor C1a şi C2b şi se ţine seama de
÷
faptul că B ’ (opuse la vârf). Menţionăm că se poate da şi o demonstraţie
′ GC ′ ≡ BGC

directă procedând ca ı̂n demonstraţia Propoziţiei 2.


’ = 90◦ , care a fost evitat ı̂n secţiunea precedentă.
3. Vom examina cazul m(BGC)
Propoziţia 3. Următoarele afirmaţii sunt echivalente:
(i) cercurile C1a şi C2a coincid;
(ii) medianele BB ′ şi CC ′ sunt perpendiculare, i.e. m(BGC) ’ = 90◦ ;
(iii) hexagonul (H) degenerează ı̂n dreptunghiul (D) : A′ O1b A′′ O2c , unde A′′ este
mijlocul segmentului AG (Fig. 4).
Demonstraţie. (i)⇐⇒(ii). Condiţia (i) implică faptul că paralelogramul GBA1 C
este inscriptibil, adică este dreptunghi; aşadar m(BGC)’ = 90◦ .
Invers, dacă (ii) este verificată, atunci GBA1 C este dreptunghi, deci cercul C1a
trece prin C şi, ca urmare, coincide cu cercul C2a .
(ii)⇐⇒(iii). Dacă m(BGC)’ = 90◦ , atunci cercurile C1a şi C2a coincid. Evident,
centrele lor O1a şi O2a coincid ı̂n A′ . În consecinţă, simetricele lor faţă de G, adică
punctele O2b şi O1c , coincid ı̂n A′′ . Afirmaţia (iii) decurge imediat.
Invers, să presupune că (H) degenerează ı̂n dreptunghiul A′ O1b A′′ O2c . Utilizând
faptul că linia centrelor a două cercuri este perpendiculară pe coarda comună lor,
deducem că BG ⊥ O1a O2c şi CG ⊥ O2a O1b . Aşadar, medianele BB ′ şi CC ′ sunt
perpendiculare, adică (ii).

Observaţii. 1) Conform problemei M037 din [1], condiţia (ii) este echivalentă cu
fiecare dintre formulele:
(6) 5a2 = b2 + c2 ,
(7) 9a2 = 2(m2a + m2b + m2c )

35
3
(ceea ce se arată uşor cu teorema medianei şi faptul că ı̂n cazul nostru ma = a).
2
′ ′ 2 2 2
2) Dacă △ABC verifică condiţia BB ⊥ CC (sau 5a = b + c ) atunci rozeta
a a
(R) este degenerată, având numai patru cercuri-petale: C(A′ , ), C(A′′ , ), C1b şi C2c .
2 2
2 2
3) Dreptunghiul (D) are două laturi egale cu mb şi două egale cu mc , diago-
3 3
nalele sale sunt egale cu a, iar diagonala O1b O2c este paralelă cu BC (Fig. 4).
4) Rozeta (R) cu patru cercuri-petale este regulată dacă (D) este pătrat, adică

2
dacă şi numai dacă mb = mc sau b = c. În acest caz, latura pătratului este a.
′ ′
2
Cu …uşurinţă
… aflăm că triunghiul isoscel AB = AC şi BB ⊥ CC este triunghiul
5 5
(a, a, a).
2 2
4. Utilizând rezultatele obţinute ı̂n secţiunile anterioare, vom determina triun-
ghiurile a căror rozetă asociată este nedegenerată şi regulată.
Teoremă. Un triunghi are rozeta √ √asociată regulată dacă şi numai dacă este echi-
lateral sau isoscel cu laturile a, 7a, 7a.
Demonstraţie. Să presupunem că triunghiul ABC are rozeta (R) regulată.
Să considerăm un cerc-petală, de exemplu C1a , cel circumscris triunghiului GBA1 .
Alăturat triunghiului GBA1 sunt triunghiurile GCA1 şi GBC1 , ce au C2a şi C2c drept
cercuri circumscrise lor. Centrele O1a , O2a , O2c ale acestor cercuri au diferite poziţii
faţă de triunghiul ABC, funcţie de forma triunghiurilor GBA1 , GCA1 şi GBC1 . Vom
examina două dintre cazurile posibile, ı̂ntrucât pentru restul cazurilor se procedează
la fel.
Considerăm cazul O1a − A′ − O2a , O1a − B ′′ − O2c (Fig. 2). Rozeta (R) fiind
regulată, avem că O1a ∈ C2a şi O2a ∈ C1a . Conform Propoziţiei 3, unghiul BGC ’ este
◦ ◦
de 120 sau 60 . În cazul nostru, urmărind unghiurile marcate pe Fig. 2, obţinem că
’ = 120◦. Pe de altă parte, O1a ∈ C2c şi O2c ∈ C1a , implică m(GO
m(BGC) ◊ 2c B) = 120

şi apoi m(BC ÷ ◦ ÷ ′ ′ ◦ ’ ◦


1 G) = 60 , m(C GA ) = 120 şi m(CGA) = 120 . Aşadar, triunghiul
ABC verifică condiţiile m(BGC) ’ = m(CGA) ’ = 120 şi, ca urmare, este echilateral.

În cazul O2a − A′ − O1a , O1a − B ′′ − O2c (Fig. 3), din faptul că rozeta (R) este
regulată urmează că (C1a , C2a ) şi (C2a , C2c ) sunt perechi de cercuri cu centrele unul
pe altul. Deoarece O2a este şi pe C1a şi pe C2c , rezultă că O2a coincide cu B, ceea
ce antrenează că O1a coincide cu C. În consecinţă, triunghiul BGC este echilateral,
de unde deducem că triunghiul ABC √ este isoscel cu AB = AC. În fine, se obţine
imediat că BC = a, AB = AC = 7a.
Întrucât alte poziţii ale centrelor O1a , O2a , O2c conduc la cazurile examinate mai
sus, demonstraţia primei părţi a teoremei este ı̂ncheiată.
Invers, se constată imediat că un triunghi echilateral de latură a are rozeta asociată
a
regulată, de centru G (centrul triunghiului echilateral) şi rază (Fig. 5a).
3 √
Fie acum ABC un triunghi isoscel cu BC = a şi AB = AC = 7a. Fie G centrul
său de greutate. Prelungim latura BC ı̂n exterior cu segmentele BB ∗ = CC ∗ = a
(Fig. 5b). Obţinem triunghiul echilateral AB ∗ C ∗ de latură 3a şi centru G. Se cons-

36
tată cu uşurinţă că rozetele asociate triunghiurilor ABC şi AB ∗ C ∗ coincid, adică
sunt formate din aceleaşi cercuri. Cum rozeta triunghiului echilateral AB ∗ C ∗ este
regulată, rezultă că şi triunghiul ABC are rozetă regulată. Demonstraţia teoremei
este completă.
În concluzie, singurele triunghiuri care au rozeta √ √asociată regulată sunt triun-
ghiurile
… echilaterale
… şi cele isoscele de forma (a, 7a, 7a). Triunghiurile de forma
5 5
(a, a, a) au rozeta asociată regulată, dar formată din patru cercuri-petale.
2 2

Bibliografie
1. D. Brânzei, S. Aniţa, M. Chirciu – Geometrie. Clasa a IX-a, ed. a III-a, Ed.
Paralela 45, Piteşti, 1998.
2. T. Lalescu – Geometria triunghiului, Ed. Tineretului, Bucureşti, 1958.

(răspuns la ,,recreaţia” de la p. 20 )
Întrucât anul 2017 se scrie ı̂n mod unic ca sumă de două pătrate perfecte:

2017 = 442 + 92 = 1936 + 81,

rezultă că venerabila persoană s-a născut ı̂n anul 1936 şi are 81 de ani.

Remarcă. Cunoaştem două astfel de persoane: D.M. Bătineţu-Giurgiu, co-


laborator constant al revistei Recreaţii Matematice şi popularul actor Florin Persic,
care, la 27 ianuarie 2017, au ı̂mplinit 81 de ani!

37
Un problème de la combinatoire des ensembles
Adrien REISNER1

Abstract. Let Ω be a set and H a set of Ω- subsets. We define the set πH (S) = {S ∩H, H ∈ H}.
We here study the saturated sets of Ω, i.e. such that Card(πH (S)) = 2|S| , and we find an upper
bound of |πH (S)|.
Keywords: cardinal number, set, saturated set.
MSC 2010: 97K20, 03E05.

Soit Ω un ensemble. H étant un ensemble de parties de Ω, on définit pour toute


partie S de Ω l′ ensemble

πH (S) = {S ∩ H, H ∈ H}.

On se propose dans cet article de trouver un majorant de |πH (S)|, où |E| désigne le
cardinal du sous-ensemble E. P(E) étant l′ ensemble des parties de E, introduisons la
définition suivante:

Définition. On dit qu′ une partie S de Ω est saturée, lorsque on a:

πH (S) = P(S).

On suppose que d = max |S| existe et est fini.


S saturée

Quelques propriétés d′ une partie saturée. La proposition suivante est


presque évidente:

Proposition 1. a) Pour toute partie saturée S1 de Ω on a:

∀s ∈ P(S1 ), ∃H ∈ H, s ⊂ H.

En particulier, ∃H ∈ H tel que S1 ⊂ H.


b) Tout sous-ensemble d′ une partie saturée est encore saturée.
Démonstration. a) En effet, étant donné que πH (S1 ) = P(S1 ), on en déduit
immédiatement l′ assertion a).
b) Σ étant un sous-ensemble de S1 , on a : P(Σ) ⊂ P(S1 ), d′ où on en déduit

l assertion b).

Remarque. En particulier, pour tout entier naturel n ≤ d il existe une partie


saturée de cardinal n : il suffit de considérer un sous-ensemble de cardinal n d′ une
partie saturée de cardinal maximale d.
1 TELECOM ParisTech; adrien.reisner@yahoo.fr

38
On désignera dans la suite par S0 un sous-ensemble
 de Ω saturé de cardinal d.
Soit S un sous-ensemble quelconque de Ω. |S| i étant le nombre de parties à i
éléments de S, il est clair que la maximum du cardinal de πH (S) est un entier inférieur
ou égal à |P(S)| = 2|S| :
|S| Ç å
X |S|
|πH (S)| ≤ = |P(S)| = 2|S| .
i=0
i

Distinguons deux cas suivant la position de l′ entier |S| par rapport à d.

Proposition 2. Si |S| ≤ d, on a:

Xd Ç å
|S|
|πH (S)| ≤ = 2|S| ,
i=0
i

avec égalité si et seulement si S est saturée. En particulier,

|πH (S0 )| = 2d .

Démonstration. Il est évident que le maximum du cardinal de πH (S) est le


cardinal de P(S) - S est alors saturé -, et que ce cas a lieu, par exemple, lorsque
S ⊂ S0 - voir remarque b) précédente. En effet, S étant saturé, dans ce cas on a:
|S| Ç å d Ç å
X |S| X |S|
|πH (S)| = = =| P(S) |= 2|S| ,
i=0
i i=0
i

|S|
avec la convention i = 0 si i > |S|. La proposition est ainsi démontrée.

Proposition 3. Pour tout sous-ensemble S de Ω tel que |S| ≥ d, on a:

Xd Ç å
|S|
|πH (S)| ≤ , (•)
i=0
i

avec égalité si et seulement si S est une partie saturée de cardinal maximale d.


Démonstration. Afin d′ établir l′ inégalité (•) raisonnons par l′ absurde. On se
propose donc de démontrer que l′ égalité :

Xd Ç å
|S|
|πH (S)| = 1 + est impossible
i=0
i
P 
et, par suite, l′ inégalité stricte |πH (S)| > di=0 |S| i est impossible.
En effet, si |S| = d + 1 alors, on aurait :

Xd Ç å Ç å
Xd Ç å d+1 Ç
X |S|
å
|S| d+1 |S|
|πH (S)| = 1 + = + = = 2d+1
i=0
i d + 1 i=0
i i=0
i

39
et S serait une partie saturée de cardinal d + 1 ce qui contredit la définition de d
comme maximum du cardinal d′ une partie saturée. P 
Supposons maintenant que S vérifie |S| > d + 1 et |πH (S)| = 1 + di=0 |S| i . La
P 
somme di=0 |S| i est la somme des cardinaux de tous les parties à 0, 1, ..., d éléments
de S. Or, pour tout sous-ensemble S ′ ⊂ S de cardinal d + 1, on a: P(S ′ ) ⊂ P(S) et,
par suite, en ne considérant que les éléments de S ′ , il existe donc un ensemble S ′ de
P ′ 
cardinal d + 1 tel que |πH (S ′ )| = 1 + di=0 |Si | , ce qui est impossible compte tenu
du cas précédemment étudié.

Les deux propositions précédentes permettent alors de conclure avec le corollaire


suivant:
Corollaire. Pour tout sous-ensemble S de Ω on a l′ inégalité suivante:
Xd Ç å
|S|
|πH (S)| ≤
i=0
i

avec égalité si et seulement si S est une partie saturée de cardinal maximale d.

Application. Soit Ω un ensemble à n éléments et h un entier vérifiant 1 ≤ h ≤


n − 1. Considérons un élément ω0 ∈ Ω et l′ ensemble
H = {H ⊂ Ω; 1 ≤ |H| ≤ h , ω0 ∈ H}.
On prouve que les parties saturées sont les sous-ensembles H \ {ω0 } avec H ∈ H.
En effet, si H ∈ H , on voit facilement que πH (H \ {ω0 }) = P(H \ {ω0 }). D′ autre
part, soit un sous-ensemble S tel que |S| = h et ω0 6∈ S. On constate que l′ ensemble
πH (S) ne contient pas S lui-même et donc S n′ est pas saturé.
On en déduit que les H \ {ω0 } où H ∈ H et |H| = h sont des parties saturées de
cardinal maximal, i.e. d = max |S| = h − 1. Le Corollaire montre alors que pour
S saturée
tout sous-ensemble S de Ω , on a:
h−1 Ç å h Ç å
X |S| X |S|
|πH (S)| ≤ = .
i=0
i i=1
i−1

Dans le domaine des graphes on peut trouver d′ autres applications du corollaire.

Bibliographie
1. J.M. Arnaudiès, H. Fraysse – Cours d′ algèbre, t. 1, Dunod , Paris, 1987.
2. N.L. Biggs – Discrete Mathematics, Oxford Science Publications, 1986.
3. J.H. van Lint, R.M. Wilson – A course in combinatorics, Cambridge Univerity
Press, 2001.
4. L. Schwartz – Analyse I: Théorie des ensembles et topologie, Hermann, Paris, 1991.

40
O privire asupra unei probleme arabe
Temistocle BÎRSAN1

Abstract. The aim of this Note consists in investigating an old Arabian problem, in order to
offer a pattern for the way to find a subject of study starting from a well-known problem.
Keywords: partition, natural number, common multiple.
MSC 2010: 97F80.

Recent, am primit online o veche şi cunoscută problemă arabă. Faptul mi-a
ı̂ndreptat gândul spre alte probleme care, cândva, m-au fascinat: lupul, capra şi varza,
felurite probleme cu râuri şi bărci sau povestirea cu tâlc Cinci pâni a lui Ion Creangă.
Acum, am privit cu ,,ochi critic” problema primită, adică mi-am pus ı̂ntrebări pe
marginea ei. În final, s-a adunat un material care constituie un exemplu de modul ı̂n
care o problemă ajunsă sub privirile noastre poate deveni un subiect de studiu.
Iată problema şi soluţia primite (şi cunoscute):
Problemă arabă. Un bătrân arab a lăsat moştenire celor trei fii ai săi, prin
testament, 17 cămile. Testamentul prevedea ca fiul cel mare să primească jumătate
din numărul lor, fiul cel mijlociu o treime şi fiul cel mic a noua parte. Cei trei fii
nu ajung la o ı̂nţelegere şi decid să meargă la un ı̂nţelept. Acesta, după ce ascultă
prevederea testamentară, procedează astfel: adaugă o cămilă dintr-ale sale la cele 17
cămile şi apoi dă celui mare 9 cămile (jumătate din numărul de 18 cămile), celui
mijlociu 6 cămile (o treime din acest număr) şi celui mic 2 cămile (a noua parte),
iar cămila rămasă, cămila sa, şi-a luat-o ı̂napoi.
Ce dificultăţi i-au făcut pe fiii moştenitori să apeleze la ı̂nţelept? Mai ı̂ntâi, din
prevederea testamentară rezultă că nu trebuie ı̂mpărţite toate cămilele. Într-adevăr,
1 1 1 9+6+2 17 17 1
din faptul că + + = = < 1 şi 1 − = , urmează că a
2 3 9 18 18 18 18
optsprezecea parte din cele 17 cămile (i.e., mai puţin de o cămilă) ar trebui să nu fie
ı̂mpărţită. Apoi, dacă s-ar respecta testamentul, fiilor nu le-ar reveni un număr ı̂ntreg
de cămile (fiului mare i-ar reveni 8 cămile ı̂ntregi şi ı̂ncă o jumătate de cămilă etc.).
Evident, aceste dificultăţi apar din cauză că 17 nu este divizibil cu 2, 3 şi 9; ı̂n caz
contrar, dacă numărul de cămile ar fi fost divizibil cu 2, 3 şi 9, fiecare fiu ar fi putut
lua partea sa şi ar fi rămas un număr de cămile neprevăzute lor ı̂n testament.
Cum rezolvă ı̂nţeleptul problema, ce idee pune la baza rezolvării? Acesta consideră
numărul 18, care este cel mai mic număr ce depăşeşte pe 17 şi care este divizibil cu
2, 3 şi 9. În consecinţă, adaugă o cămilă, din propriile cămile, pentru a obţine un
număr total de 18 cămile. Raportat la numărul nou de cămile, ı̂mpărţirea se face
uşor: fiul cel mare primeşte 9 cămile, cel mijlociu 6 cămile şi cel mic 2 cămile. Cum
la un loc cei trei fii primesc 9 + 6 + 2 = 17 cămile, mai rămâne o cămilă nedistribuită
1 Prof.dr., Univ. Tehnică ,,Gh. Asachi”, Iaşi; t birsan@yahoo.ro

41
- tocmai cămila adăugată -, pe care ı̂nţeleptul şi-o recuperează. O adevărată lovitură
de magician! Matematică sau poveste din O mie şi una de nopţi !??
Să observăm că ı̂nţeleptul, văzând că testamentul nu poate fi respectat literă cu
literă, a dat o soluţie aproximativă problemei, soluţie care are ı̂n vedere respectarea
următoarelor cerinţe fireşti:
a) fiii să primească un număr ı̂ntreg de cămile;
b) parţile primite de fii să fie proporţionale cu cele cuvenite (prin testament);
c) suma părţilor primite de fii să nu depăşească numărul total al cămilelor.
Să ı̂ncercăm să desluşim secretul acestui final ,,fericit” al rezolvării problemei
arabe. Să vedem ı̂n ce măsură procedeul ı̂nţeleptului poate fi aplicat şi ı̂n cazul altor
probleme de acest tip. Faptul că ı̂nţeleptul completează cu o singură cămilă numărul
lor este accidental, după cum arată următorul
2 1 1
Exemplu. Aceeaşi problemă cu datele: 28 de cămile şi fracţiile , , .
3 6 10
În acest caz, ı̂nţeleptul trebuie să adauge 2 cămile pentru a ajunge la un număr
de 30 cămile (număr divizibil cu numitorii fracţiilor), iar părţile fiilor (calculate din
30 de cămile) vor fi: 20 cămile, 5 cămile şi 3 cămile. Se vede că ı̂nţeleptul va recupera
cele 2 cămile ale sale.
Problemă. Să se ı̂mpartă N cămile la k fii F1 , F2 , ..., Fk astfel ı̂ncât partea fiului
pi  P pi
i=k
Fi , i = 1, k, să fie din cele N cămile, unde pi , qi ∈ N∗ i = 1, k şi ≤ 1.
qi i=1 qi
pi
Fără a restrânge generalitatea, presupunem că fracţiile , i = 1, k, sunt ire-
qi
ductibile. Evident, părţile fiilor trebuie să fie numere ı̂ntregi de cămile.
P pi
i=k
I. Cazul ı̂n care = 1 este trivial: (i) dacă N este divizibil cu numitorii
i=1 qi
pi
q1 , q2 , ..., qk , atunci partea fiului Fi este N ∈ N şi ı̂mpărţirea este
qi
p1 p2 pk
(1) N= N + N + ... + N ;
q1 q2 qk
(ii) dacă măcar unul dintre numitori nu divide pe N, ı̂mpărţirea nu este posibilă.
P pi
i=k
II. Dacă < 1, din numărul N de cămile rămâne o parte nedistribuită fiilor.
i=1 qi
Se impune să considerăm două subcazuri:
1. N se divide
Å cu q1 ,ãq2 , ..., qk ; ca urmare, atât părţile ce revin fiilor cât şi partea
P pi
i=k
rămasă α = 1 − N sunt numere naturale şi ı̂mpărţirea se face exact (i.e.,
i=1 qi
sunt respectate cerinţele din enunţul problemei), fiind dată de
p1 p2 pk
(2) N= N + N + ... + N + α.
q1 q2 qk

2. N nu se divide cu toţi numitorii q1 , q2 , ..., qk . Împărţirea nu se poate face


exact, dar este posibil, după modelul problemei arabe, să dăm problemei o soluţie
aproximativă ce verifică condiţiile a) - c) de mai sus.

42
Fie M primul multiplu comun al numerelor q1 , q2 , ..., qk mai mare ca N, deci
p1 p2 pk
M > N. Considerăm numerele: M, M, ..., M. Avem:
q1 q2 qk
pi ∗
a) M ∈ N pentru orice i = 1, k;
qi
p1 p2 pk
b) evident, aceste numere sunt proporţionale cu numerele N, N, ..., N,
q1 q2 qk
M
factorul de proporţionalitate fiind > 1;
N
P pi
i=k
c) condiţia M ≤ N este ı̂ndeplinită
i=1 qi
P pi
i=k
(i) dacă M = N, echivalent cu faptul că nu mai rămân cămile după ce fii ı̂şi
i=1 qi
p1 p2 pk
iau părţile M, M, ..., M (aşa cum este cazul ı̂n problema arabă) şi, deci, avem
q1 q2 qk
formula de ı̂mpărţire a cămilelor:
i=k
X pi
(3) N= M,
i=1
qi

P
i=k pi
(ii) sau dacă M < N, situaţie ı̂n care mai rămâne un număr de cămile după
i=1 qi
ce fii ı̂şi iau părţile cuvenite şi are loc formula
i=k
X i=k
X
pi pi
(4) N= M + β, unde β = N − M.
i=1
qi i=1
qi

pi P
i=k
Din cele de mai sus rezultă că, dacă M > N, problema nu are soluţie apro-
i=1 qi
ximativă (ı̂n sensul precizat), deoarece ı̂n acest caz condiţia c) nu este ı̂ndeplinită.
Exemplu. Fie N ∈ N∗ supus condiţiei 30 ≤ N ≤ 60. Patru persoane primesc
1 1 1 2
, , , din numărul de N cămile. Discuţie după valorile lui N asupra posibilităţii
3 5 6 15
efectuării acestei distribuţii.
Dacă N = 30 sau N = 60, atunci el este divizibil cu 3, 5, 6, 15, deci calculul părţilor
cuvenite persoanelor conduce la numere ı̂ntregi. Avem: pentru N = 30 părţile sunt
10, 6, 5, 4 şi mai rămân 5 cămile (30 = 10 + 6 + 5 + 4 + 5), iar pentru N = 60 părţile
sunt 20, 12, 10, 8 şi mai rămân 10 cămile (60 = 20 + 12 + 10 + 8 + 10).
1 1 1
Dacă N = 50, atunci M = 60, părţile cuvenite sunt: ·60 = 20, ·60 = 12, ·60 =
3 5 6
2
10, · 60 = 8 şi are loc o ı̂mpărţire a cămilelor de forma (3): 50 = 20 + 12 + 10 + 8.
15
În cazul 50 < N < 60, M şi părţile cuvenite sunt aceleaşi, dar mai rămâne un
rest; avem: N = 20 + 12 + 10 + 8 + β, cu β = N − 50.
În sfârşit, dacă 30 < N < 50 problema nu admite nici o soluţie ce să verifice
condiţiile a) - c); ı̂n acest caz suma părţilor cuvenite, egală tot cu 50, depăşeşte N .

43
Academia Mihăileană (1835-1864)

În prima jumătate a secolului al XIX-lea, ı̂ntr-un context legislativ favorabil, a fost
organizată cea mai importantă instituţie de ı̂nvăţământ cu predare ı̂n limba română
din Moldova – Academia Mihăileană din Iaşi – şcoală din care vor descinde, peste
trei decenii, Universitatea ,,Al.I.Cuza” şi Liceul Naţional. Academia Mihăileană nu
trebuie ı̂nsă considerată o şcoală cu totul nouă, ea continuând tradiţia unei vechi
instituţii de cultură din veacul al XVII-lea, Şcoala Vasiliană (9 mai 1640), care a fost
periodic reformată şi reorganizată. În perioada fanariotă, aceasta s-a transformat
ı̂ntr-o academie cu predare ı̂n limba greacă, care şi-a ţinut cursurile iniţial tot la Trei
Ierarhi, apoi ı̂n curtea Mitropoliei şi a funcţionat până la tulburările din 1821. În
1828 domnitorul Ioniţă Sandu Sturza nu a făcut altceva decât să restaureze vechiul
aşezământ, sub numele de Gimnaziul Vasilian, care a cuprins treptat şi cursuri supe-
rioare şi care a fost reorganizată sub numele de Academia Mihăileană1 .
Primele cursuri ale Academiei ieşene au fost deschise cu câţiva ani ı̂nainte de
inaugurarea oficială a instituţiei, iar pregătirile au ı̂nceput chiar la scurt timp după
organizarea Gimnaziului Vasilian (23 ianuarie 1828), fiind accelerate după punerea
ı̂n vigoare a Regulamentului Organic. La 3 iunie 1830, cu mare solemnitate, era
inaugurat primul curs de legi ı̂n limba română, ţinut de jurisconsultul statului, Chris-
tian Flechtenmacher. La festivitate au participat marii boieri şi reprezentanţii cleru-
lui, ı̂n conferinţa de deschidere ţinându-se o prelegere de istoria dreptului românesc.
1 Această idee a continuităţii ı̂ntre instituţiile de ı̂nvăţământ care au funcţionat ı̂n Moldova pe

parcursul a două secole o ı̂ntâlnim şi ı̂n Regulamentul Organic, care prevedea necesitatea alcătuirii
unui regulament al şcolilor pentru organizarea ,,vechei academii vasiliene, ı̂ntemeiată ı̂n Mănăstirea
Trei Ierarhi de fericitul ı̂ntru nemuritoare aducere aminte, Domnul Vasile Voievod” (Dumitru Vitcu,
Gabriel Bădărău, Regulamentul Organic al Moldovei, Iaşi, Editura Junimea, 2004, p. 165).

44
Ulterior, numărul cursurilor a crescut repede, anunţând astfel disciplinele din pro-
grama viitoarei academii, respectiv cele de limbi străine – limba greacă (predată de
Atanasie Terapianul), limba rusă (Vasile Peltichi), limba franceză (Charles Tissot),
limba germană (Samuil Botezatu) – de desen (Joseph Adler), de desenarea figurilor şi a
zugrăviturii istorice ı̂n ulei (Johan Müller)2 . În vederea asigurării viitoarei academii cu
personalul didactic necesar, ı̂n februarie 1832, membrii Epitropiei lansau o publicaţie
ı̂n paginile Albinei româneşti, prin care anunţau existenţa a cinci posturi vacante3 ,
ı̂nsă, ı̂n perioada imediat următoare, au fost numiţi doar doi profesori, proiectul or-
ganizării academiei fiind amânat.
Pentru a nu mai ı̂ntârzia deschiderea academiei până la finalizarea lucrărilor de
amenajare a localului propriu, Epitropia a decis ı̂nceperea la 20 noiembrie 1834, ı̂n
cadrul Gimnaziului Vasilian, a cursurilor anului I de filosofie susţinute de Eftimie
Murgu, Vasile Fabian şi Gheorghe Săulescu. Această dată marchează de fapt momen-
tul ı̂nfiinţării Academiei ieşene, ı̂ntrucât anul I filosofie nu mai aparţinea cursurilor
secundare ca activitate didactică şi conţinut. În anul următor, la 16 iunie 1835, a
avut loc doar inaugurarea noului local, făcută cu mare solemnitate şi deschisă prin
Te-Deum-ul oficiat la biserica Talpalari4. Ulterior, ı̂ntreaga asistenţă s-a deplasat
la Academie, unde domnitorul şi referendarul şcolilor au ţinut discursuri ı̂n care au
evidenţiat progresele făcute ı̂n domeniul ı̂nvăţământului din Moldova ı̂n ultimii ani,
manifestându-şi ı̂ncrederea că, ı̂n viitor, va continua dezvoltarea acestuia, deoarece
,,singurul mijloc al fericirii prea iubitei noastre patrii este o dreptă luminare a fiilor
ei”5 .
Primele demersuri pentru asigurarea spaţiului necesar desfăşurării cursurilor s-au
făcut ı̂n toamna anului 1834, când Epitropia ı̂nvăţăturilor publice a propus domni-
torului achiziţionarea caselor lui Petrache Cazimir, situate lângă biserica Talpalari6 .
Cumpărarea s-a realizat imediat, ı̂nsă, pentru susţinerea cursurilor ı̂n această clădire,
Epitropia a făcut noi cereri către conducerea ţării pentru a aproba fonduri ı̂n vederea
asigurării mobilierului necesar şi realizării reparaţiilor necesare7 . În primăvara anu-
lui următor, conducerea şcolilor a propus achiziţionarea caselor serdarului Alexandru
Anastasie Voinescu, situate peste drum, unde actualmente se află Colegiul Naţional,
iar pe atunci era hotelul Petersburg (considerat de contemporani loc ı̂n care se adunau
,,rele tovărăşii”)8 . Din cauza lipsei fondurilor, cumpărarea s-a facut abia după doi ani,
apoi clădirea a fost amenajată şi, ı̂n aprilie 1838, internatul, economatul, bucătăria
şi administraţia funcţionau ı̂n casele Voinescu, iar cursurile, biblioteca, cabinetul de
2 Gabriel Bădărău, Academia Mihăileană, ı̂n Istoria Universităţii din Iaşi, Editura Universităţii

Alexandru Ioan Cuza, Iaşi, 2010, p. 103.


3 Albina Românească, nr. 15, 21 februarie 1832
4 Gabriel Bădărău, op. cit., p. 105.
5 V. A. Urechia, Istoria şcoalelor de la 1800-1864, vol. I, Bucureşti, Imprimeria Statului, 1892, p.

245.
6 Ibidem, vol. I, p. 267.
7 În anul 1835, cursurile gimnaziale de la Trei Ierarhi, cursul de filosofie şi cursurile extraordinare

existente au fost mutate ı̂n clădirea academiei, ı̂n incinta mănăstirii Trei Ierarhi rămânând doar cursu-
rile ı̂ncepătoare şi, până la cumpărarea şi amenajarea caselor Voinescu, internatul pentru bursierii
săraci.
8 C. I. Andreescu, Evoluţia ı̂nvăţământului ı̂n Moldova şi istoricul Liceului Naţional din Iaşi, Iaşi,

Tipografia Brawo, 1935, p. 60.

45
fizică şi matematică, cabinetul de istorie naturală ı̂n casele lui Petrache Cazimir.
Pentru a ı̂nlesni trecerea elevilor dintr-o clădire ı̂n alta, ı̂n 1835 a fost avansată ideea
construirii unui arc de legătură, ı̂nsă lucrarea a fost terminată ı̂n 1845, deoarece suma
oferită a fost mică, iar lucrarea a fost finalizată după multe ı̂ntârzieri, ı̂ntreruperi,
cereri de bani şi chiar schimbări de antreprenori. În anul şcolar 1845-1846 s-a mod-
ificat repartizarea sălilor Academiei, respectiv ı̂n casele Cazimir au fost cazaţi 120
de elevii interni, iar ı̂n casele Voinescu s-au mutat cursurile gimnaziale şi academice.
Astfel, timp de câteva decenii, Academia şi internatul au funcţionat ı̂n cele două
case, unite printr-un arc, până când, ı̂n urma intervenţiilor repetate ale directoru-
lui de atunci (V. Paladi), s-a hotărât demolarea vechii clădiri a şcolii şi construirea
actualului local al Colegiului Naţional (1890)9.
Pentru a asigura bunul mers al instituţiei, ı̂n 1835 a fost ı̂ntocmit de către Epitropie
un regulament al şcolilor publice care cuprindea prevederi referitoare la modul de
administrare a acestora şi de funcţionare a comitetului academic, numirea cadrelor
didactice pe post, drepturile şi responsabilităţile profesorilor, admiterea elevilor, dis-
ciplină, examene şi diplome. Şcolile erau clasificate ı̂n ordinare şi extraordinare, ı̂n
prima categorie intrând şcolile ı̂ncepătoare, gimnaziul cu două clase pregătitoare şi
două de umanioare şi academia cu trei facultăţi (filosofie, drept şi teologie). În cate-
goria şcolilor extraordinare erau cuprinse şcoala reală, institutul tehnic, cursurile de
limbi străine, desenul de figuri istorice şi zugrăveli, cursul de arte frumoase, desenul de
arhitectură şi geometrie, care puteau fi urmate de absolvenţii cursurilor ı̂ncepătoare10.
În primul an şcolar, activitatea didactică s-a desfăşurat sub semnul provizoratu-
lui, luându-se măsuri privind organizarea instituţiei pe tot parcursul acestuia. Astfel,
abia ı̂n luna iunie a anului 1835 s-au impus reguli privind disciplina, s-a ı̂ntocmit un
program provizoriu şi s-au fixat examenele, care au fost susţinute cu ı̂ntârziere (după
1 iulie 1835 pentru gimnaziu şi la 1 octombrie pentru anul I de filosofie). De asemenea,
deoarece elevii nu aveau nivelul de pregătire necesar pentru urmarea cursurilor aca-
demice, s-a luat hotărârea ca aceştia să studieze şi pe timpul vacanţelor sau să urmeze
cele două clase pregătitoare. Cu toate aceste dificultăţi, conform raportului ı̂ntocmit
la sfârşitului anului, au fost ı̂nregistraţi 322 elevi la clasa elementară şi 64 la clasa
normală, 87 la gimnaziu, dintre care unii au urmat şi cursuri extraordinare de legi,
franceză, germană, rusă şi elenă. Dintre aceştia, 26 au fost aleşi iniţial pentru cursurile
superioare de filosofie, iar, ı̂n cele din urmă, şase au fost trimişi ı̂n străinătate11 .
În anul şcolar 1835-1836 a continuat procesul de organizare prin completarea
comitetutului academic cu profesorii necesari şi stabilirea măsurilor ce urmau să fie
luate ı̂n cazul ı̂n care aceştia sau elevii lipseau de la cursuri fără motive ı̂ntemeiate. De
asemenea, s-a mai luat o hotărâre cu consecinţe importante şi anume condiţionarea
accesului ı̂n slujbele statului de finalizarea studiilor gimnaziale. Această măsură ı̂n
spirit liberal, care deschidea accesul la funcţiile publice pentru toate categoriile sociale,
a determinat nemulţumirea boierilor conservatori, susţinuţi de reprezentantul Rusiei
ı̂n Moldova. Astfel, ı̂n anii următori s-a dus o luptă acerbă ı̂ntre două curente, unul
9 Gabriel Bădărău, op. cit., p. 105-107. Casele Cazimir, cu cunoscuta cupolă turco-bizantină, au

rezistat până ı̂n 1963, când, din motive de utilitate publică, au fost dărâmate.
10 V. A. Urechia, op. cit., vol. IV, p. 389-421.
11 C. I. Andreescu, op. cit., p. 68-70.

46
liberal şi naţionalist, celălalt conservator, care a pledat pentru reformarea profundă
a instituţiei prin ı̂nlocuirea limbii române cu limba franceză. Într-o primă etapă, a
biruit curentul naţionalist, care avea susţinători de seamă precum Gheorghe Asachi
şi mitropolitul Veniamin Costachi12 .
În perioada 1837-1840, calitatea actului didactic a cunoscut ı̂mbunătăţiri semni-
ficative prin ı̂nmulţirea cursurilor academice şi a obiectelor extraordinare, tipărirea
cărţilor şi angajarea unui număr mare de profesori, dintre care unii aveau titlul de
doctor obţinut la universităţi de prestigiu din Europa. Un suflu nou, impregnat de
educaţia occidentală, a fost imprimat instituţiei ieşene de Charles Maisonabe, rector
al academiei şi profesor de legi. El a stabilit regulile privind primirea şi găzduirea ı̂n
internat, comportamentul profesorilor şi elevilor, ı̂nsă activitatea sa - criticată asiduu
de consulul rus - a fost de scurtă durată, la ı̂nceputul anului 1838 cursul fiind sus-
pendat. Cu toate progresele realizate, domnitorul Mihail Sturdza nu a renunţat la
ideea reorganizării şcolii pe baze noi, ı̂nsă Epitropia şi comitetul academic au prezen-
tat (la 1 octombrie 1838) domnitorului un proiect de completare a regulamentului din
1835 cu hotărâri care nu duceau la modificarea statutului limbii române. La ı̂nceputul
anului următor ideea reformării a fost reluată, membrii Adunării Obşteşti fiind trimişi
să asiste la examene pentru a evalua activitatea didactică13 .
Măsurile luate ı̂n anul şcolar 1840-1841, precum şi hrisovul emis la 8 noiembrie
1840, ı̂ncheiau o primă etapă a organizării instituţiei, etapă ı̂n care a fost asigu-
rat spaţiul necesar atât pentru susţinerea cursurilor, cât şi pentru internat, au fost
achiziţionate materiale didactice, iar numărul cursurilor a sporit constant. În acest
an s-a stabilit obligativitatea studierii limbilor clasice (latina şi greaca) pentru cei
care urmau cursurile academice, s-a introdus limba italiană, care era deja studiată
la Bucureşti, un curs de gimnastică, s-au făcut demersuri pentru deschiderea anului
II de legi şi s-a organizat biblioteca, ale cărei baze fuseseră puse prin donaţia făcută
de domnitorul Mihail Sturdza la 8 noiembrie 1839. Hrisovul emis ı̂n toamna anu-
lui 1840 scotea ı̂n evidenţă necesitatea implicării statului ı̂n sistemul de ı̂nvăţământ
pentru pregătirea tinerilor, efortul depus de actualul domnitor, pe vremea ı̂n care
era epitrop al şcolilor, ı̂n vederea introducerii limbii române şi progresele realizate ı̂n
acest domeniu. Conform acestui act, sistemul de ı̂nvăţământ era organizat pe trei
nivele: ı̂nvăţământul primar reprezentat de şcolile ı̂ncepătoare din ţinuturi şi şcoala
de la Trei Ierarhi, ı̂nvăţământul secundar prin Colegiul Vasilian şi ı̂nvăţământul uni-
versitar ı̂n formă incipientă prin Academia Mihaileană (cu trei facultăţi, filosofie, legi
şi teologie)14 .
Din nefericire, ı̂n deceniul al cincilea al secolului al XIX-lea, Academia Mihăileană
a cunoscut un declin constant determinat de lupta acerbă dintre cele două curente ale
elitei politice şi culturale, respectiv cel conservator şi cel progresist. Ea a fost generată
de ı̂ntărirea şi completarea de către domnitor a unei hotărâri mai vechi care stabilea
că, pentru primirea ı̂n funcţiile publice, tinerii trebuie să prezinte certificatul de ab-
12 V. A. Urechia, op. cit., vol. I, p. 280.
13 A. D. Xenopol, Constantin Erbiceanu, Serbarea şcolară la Iaşi cu ocazia ı̂mplinirii a cincizeci
de ani de la ı̂nfiinţarea ı̂nvăţământului superior ı̂n Moldova. Acte şi documente, Iaşi, Tipografia
Naţională, 1885, p. 63-64; C. I. Andreescu, op. cit., p. 87-96.
14 C. I. Andreescu, op. cit., p. 96-100.

47
solvire a studiilor. Această dispoziţie ı̂i favoriza pe ştiitorii de carte şi ı̂i nemulţumea
pe boieri, care au ı̂ncercat prin toate mijloacele de care dispuneau să ı̂ngrădească
accesul claselor sărace la studii, mai ales la cele academice. O modalitate de a ı̂ngrădi
accesul la studiile academice a claselor de jos a fost hotărârea de a ı̂nlocui limba de
predare, româna cu franceza. Însă, timp de aproape un deceniu, această luptă din-
tre cele două grupări a ı̂mbrăcat diferite forme. Astfel, la 28 ianuarie 1842 a avut
loc un eveniment care a influenţat mersul şcolii şi al culturii româneşti, retragerea
mitropolitului Veniamin Costachi din funcţie ca urmare a intrigilor curentului rusofil.
Un alt eveniment a fost ı̂ntreruperea cursului de istoria românilor, susţinut de Mi-
hail Kogălniceanu, după numai câteva lecţii, deoarece nu era pe placul guvernanţilor
filoruşi din acea perioadă, ı̂ntrucât ı̂nflăcăra auditoriul prin prezentarea eroilor şi vic-
toriilor noastre. Această ı̂ncordare a fost accentuată şi de faptul că Ion Ghica, fire
autoritară, ı̂n calitate de inspector, a luat mai multe măsuri menite să ducă la o mai
bună organizare, ı̂nsă ele nu au fost primite cu entuziam de corpul profesoral şi de
elevi. Printre acestea se numără hotărârea ca profesorii să respecte programul de
lucru şi să intre la ore după sunarea clopoţelului, să nu aplice elevilor pedepse corpo-
rale, elevii din internat să repete sub supravegherea pedagogului, să nu lipsească de
la biserică ı̂n zilele de sărbătoare şi să nu părăsească institutul fără voie.
În ciuda faptului că s-au făcut, totuşi, o serie de progrese prin completarea cur-
sului de legi cu anul III şi deschiderea anului V la gimnaziu, introducerea practicii la
drept, dotarea sălii de gimnastică şi a bibliotecii, declinul a continuat. Astfel, s-au
ı̂nregistrat demisii, ı̂nlocuiri şi concedieri ı̂n lanţ, au fost desfiinţate unele cursuri,
numărul elevilor a scăzut - ı̂n special al celor săraci - iar lefurile dascălilor erau plătite
cu mari ı̂ntârzieri. În ceea ce priveşte conducerea sistemului de ı̂nvăţământ - Comite-
tul academic şi Epitropia şcolilor - aceasta a ajuns să fie subordonată domnitorului
(membrii de seamă şi-au dat demisia, fiind ı̂nlocuiţi cu persoane apropiate domni-
torului sau chiar rude), care de la un energic susţinător al dezvoltării ı̂nvăţământului
ı̂n limba română ı̂n calitate de epitrop, a ajuns să sprijine gruparea boierilor rusofili,
care vedea ı̂n academie un pericol, deoarece putea duce treptat la ı̂nlocuirea unei elite
politice cu alta provenită din categorii mai sărace15.
Tensiunile accentuându-se, se ajunge până acolo ı̂ncât boierii conservatori, spriji-
niţi de consulul rus, au cerut domnitorului ı̂n 1843 desfiinţarea cursului superior al
academiei. Această doleanţă a fost ı̂naintată Adunării Obşteşti, condusă de un boier
cu idei liberale/progresiste, care a constatat că ,,şcoala românească şi academică
nu constituie un pericol”, spre dezamăgirea opoziţiei conservatoare. Cu toate aces-
tea, domnitorul a susţinut ideea unei reforme care, ı̂n accepţiunea sa, ar fi dus la
concilierea celor două grupuri antagoniste, ı̂nsă, prin poziţia adoptată, a determinat
dezorganizarea Academiei Mihăilene ı̂n ultimii ani de domnie. Astfel, la mijlocul dece-
niului cinci, numărul elevilor scăzuse foarte mult, iar internatele nu mai funcţionau,
fiind ı̂nlocuite cu pensionatul particular al profesorului francez Malgouvern. Ulterior,
acelaşi profesor, cu aprobarea conducerii ţării, a preluat gratuit localul academiei,
unde a deschis un colegiu cu predare ı̂n limba franceză16 .
Urcarea pe tronul Moldovei a domnitorului Grigore Alexandru Ghica ı̂n 1849
15 Ibidem, p. 100-111.
16 Ibidem, p. 109.

48
- revoluţionar paşoptist şcolit ı̂n Germania - marchează o nouă etapă ı̂n evoluţia
Academiei Mihăilene. Chiar din primul an de domnie a luat măsura desfiinţării in-
stitutului lui Malgouvern, Epitropia fiind ı̂nsărcinată să pună bazele unui aşezământ
scolastic ı̂n conformitate cu regulamentele şcolare. În ianuarie 1851 a fost finalizat
noul regulament şcolar prin care se statornicea că ,,ı̂nvăţătura publică este gratuită
şi slobodă pentru toţi locuitorii Moldovei”, iar ,,limba naţională va fi...singurul organ
al ı̂nvăţăturii publice”, instituţiile fiind ı̂mpărţite ı̂n trei categorii, respectiv şcoli pri-
mare (ı̂n această categorie intrând şcolile elementare din oraşe şi din capitală, şcolile
de fete, şcoala de meserii, şcolile de militari, şcolile armene şi cele evreeşti), şcoli
secundare (şcolile reale, ferma model şi gimnaziile care durau 7 ani) şi şcolile supe-
rioare, care cuprindeau ı̂n acest an patru facultăţi, respectiv filosofie, drept, teologie şi
medicină. De asemenea, a fost reı̂nfiinţat internatul pentru tinerii care urmau şcolile
secundare, cele mai multe locuri fiind acordate gratuit elevilor săraci, dar sârguincioşi.
Prin acest regulament, au fost luate hotărâri privind administrarea şcolilor, numirea
profesorilor, examenele semestriale, calicativele acordate (foarte bine, bine, de mijloc,
rău), vacanţe (15 iulie -1 septembrie), normele ce trebuiau respectate pentru alcătuirea
manualelor şi dotarea bibliotecii. Acest regulament a fost completat cu alte măsuri
care priveau condiţiile primirii ı̂n internat, obligaţiile pedagogilor, regulile privind
cheltuirea banilor pentru internat, instrucţiunile pentru profesorii de limbi străine
şi uniformele şcolare. Personalul didactic a fost sporit ı̂n mod constant, ı̂n această
perioadă susţinând cursuri la Academia Mihăileană personalităţi marcante ale cul-
turii româneşti precum Dimitrie Gusti, Teodor Codrescu, August Treboniu Laurian,
Grigore Cobălcescu17 .
Măsurile luate ı̂n cei şapte ani de domnie a lui Grigore Alexandru Ghica au creat o
bază solidă pentru acest sistem şi pentru Academia Mihăileană, ı̂n mod special, prin
hotărârile ı̂nţelepte şi energice cu care a ı̂nfăptuit reforma. Deşi a urmat o perioadă
de frământări politice, ı̂n care corpul profesoral şi elevii au fost cuprinşi de nelinişte,
şcoală moldavă a trecut cu bine peste această etapă, definitivându-şi organizarea
modernă prin reformele domnitorului Alexandru Ioan Cuza.
Astfel, Academia Mihaileană, deşi a cunoscut o evoluţie sinuoasă de la stadiul
ı̂n care nu reprezenta decât o completare a ı̂nvăţăturilor Gimnaziului Vasilian prin
câteva cursuri superioare la etapa ı̂n care cuprindea ı̂nvăţământul secundar şi su-
perior din Moldova (având sub protecţia sa şi şcoala primară de la Trei Ierarhi), a
reprezentat pentru trei decenii o şcoală emblematică pentru cultura ieşeană. Prin
această instituţie, care includea atât ı̂nvăţământul secundar, cât şi pe cel superior,
s-a făcut trecerea definitivă la un ı̂nvăţământ modern, organizat pe mai multe nivele
prin reforma din 1864. Însă până la vremea lui Cuza-Vodă, Mihail Sturdza şi Grigore
Alexandru Ghica au fost domnitorii care au jucat un rol important ı̂n ridicarea cul-
turală a neamului românesc, chiar dacă cel dintâi şi-a distrus opera, fără ı̂nsă a putea
opri curentul progresist, care prinsese rădăcini adânci ı̂n perioada sa.

Prof. dr. Adriana RADU


Colegiul Naţional din Iaşi
17 Ibidem, p. 116-130.

49
Concursul interjudeţean ,,Speranţe Olimpice”
Ediţia a XVI-a, Paşcani, 5 noiembrie 2016

Clasa a III-a
1. a) Aflaţi numărul a din egalitatea: 389 − 17 − 29 − a = 100.
b) Care dintre următoarele numere poate fi suma a cinci numere consecutive im-
pare: 80; 310; 402?
c) Să se culculeze suma numerelor pare de forma ab ı̂n care a, b sunt cifre conse-
cutive ı̂n ordine crescătoare.
Mihai Crăciun
2. a) Aflaţi numerele x, y ştiind că x + y + xy + yx = x0y.
b) Este posibil să se grupeze numerele de la 91 la 121 (inclusiv) ı̂n două grupe
astfel ı̂ncât una să conţină numai numere pare şi cealaltă să conţină numai numere
impare şi suma numerelor din fiecare grupă să fie aceeaşi?
Mihai Crăciun
3. a) Aflaţi suma a 7 numere consecutive ştiind că trei dintre ele sunt 100, 101, 102.
b) Aşezaţi numerele 50, 150, 250, 350, 450, 550, 650, 750, 850 ı̂n trei grupe astfel
ı̂ncât suma numerelor din fiecare grupă să fie aceeaşi.
c) Într-o urnă sunt bile. Triplăm numărul bilelor şi scoatem din cutie 17 bile, apoi
triplăm numărul bilelor rămase şi iar scoatem 17 bile şi aşa mai departe. Putem goli
cutia prin repetarea acestei operaţii?
Recreaţii Matematice, 2/2012

Clasa a IV-a
1. a) Aflaţi a dacă 88 − 70 × [60 − 5 × (132 : 4 − 63 : a) + 1] − 6 = 12.
b) În urmă cu 11 ani, suma vârstelor surorilor lui Mihai era de 18 ani. Acum,
suma vârstelor surorilor lui Mihai este de 73 ani. Câte surori are Mihai?
c) Suma a patru numere impare consecutive este 2016. Aflaţi numerele.
Mihai Crăciun
2. Se dă şirul 1, 14, 27, 40, . . . .
a) Completaţi şirul cu ı̂ncă cinci termeni.
b) Stabiliţi dacă 2016 este termen al şirului.
c) Aflaţi al 2017-lea termen al şirului.
Mihai Crăciun
3. a) Determinaţi numerele xy cu proprietatea că x3y − y2x = 109.
b) Suma a trei numere a, b, c este 160. Dacă ı̂nsumăm triplul lui a cu triplul lui b
şi cu ı̂mpătritul lui c, obţinem 500. Care sunt cele trei numere, dacă numărul c este
diferenţa dintre a şi b?

50
c) Pentru un număr n natural nenul, notăm cu S(n) suma cifrelor sale (de exem-
plu: S(247) = 2 + 4 + 7 = 13). Câte numere de forma abc ı̂ndeplinesc condiţia că
S(S(abc)) = 10 ?
Recreaţii Matematice, 1/2014

Clasa a V-a
1. a) Scrieţi numărul 53 ca sumă de două pătrare perfecte şi numărul 281 ca sumă
de trei pătrate perfecte.
b) Arataţi că, dacă n este număr natural par, atunci numărul 53n ·281n+1 se poate
scrie ca sumă de trei pătrate perfecte, iar dacă n este număr natural impar, atunci
numărul 53n · 281n+1 se poate scrie ca sumă de două pătrate perfecte.
2. a) Este posibil ca, după ı̂nlocuirea steluţelor din relaţia 1 ∗ 5 ∗ 9 ∗ 13 ∗ 17 ∗ . . . ∗
2017 = 2016 cu semnele + sau –, să obţinem o relaţie adevărată?
b) Pe tablă sunt scrise numerele 2, 1, 1, 7. Ştergem de pe tablă oricare două numere
şi scriem ı̂n locul lor succesorii acestora. Este posibil ca ı̂n urma mai multor operaţii
de acest fel să obţinem patru numere egale?
3. a) Fie numărul N = 63 + 632 + 633 + . . . + 632016 . Aflaţi ultimele şase cifre ale
numărului 56 · N .
b) Pe un ecran este scris numărul 1. După primul minut, după numărul iniţial
se scrie un număr de 4 ori mai mare. Apoi, după fiecare minut, este scris numărul
de patru ori mai mare decât suma tuturor numerelor dinaintea lui. Aflaţi după câte
minute suma numerelor care au apărut pe ecran este 12510 .

Clasa a VI-a
1. Gasiţi numerele prime p, q, r, distincte, ştiind că

1 1 1 1
+ + − ∈ N.
p q r pqr

Gazeta Matematică, 11/2011


2. Determinaţi numerele naturale nenule a, b, c şi d pentru care ad = bc, ab + cd =
50 şi a ≤ c ≤ b.
Recreaţii Matematice, 1/2016
3. a) Determinaţi cifrele a, b, c, d, e, f ∈ {1, 2, 3, 4, 5, 6, 7, 8}, a, b, d ≥ 2, cu propri-
etatea că relaţia a·bc = def nu are loc, dar dacă mărim cu 1 fiecare cifră, noua relaţie
este adevărată, iar dacă micşoram cu 1 fiecare cifră obţinem o altă relaţie adevărată.
b) Dacă numerele naturale x, y, z verifică x2 + y 2 − 10z = 1, atunci 10 | xy.
Andrei Eckstein
Clasa a VII-a
n+1
1. a) Un număr raţional se numeşte ,,bun” dacă este de forma , n ∈ N∗ . Fie
n
k ∈ N, k ≥ 2. Să se arate că orice număr bun se scrie ca produs de k numere bune.
Andrei Eckstein

51
a − 2016 b+1
b) Determinaţi numerele naturale nenule a şi b astfel ı̂ncât şi
b a − 2016
să fie numere naturale.
2. a) Dacă a, b sunt numere reale mai mari decât 1 şi n este un număr natural
1 1 1
nenul, arătaţi că ab + n n > n−1 + n−1 .
a b a b
Recreaţii Matematice, 2/2015
1 1 1
b) Câte soluţii naturale are ecuaţia + = ?
x y 2016
3. a) În interiorul dreptunghiului ABCD se consideră punctul M astfel ı̂ncât
m(M ÷ BC) = m(M ÷ ÷
CB) = 15◦ . Ştiind că AB = 2BC, aflaţi m(AM D).
b) Se dă triunghiul ABC şi punctele D, E ∈ (BC) astfel ı̂ncât AB = BD şi
AC = CE. Dacă I este centrul cercului ı̂nscris ı̂n triunghiul ABC şi AI = DE,

calculaţi m(BAC).

Clasa a VIII-a
1. a) Dacă a, b, c ∈ (0, ∞), arătaţi că are loc inegalitatea:

a b c 3
b+c
+ 
c+a + a+b
≥ .
a + min a, 2 b + min b, 2 c + min c, 2 2

Traian Tămâian
b) Fie a, b, c, d ∈ (0, ∞) astfel ı̂ncât ab(c + d) ≥ (a + b)cd şi ab + cd ≥ (a + b)(c + d).
Comparaţi numerele a + b şi c + d.
Recreaţii Matematice, 1/2013
2. a) Rezolvaţi ı̂n Z × Z ecuaţia:

3x + 4x + 5x + 6x + 7x = y 2 .

RMT, 1/2012
b) Rezolvaţi ecuaţia:
… … …
3 1 1 1 1 1 1 1
+ 1 + 2 + 2 + 1 + 2 + 2 + ... + 1 + + = 2016 + x − .
2 2 3 3 4 20152 20162 2016
3. a) Fie V ABCD o piramidă patrulateră regulată, {O} = AC ∩ BD. Punctul
M este mijlocul lui V O, punctul N este mijlocul lui BM , iar P ∈ (AO) astfel ı̂ncât
AP = 3P O. Arătaţi că P N k(V DC).
Gazeta Matematică, 1/2015
′ ′ ′ ′ ′ M A′
b) Fie cubul ABCDA B C D de latură a şi M ∈ [A C] cu proprietatea că =
MC
3
. Să se determine un punct P ∈ [AA′ ] astfel ı̂ncât suma P M + P C ′ să fie minimă şi
2
să se afle lungimea segmentului P A′ .
Petru Asaftei

52
Soluţiile problemelor propuse ı̂n nr. 2/2016
Clasele primare
P355. Găsiţi trei numere consecutive ı̂n şirul numerelor de la 1 la 30 care să aibă
suma 30.
(Clasa pregătitoare) Mariana Manoli, elevă, Iaşi
Soluţie. 9 + 10 + 11 = 30.
P356. Colorează figura geometrică care nu se află la stânga pătratului şi nu este
dreptunghi:  △
(Clasa pregătitoare) Mihaela Munteanu, elevă, Iaşi
Soluţie. Triunghiul ı̂ndeplineşte condiţiile problemei.
P357. Fie şirul 1; 2; 3; 2; 3; 5; 3; ; . Completaţi casetele libere respectând regula
de formare şi găsiţi cifra care apare de cele mai multe ori ı̂n şirul obţinut.
(Clasa I ) Emanuela Păduraru, elevă, Iaşi
Soluţie. 1 + 2 = 3, 2 + 3 = 5 şi urmează 3 + 4 = 7. Şirul se completează cu cifrele
4 şi 7. Cifra 3 apare cel mai des ı̂n şir.
P358. Care dintre numerele 23, 12, 3, 45, 76 este cel mai ı̂ndepărtat de 34?
(Clasa I ) Mădălina Ciobanu, elevă, Iaşi
Soluţie. 34 − 3 = 31, 34 − 12 = 22, 34 − 23 = 11, 45 − 34 = 11, 76 − 34 = 32.
Numărul cel mai ı̂ndepărtat de 34 este 76.
P359. Aflaţi toate numerele de două cifre care au suma cifrelor mai mică decât
suma cifrelor vecinului mai mic. (Exemplu: numărul 50 ı̂ndeplineşte condiţia deoarece
are suma cifrelor mai mică decât suma cifrelor numărului 49.)
(Clasa I ) Mihaela Buleandră, elevă, Iaşi
Soluţie. Numerele 10, 20, 30, 40, 50, 60, 70, 80, 90 ı̂ndeplinesc condiţia cerută.
P360. Scrie ı̂n casete numai numere formate cu cifra 3 astfel ı̂ncât să fie corect
calculul + + = 399.
(Clasa a II-a) Ana Stoica, elevă, Iaşi
Soluţie. Un exemplu este: 33 + 33 + 333 = 399.
P361. Un ţăran avea 60 de păsări, găini şi raţe. După ce vinde 18 găini şi câteva
raţe, observă că numărul găinilor s-a ı̂njumătăţit, iar cel al raţelor s-a redus la un
sfert. Câte raţe i-au rămas?
(Clasa a II-a) Ana Ionescu, elevă, Iaşi
Soluţie. Ţăranul avea 18 + 18 = 36 găini şi 60 − 36 = 24 raţe. Ţăranului i-au
rămas 24 : 4 = 6 raţe.
P362. Suma a trei numere impare consecutive este 39. Care sunt aceste numere?
(Clasa a II-a) Teodor Pătraşcu, elev, Iaşi
Soluţie. Diferenţa dintre două numere impare consecutive este 2; 39 − (2 + 4) =
39 − 6 = 33. Deoarece 33 = 11 + 11 + 11, ı̂nseamnă că numerele sunt 11, 13 şi 15.

53
P363. Aflaţi valoarea numărului a pentru care
10 × 10 − 10 × [10 × 10 − 10 × (10 − 10 : a)] = 0.
(Clasa a III-a) Nicolae Ivăşchescu, Canada
Soluţie. Deducem că 10 × 10 − 10 × (10 − 10 : a) = 10, de unde 10 × (10 − 10 :
a) = 90, ceea ce ı̂nseamnă că 10 − 10 : a = 9 şi a = 10.
P364. Putem ı̂mpărţi numerele 11, 12, 13, 14, 15, 16, 17, 18, 19 ı̂n trei grupe de câte
trei numere astfel ı̂ncât ı̂n fiecare grupă suma numerelor să fie pară? Justificaţi!
(Clasa a III-a) Adina Relinschi, elevă, Iaşi
Soluţie. În şirul numerelor date cinci numere sunt impare, deci suma tutror
numerelor date este un număr impar. Dacă am putea forma cele trei grupe, am
ajunge la concluzia că suma tuturor numerelor este un număr par, fals. Împărţirea
ı̂n condiţiile cerute nu se poate face.
P365. Suma a două numere este un număr de două cifre ce are suma cifrelor 12.
Aflaţi cele două numere ştiind că triplul unuia este dublul celuilalt.
(Clasa a III-a) Maria Crăcană, elevă, Iaşi
Soluţie. Dacă numerele sunt x şi y, x < y, 3x = 2y, deducem că 2(x + y) =
2x + 2y = 2x + 3x = 5x. Obţinem că numărul x + y se ı̂mparte exact la 5. Din relaţia
x + y = ab, a + b = 12 şi ab se ı̂mparte exact la 5, deducem a = 7, b = 5. Rezultă
x = 30, y = 45.
P366. Arătaţi că, oricum am alege şase numere dintre numerele 10, 12, 14, 16, 18,
20, 22, 24, 26, 28, există două astfel ı̂ncât ultima cifră a sumei lor este zero.
(Clasa a III-a) Ecaterina Brı̂nzac, elevă, Iaşi
Soluţie. Formăm perechile: (10, 20), (12, 18), (14, 16), (22, 28), (24, 26). Oricum
am alege şase numere, două sunt din aceeaşi pereche, deci au suma un număr cu
ultima cifră zero.
P367. Fie a şi b două numere naturale nenule astfel ı̂ncât 2 × b − 3 × a = 17.
Arătaţi că b ≥ 10.
(Clasa a IV-a) Ionuţ-Florin Voinea, elev, Bucureşti
Soluţie. Presupunem că b < 10. Cum a ≥ 1, atunci 2b − 3a < 17, fals. Deci
b ≥ 10.
P368. Arătaţi că numărul 1 × 3 × 5 × . . . × 51 − 1 × 3 × 5 × . . . × 49 se ı̂mparte
exact la numărul 47 × 49 × 50.
(Clasa a IV-a) Cristina Chelaru, elevă, Iaşi
Soluţie. 1 × 3 × 5 × . . .× 51 − 1 × 3 × 5 × . . .× 49 = 1 × 3 × 5 × . . .× 49 × (51 − 1) =
1 × 3 × 5 × . . . × 47 × 49 × 50, de unde concluzia.
P369. Figura alăturată s-a format prin alipirea drept- F E D
unghiurilor ABEF şi BCDE. În câte moduri putem să
alegem trei laturi din cele şapte existente astfel ı̂ncât ori-
care două dintre ele să nu aibă capete comune?
(Clasa a IV-a) Daniela Mititelu, elevă, Iaşi A B C
Soluţie. Latura AF poate intra ı̂n grupele (AF, BE, CD) şi (AF, BC, DE), iar
latura AB poate intra numai ı̂n grupa (AB, EF, CD). Alte grupe nu mai sunt.
Alegerea se poate face ı̂n 3 moduri.

54
1 1 2 1 2 3 1 2 3 4
P370. Se consideră şirul de fracţii , , , , , , , , , , . . . Ce fracţie tre-
1 2 1 3 2 1 4 3 2 1
buie scrisă pe locul 57?
(Clasa a IV-a) Petru Asaftei, Iaşi
Soluţie. Numărătorii fracţiilor pot fi organizaţi astfel: 1; 1, 2; 1, 2, 3; 1, 2, 3, 4; . . .
Cum 1 + 2 + 3 + . . . + 10 = 55, rezultă că numărătorul fracţiei de pe locul 57 se află
2
pe locul doi ı̂n secvenţa 1, 2, 3, . . . , 11. Pe locul 57 este fracţia .
10

Clasa a V-a
V.207 Determinaţi mulţimile A şi B ştiind că sunt ı̂ndeplinite simultan condiţiile:
(i) A∪B = {1, 2, 3, 4, 5, 6, 8}; (ii) A∩B = {1, 2}; (iii) 8 ∈ / A\B; (iv) suma elementelor
lui A este egală cu suma elementelor lui B.
Valeriu Iovan, Craiova
Soluţie. Se arată că A = {1, 2, 3, 4, 6} şi B = {1, 2, 5, 8}.
V.208 Numerele naturale a, b şi c sunt astfel ı̂ncât 2ab = c, bc = 8a şi ca = 2b.
Calculaţi suma S = a + b + c.
Ionuţ-Florin Voinea, elev, Bucureşti
Soluţie. Dacă unul dintre numere este 0, atunci şi celelalte vor fi 0, prin urmare
S = 0. Dacă toate sunt nenule, ı̂nmulţind membru cu membru egalităţile din enunţ,
obţinem că 2a2 b2 c2 = 16abc, de unde abc = 8. Deducem că a = 1, b = 2, c = 4, deci
S = 7. În concluzie, S ∈ {0, 7}.
V.209 Determinaţi cel mai mic şi cel mai mare dintre numerele naturale de 25
cifre care au suma cifrelor 25 şi sunt divizibile cu 25.
Constantin Dragomir, Piteşti
Soluţie. Cel mai mic număr căutat este 1 00 | .{z
. . 00} 3975, iar cel mai mare este
20 de 0
997 00 . . 00}.
| .{z
22 de 0

V.210 Dacă numărul A = (a − b)(99a + 100b) + (a + b)(111a + 112b), a, b ∈ N,


este divizibil cu 3, arătaţi că cel puţin unul dintre numerele a şi b este divizibil cu 3.
Teodor-Ioan Băltoi, elev, Roman
Soluţie. Cum 99 = M3 , 100 = M3 + 1, 111 = M3 , 112 = M3 + 1, rezultă că
A = (a − b)(M3 + b) + (a + b)(M3 + b) = M3 + (a − b) · b + (a + b) · b = M3 + 2ab.
Deducem că 2ab = M3 , de unde cerinţa problemei.
V.211 Pe un teren dreptunghiular cu aria de 900 m2 se construieşte o piscină
dreptunghiulară cu aria de 400 m2 . Atât dimensiunile terenului cât şi cele ale piscinei
se exprimă, ı̂n metri, prin numere naturale divizibile cu 5. De jur ı̂mprejurul terenului
şi de jur ı̂mprejurul piscinei se află câte un gard. În condiţiile problemei, care este
lungimea totală minimă a acestor garduri?
Vlad-Mihai Ciuperceanu, elev, Craiova
Soluţie. Fie L = 5a, l = 5b, a, b ∈ N∗ , dimensiunile terenului; atunci a · b = 36 şi

55
20 10

a + b = minim. Calculând suma a + b pentru


perechile (a, b) ∈ {(1, 36); (2, 18); (3, 12); (4, 9); (6, 6); (9, 4); 20
(12, 3); (18, 2); (36, 1)}, obţinem valoare minimă când a = b = 6,
deci terenul are perimetru minim când are forma unui pătrat cu
latura de 30 m. Analog, piscina va avea forma unui pătrat cu 10
latura de 20 m.
Lungimea totală minimă a gardurilor se obţine când acestea au o porţiune (cât
mai mare) comună, adică ı̂n situaţia din figură şi este egală cu 4 × 30 + 2 × 20 = 160m.

V.212 În vârfurile unui cub sunt scrise cinci numere egale cu 0 şi trei numere egale
cu 1. Un pas ı̂nseamnă mărirea cu 1 a numerelor din vârfurile unei muchii oarecare.
Este posibil ca, după un număr de paşi, cele opt numere din vârfurile cubului să devină
egale?
Ioan-Viorel Codreanu, Satulung (Maramureş)
Soluţie. Suma celor opt numere din vârfurile cubului se măreşte la fiecare pas
cu 2, deci nu ı̂şi schimbă paritatea. Iniţial, această sumă este egală cu 3, deci este
impară şi atunci va fi mereu impară. Dacă cele opt numere ar deveni egale, suma lor
ar fi pară. Rezultă că raspunsul la ı̂ntrebarea din problemă este negativ.
V.213 Se consideră produsul P = ab · cd · e. Înlocuiţi cele cinci litere cu cifrele
1, 2, 3, 4, 5, folosind fiecare cifră câte o singură dată, astfel ı̂ncât produsul P să fie
maxim. (Enunţ corectat.)
Gabriel Popa, Iaşi
Soluţie. Avem: P = (10a + b)(10c + d)e = 100ace + 10e(ad + bc) + bde. Pentru a
maximiza P , impunem ı̂ntâi ca ace să fie maxim, deci {a, c, e} = {3, 4, 5} şi {b, d} =
{1, 2}, apoi ca e(ad + bc) să fie maxim. Această condiţie revine la e(2a + c) =maxim
sau e(a + 2c) = maxim şi, verificând pentru cele 6 permutări posibile ale literelor
a, c, e, obţinem că e = 5, a = 4, c = 3, respectiv e = 5, a = 3, c = 4. În concluzie,
Pmax = 41 · 32 · 5 = 6560 pentru (a, b, c, d, e) ∈ {(4, 1, 3, 2, 5); (3, 2, 4, 1, 5)}.

Clasa a VI-a
VI.207. Pentru a promova un examen, trebuie susţinute şi promovate patru probe.
Dintre elevii unei şcoli care participă la acest examen, 70% au trecut proba A, 80%
au trecut proba B, 75% au trecut proba C şi 85% au trecut proba D. Arătaţi că cel
puţin 10% dintre elevii şcolii au promovat examenul.
Neculai Stanciu, Buzău
Soluţie. Observăm că 30% nu au trecut proba A, 20% nu au trecut proba B,
25% nu au trecut proba C şi 15% nu au trecut proba D. În total, cel mult (30 + 20 +
25 + 15)% = 90% nu au trecut una dintre probe, prin urmare cel puţin 10% dintre
elevii şcolii le-au trecut pe toate, deci au promovat examenul.
n3 − 3
VI.208. Notăm cu P produsul numerelor naturale de forma , n ∈ N.
n+3
Determinaţi numărul divizorilor naturali ai numărului P.
Ionel Tudor, Călugăreni şi Viorica Dogaru, Giurgiu

56
n3 − 3
Soluţie. Dacă ∈ N, atunci n+3|n3 −3, prin urmare n+3|n2 (n+3)−(n3 −3),
n+3
adică n + 3|3n2 + 3. Apoi, n + 3|3n(n + 3) − (3n2 + 3), deci n + 3|9n − 3. Rezultă
că n + 3|9(n + 3) − (9n − 3), prin urmare n + 3|30. Obţinem că n ∈ {2, 3, 7, 12, 27},
n3 − 3
aşadar numerele de forma sunt 1, 4, 34, 115 şi 656. Descompus ı̂n factori primi,
n+3
produsul acestor numere este P = 27 · 5 · 17 · 23 · 41, iar numărul divizorilor lui P este
8 · 2 · 2 · 2 · 2 = 128.
VI.209. Demonstraţi că orice număr natural n admite o scriere unică n = a0 +
a1 · 3 + a2 · 32 + . . . + ak · 3k , unde k ∈ N şi a0 , a1 , . . . , ak ∈ {−1, 0, 1}.
Scrieţi numărul 2016 sub această formă.
Gheorghe Iurea, Iaşi
Soluţie. Pentru unicitatea scrierii, presupunem că n = a0 + a1 · 3 + a2 · 32 +
.
. . . = b + b · 3 + b · 32 + . . .; atunci a − b ..3 şi, cum a − b ∈ {−2, −1, 0, 1, 2},
0 1 2 0 0 0 0
rezultă că a0 = b0 . Scădem a0 din ambii membri, ı̂mpărţim prin 3 şi obţinem că
.
a + a · 3 + . . . = b + b · 3 + . . . , de unde a − b ..3, deci a = b . Din aproape ı̂n
1 2 1 2 1 1 1 1
aproape, ai = bi pentru orice i ∈ N.
Pentru a dovedi existenţa unei astfel de scrieri, descompunem numărul n ı̂n baza
3 : n = xp xp−1 . . . x1 x0 , unde xi ∈ {0, 1, 2}, deci n = xp ·3p +xp−1 ·3p−1 +. . .+x1 ·3+x0 .
Începând de la sfârşit, fie xt prima cifră egală cu 2; atunci n = xp · 3p + . . . + xt+1 ·
3t+1 + (3 − 1) · 3t + xt−1 · 3t−1 + . . . + x1 · 3 + x0 = xp · 3p + . . . + (xt+1 + 1)3t+1 − 3t +
xt−1 · 3t−1 + . . . + x1 · 3 + x0 . Continuăm procedeul, până eliminăm toate numerele
xi egale cu 2.
Avem: 2016 = 2202200(3) = 2 · 36 + 2 · 35 + 2 · 33 + 2 · 32 = 37 − 36 + 36 − 35 +
3 − 33 + 33 − 32 = 37 − 35 + 34 − 32 .
4

VI.210. Dacă n ∈ N∗ , notăm (2n)!! = 2·4·6·. . .·(2n). Determinaţi k, x ∈ N, k 6= 0,


pentru care (2k)!! = x2 + 2.
Denisa Drăghia, elevă, Craiova
Soluţie. Pentru k = 1, obţinem x2 + 2 = 2, deci x = 0. Când k > 1, avem
că (2k)!! este multiplu de 4, prin urmare x2 = M4 + 2, contradicţie. Rămâne unica
soluţie k = 1, x = 0.
VI.211. Fie x, y ∈ N∗ , x > y, astfel ı̂ncât x − 2016y 2 = y − 2015x2 . Arătaţi că
x − y este pătrat perfect.
Iulian Oleniuc, elev, Iaşi
Soluţie. Observăm că x2 = 2016(x2 − y 2 ) + (x − y) = (x − y)[2016(x + y) + 1] şi
y 2 = 2015(x2 − y 2 ) + (x − y) = (x − y)[2015(x + y) + 1]. Se arată uşor că numerele
2016(x + y) + 1 şi 2015(x + y) + 1 sunt relativ prime, prin urmare x − y = (x2 , y 2 ) =
(x, y)2 , deci x − y este pătrat perfect.
VI.212. Pe latura AB a triunghiului ascuţitunghic ABC se fixează un punct F
’ ) + 2m(BCF
astfel ı̂ncât m(ACF ’ ) < 90◦ . Determinaţi poziţiile punctelor M ∈ (BC)
şi N ∈ (AC) pentru care suma F M + M N ste minimă.
Mariana-Liliana Popescu, Suceava
Soluţie. Pentru un punct fixat N ∈ (AC), punctul M ∈ (BC) care minimizează

57
A
′ ′
suma F M + M N este {M } = F N ∩ BC, unde F este si-
metricul lui F faţă de BC. Într-adevăr, pentru M ′ ∈ BC,
avem: F M ′ +M ′ N = F ′ M ′ +M ′ N ≥ F ′ N = F ′ M +M N. N
Apoi, suma F M + M N = F ′ N va fi minimă când F
F ′ N ⊥ AC, deci când N = P rAC F ′ . Din ipoteza proble-
mei, unghiul F ÷ ′ CA este ascuţit, prin urmare N ∈ (AC).
B C
M M
VI.213. Pe foaie este desenat un triunghi şi două
dintre mediatoarele sale. Folosind doar o riglă negradată,
construiţi cea de-a treia mediatoare. F
Nicolae Ivăşchescu, Craiova
Soluţie. Fie d1 şi d2 mediatoarele desenate ale laturilor BC şi AC ale triunghiului
ABC, care trec prin mijloacele M respectiv N ale acestor laturi şi se intersectează
ı̂n O. Fie {G} = AM ∩ BN centrul de greutate al triunghiului şi {P } = CG ∩ AB;
atunci P va fi mijlocul lui AB, iar cea de-a treia mediatoare este dreapta OP. Toate
construcţiile descrise se pot efectua folosind doar rigla negradată.

Clasa a VII-a
VII.207. Determinaţi numerele reale x şi y pentru care x3 + y 3 = x4 + y 4 = 1.
Viorica Momiţă, Iaşi
Soluţie. Dacă x < 0, y < 0, atunci x3 + y 3 < 0, fals. Dacă x < 0 şi y > 0, atunci
y 3 = 1 − x3 > 1, deci y > 1; deducem că x4 + y 4 > 0 + 1 = 1, fals. Rămâne că x ≥ 0,
y ≥ 0 şi, cum x4 + y 4 = 1, rezultă că 0 ≤ x ≤ 1 şi 0 ≤ y ≤ 1. În aceste condiţii,
1 = x3 + y 3 ≥ x4 + y 4 = 1, de unde (x, y) ∈ {(0, 1); (1, 0)}.
VII.208. Demonstraţi că numărul N = 2016n+1 − 2015n − 2016, n ∈ N∗ , are cel
puţin 27 de divizori naturali.
Alessandro Ventullo, Milano, Italia
Soluţie. Folosind faptul că (a + 1)n = Ma2 + na + 1, obţinem că 2016n+1 =
M20152 + (n + 1) · 2015 + 1 şi atunci N se divide cu 20152 . Cum 20152 = 52 · 132 · 312
are 27 de divizori, urmează concluzia problemei.
VII.209. Arătaţi că, pentru orice număr real a, are loc identitatea (10a2 − 8a)2 +
(11a2 − 20a + 8)2 + (12a2 − 20a + 8)2 = (13a2 − 20a + 8)2 + (14a2 − 20a + 8)2 .
Marian Tetiva, Bârlad
Soluţie. Avem: (13a2 − 20a + 8)2 − (11a2 − 20a + 8)2 + (14a2 − 20a + 8)2 − (12a2 −
20a + 8)2 = 2a2 (24a2 − 40a + 16) + 2a2 (26a2 − 40a + 16) = 2a2 (50a2 − 80a + 32) =
4a2 (5a − 4)2 = (10a2 − 8a)2 .
VII.210. Se consideră triunghiul isosel ABC cu m(B) “ = m(C) “ = 40◦ . Notăm
cu E piciorul bisectoarei din B şi fie P ∈ (BC) astfel ı̂ncât P E = P C. Arătaţi că
AB 2
PABE = .
AE
Mirela Marin, Iaşi
Soluţie. Din asemănarea evidentă △P EC∼△ACB, obţinem că AB · CE=P E ·
BC.

58
A
Folosind teorema bisectoarei, deducem că AE ·
BC = AB · CE; atunci P E · BC = AE ·
BC, aşadar AE = P E. Se observă uşor că E
m(BP ’ E) = m(BEP ’ ) = 80 , prin urmare BE =

BP. Astfel, BC = BP + P C = BE + AE;


ı̂nlocuind ı̂n teorema bisectoarei, obţinem: B P C
EC BE + AE EC + EA BE + AE + AB AC PABE
= ⇒ = ⇒ = ⇒ PABE =
EA AB EA AB AE AB
2
AB
.
AE
VII.211. Fie M şi N mijloacele laturilor neparalele AD respectiv BC ale trapezu-
lui ABCD. Dacă {S} = AN ∩ CM, {T } = BM ∩ DN, demonstraţi că ST kAB şi
calculaţi lungimea segmentului ST ı̂n funcţie de bazele trapezului.
Mihail Frăsilă şi Constantin Petrea, Paşcani
Soluţie. Notăm a = AB, b = CD (a > b), {Q} = AN ∩ BM, {O} = M C ∩ N D,
2bx
{E} = AD ∩BC şi x = N B = N C. Din asemănări imediate, obţinem că EC = ,
a−b
(a + b)x N B a − b DE EC 2b
NE = , = , = = . Teorema lui Menelaus aplicată
a−b NE a + b DM CN a−b
DE T M N B
ı̂n △EM B cu transversala T − N − D arată că · · = 1, de unde
DM T B N E
TB 2b
= . Tot din teorema lui Menelaus, ı̂n △BN T cu transversala C − O − M,
TM a+b
ON M T BC ON a−b
deducem că · · = 1, prin urmare = . Analog se arată că
N T M B CN OT 2(a + b)
OM a−b ON MN a−b a+b
= şi, de aici, M N kST. Acum, = , aşadar = ,
OS 2(a + b) OT ST 2(a + b) 2ST
2
(a + b)
adică ST = .
a−b
VII.212. Punctul P este situat pe arcul mic BC ˜ al cercului circumscris triun-
ghiului echilateral ABC. Notăm {M } = BC ∩ AP. Ştiind că 19PABC = 27PP BC ,
arătaţi că AM = 0, 9 · BC.
Constantin Petrea, Paşcani
Soluţie. Notăm a = AB, b = AM ; din asemănarea △ABM ∼ △AP B obţinem că
AM AB a2 A
= , prin urmare AP = . Din teorema lui
AB AP b
Van Schooten, avem că P A = P B + P C, deci PP BC =
a2
BC + P A = a + . Ipoteza problemei conduce la 57a =
b
a2
27(a + ), de unde concluzia este imediată.
b
VII.213. Se consideră triunghiul ABC dreptunghic ı̂n M
A. Bisectoarea CD a unghiului C, “ D ∈ AB, intersectează B C
perpendiculara ı̂n B pe BC ı̂n punctul E. Demonstraţi că
P
2BD2 = CE · DE.
Cătălin Cristea, Craiova

59
Soluţie. Fie BF ⊥ DE, F ∈ DE; deoarece △BDE este isoscel

(m(BDE) = m(BED)’ = 90◦ − 1 m(C)), “ rezultă că DF = C
2
F E. Aplicăm teorema catatei ı̂n △BCE : BE 2 = CE ·
DE
F E ⇒ BD2 = CE · şi, de aici, cerinţa problemei.
2

Clasa a VIII-a
A D B
VIII.207. Fie x, y numere reale astfel ı̂ncât xy = x + y.
Arătaţi că x şi y fie sunt simultan numere raţionale, fie sunt F
simultan numere iraţionale. E
Claudiu-Ştefan Popa, Iaşi
Soluţie. Relaţia din enunţ se poate scrie sub forma (x + 1)(y + 1) = 1. Este
evident că x 6= −1 şi y 6= −1. Dacă, prin absurd, x ∈ Q şi y ∈ R\Q, atunci x + 1 ∈ Q∗ ,
y + 1 ∈ R\Q, deci (x + 1)(y + 1) ∈ R\Q, contradicţie.
VIII.208. Determinaţi numerele reale x, y, z cu proprietăţile: x − y + z = 2,
xz − yz = (xy − 1)2 , x2 + y 2 + z 2 = 6.
Bogdan Chiriac, Bacău
Soluţie. Ridicând la pătrat prima relaţie şi ţinând cont de celelalte două, obţinem
că 6 − 2xy + 2(xy − 1)2 = 4, deci (xy)2 − 3(xy) + 2 = 0, de unde xy ∈ {1, 2}. Rezultă
că sistemul dat este echivalent cu sistemele:
(i) x − y = 2 − z; xy = 2; z(x − y) = 1 şi
(ii) x − y = 2 − z; xy = 1; z(x − y) = 0.
Sistemul (i) conduce la ecuaţia z 2 − 2z + 1 = 0, cu soluţia z = 1, apoi la x − y = 1,
xy = 2, adică (x, y, z) ∈ {(2, 1, 1); (−1, −2, 1)}. Sistemul (ii) conduce la ecuaţia
z(2
√ − z) = √ 0, cu soluţiile
√ z ∈ {0, √ 2} şi, după ı̂nlocuire, obţinem (x, y, z) ∈ {(1 +
2, −1 + 2, 0); (1 − 2, −1 − 2, 0); (1, 1, 2); (−1, −1, 2)}. În total, sistemul din
enunţ are şase soluţii,
VIII.209. Determinaţi perechile de numere ı̂ntregi (x, y) 6= (0, 0) pentru care
x4 + 2x3 = x + y + y 2 şi y 4 + 2y 3 = y + x + x2 .
Vasile Chiriac, Bacău
Soluţie. Scăzând ecuaţiile membru cu membru, obţinem:

(x − y)[(x3 + x2 y + xy 2 + y 3 ) + 2(x2 + xy + y 2 ) + (x + y)] = 0


⇔ (x − y)[x2 (x + y + 1) + y 2 (x + y + 1) + (x + y)(x + y + 1)] = 0
⇔ (x − y)(x + y + 1)(x2 + y 2 + x + y) = 0.

Dacă x = y, ecuaţiile din enunţ devin x4 + 2x3 − x2 − 2x = 0 ⇔ x(x − 1)(x +


1)(x+2) = 0, deci x ∈ {0, 1, −1, −2}; găsim soluţiile (1, 1); (−1, −1) şi (−2, −2). Dacă
y = −x − 1, ecuaţiile sunt echivalente cu x4 + 2x3 − x2 − 2x = 0 (la fel ca mai sus);
găsim soluţiile (0, −1); (1, −2); (−1, 0) şi (−2, 1). In sfârşit, dacă x2 + y 2 + x + y = 0,
cum x(x + 1) ≥ 0, y(y + 1) ≥ 0, ∀x, y ∈ Z, rezultă că x, y ∈ {−1, 0} şi nu vom găsi
soluţii noi.

60
Soluţiile sistemului din enunţ sunt (−2, −2); (−1, −1); (1, 1); (0, −1); (−1, 0);
(1, −2) şi (−2, 1).
VIII.210. Dacă x, y, z ∈ (0, 1], demonstraţi că:
xy yz zx
+ + ≤ 1.
xy + x + y yz + y + z zx + z + x
Ovidiu Pop, Satu-Mare
1
Soluţie. În fapt, fiecare dintre termenii sumei este cel mult egal cu . Într-adevăr,
3
xy x x x 1
= x ≤ = ≤ , ∀x, y ∈ (0, 1].
xy + x + y x+1+ y x+1+x 2x + 1 3

VIII.211. Dacă a, b, c ∈ R∗+ sunt astfel ı̂ncât a + b + c = 6, arătaţi că


2016 + 2b 2016 + 2c 2016 + 2a
+ + ≥ 606.
a+8 b+8 c+8
Mihaela Berindeanu, Bucureşti
P 2016 + 2b P 1 P b+8
Soluţie. Observăm că = 2000 · +2· . Din ine-
a+8 a+8 a+8
P 1 (1 + 1 + 1)2 3
galitatea lui Bergström, ≥ = ; din inegalitatea mediilor,
… a+8 a + b + c + 24 10
P b+8 Q b+8
≥33 = 3. Acum, cerinţa problemei este imediată.
a+8 a+8
VIII.212. Două plane perpendiculare intersectează o sferă de rază R după două
cercuri de raze a respectiv b. Dacă distanţa√ de la centrul sferei la dreapta de intersecţie
a planelor este c, arătaţi că a + b + c ≤ R 6.
Maria Rusu, Târgu Frumos
Soluţie. Fie O, O1 , O2 centrul sferei şi centrele celor două cercuri de secţiune,
x = OO1 şi y = OO2 ; atunci x2 + y 2 = c2 , x2 + a2 =   R2 şi y 2 + b2 = R2 . Obţinem că

2 2 2
a b c (a + b + c)2 1
a2 + b2 + c2 = 2R2 , de unde R = + + ≥ = √ (a + b + c),
2 2 2 2+2+2 6
conform inegalităţii lui Bergström.
VIII.213. Se consideră tetraedrul V ABC cu V A ⊥ V B ⊥ V C ⊥ V A. Notăm cu
S1 , S2 , S3 ariile triunghiurilor V AC, V AB şi V BC şi cu h distanţa de la V la planul
(ABC). Arătaţi că
1 S1 S2 S3
≥ 2 2 + 2 2 + 2 .
h 2 S2 + S3 S3 + S1 S1 + S22
Cătălin Calistru, Iaşi
Soluţie. Dacă S este aria triunghiului ABC, atunci S 2 = S12 + S22 + S32 . Notăm
cu V volumul tetraedrului; cum 3V = S1 · V B = S2 · V C, obţinem că 9V 2 = 2S1 · S2 ·
VB·VC 1 S2 S12 + S22 + S32 S1 S2 S3
= 2S1 S2 S3 . Atunci 2 = = = + + .
2 h 9V 2 2S1 S2 S3 2S2 S3 2S1 S3 2S1 S2
2 2
Inegalitatea dorită rezultă dacă ţinem seama de faptul că 2ab ≤ a + b , ∀a, b ∈ R.

61
Egalitatea se atinge când S1 = S2 = S3 , deci când V A = V B = V C.

Clasa a IX-a
IX.171. Fie (xnï)n≥0Åunãşir ò cu x0 = 0, x1 = 1 şi 2xn + 3xn+2 ≤ 5xn+1 , ∀n ∈ N.
2 n
Arătaţi că xn ≤ 3 · 1 − , ∀n ∈ N.
3
Camelia Dană şi Ileana Didu, Craiova
Soluţie. În relaţia 2xk−2 −5xk−1 +3xk ≤ 0 dăm lui k valorile 2, 3, . . . , n şi sumăm
inegalităţile obţinute; rezultă că 3xn − 2xn−1 − 3x1 + 2x0 ≤ 0, deci 3xn ≤ 2xn−1 + 3.
Inegalitatea dorită se demonstrează
ñ acum uşor prin inducţie matematică, observând
Å ãn−1 ô ï Å ãn ò
2 2 2 2
că xn ≤ xn−1 + 1 ≤ · 3 1 − +1=3 1− .
3 3 3 3
IX.172. Rezolvaţi ı̂n numere naturale ecuaţia

a2 + b2 + c2 = ab + bc + ca + (abc − 1)2 .

Cosmin Manea şi Dragoş Petrică, Piteşti


Soluţie. Ecuaţia este simetrică, deci putem presupune a ≥ b ≥ c. Scriem ecuaţia
sub forma
abc a2
(abc − 4) + (b2 c2 − 2) + b(a − b) + c(b − c) + ac + 1 = 0.
2 2
Dacă bc ≥ 2, evident că a ≥ 2 şi atunci fiecare termen al sumei din stânga este
nenegativ, ultimul fiind chiar strict pozitiv; ecuaţia nu are soluţii ı̂n acest caz. Rămâne
că bc = 1, deci b = c = 1 şi, ı̂n acest caz, ecuaţia este verificată pentru orice a ∈ N∗ .
Soluţiile ecuaţiei sunt tripletele (a, 1, 1); (1, a, 1); (1, 1, a), a ∈ N∗ .
IX.173. a) Fie a, b ∈ R pentru care cos(a + b) 6= ± sin(a − b); arătaţi că

3 cos2 (a + b) − sin2 (a − b)
tg 2a + tg(a + b) + tg 2b = tg(a + b) · .
cos2 (a + b) − sin2 (a − b)

b) Demonstraţi că tg 20◦ − ctg 50◦ + tg 80◦ = 3 3.
Marian Tetiva, Bârlad
Soluţie. a) Se verifică uşor egalitatea cos 2a cos 2b = (cos2 a − sin2 a)(cos2 b −
sin2 b) = cos2 (a + b) − sin2 (a − b). Rezultă că

sin 2(a + b) 2 sin(a + b) cos(a + b)


tg 2a + tg 2b = = =
cos 2a cos 2b cos2 (a + b) − sin2 (a − b)
2 cos2 (a + b)
= tg(a + b) · =
cos2 (a + b) − sin2 (a − b)
ñ ô
3 cos2 (a + b) − sin2 (a − b)
= tg(a + b) · −1 ,
cos2 (a + b) − sin2 (a − b)

ceea ce trebuia demonstrat.

62
b) Luăm a = 10◦ , b = 70◦ ı̂n identitatea de la a); ı̂n membrul stâng vom avea
tg 20◦ + tg 80◦ + tg 140◦ = tg 20◦ − ctg 50◦ + tg 80◦ , iar ı̂n cel drept

3 cos2 80◦ − 3/4 sin 80◦ (4 cos2 80◦ − 1) sin 240◦ √


tg 80◦ · = 3 · = 3 · = 3 3.
cos2 80◦ − 3/4 cos 80◦ (4 cos2 80◦ − 3) cos 240◦

IX.174. Fie ABCD un patrulater convex ı̂n care m(DAC) ’ = 20◦ , m(DCA) ’ =
’ = 30◦ şi m(BCA)
45◦ , m(BAC) ’ = 60◦ . Demonstraţi că diagonalele AC şi BD nu
sunt perpendiculare.
Neculai Roman, Mirceşti (Iaşi)
Soluţie. Notăm cu R1 şi R2 razele cercurilor circumscrise triunghiurilor ADC
şi ABC; din teorema sinusurilor, obţinem că AC = 2R2 = 2R1 sin 115◦, deci R2 =
R1 · sin 115◦ . Dacă, prin absurd, AC ⊥ BD, atunci CO = DC ◦
√ · cos 45 = BC · cos√60

(unde {O} = AC ∩ BD), relaţie √ care devine succesiv: 2√· DC = BC ⇔ 2 ·


2R1 sin 20◦ = 2R2 sin 30◦ ⇔ 2 2R1 sin 20◦ = R1 sin 65◦ ⇔ 2 2 sin 20◦ = sin(45◦ +
1
20◦ ) ⇔ 4 sin 20◦ = cos 20◦ + sin 20◦ ⇔ tg 20◦ = . Ultima relaţie nu este adevărată
3
(tg 20◦ = 0, 36397 . . . ∈ R\Q), deci diagonalele AC şi BD nu sunt perpendiculare.
IX.175. Fie dat un triunghi ABC şi fie O centrul cercului circumscris lui. Să se
calculeze aria coroanei determinată de cercurile cu centrele ı̂n O şi tangente la cercul
ı̂nscris triunghiului dat.
Temistocle Bı̂rsan, Iaşi
Soluţie. Vom utiliza notaţiile uzuale C(O, R) şi C(I, r) pentru cercurile
circumscris şi ı̂nscris triunghiului ABC. Distingem trei A
cazuri: I. O ∈ Int(C(I, r)), II. O ∈ C(I, r) şi III. O ∈
Ext(C(I, r)), care, datorită relaţiei lui Euler
√ OI 2 =
2
R − 2Rr,√ se reformulează astfel:
√ I. R < ( 2 + 1)r, II.
R = ( 2 + 1)r şi III. R = ( 2 + 1)r (ı̂ntr-adevăr,√ O∈
C(I.r) ⇔ OI = r ⇔ R2 − 2Rr = r2 ⇔ R = ( 2 + 1)r). I
Notăm cu K aria coroanei. Avem: O
2 2
I.
√ K = π(OI + r) − π(r − OI) = 4πr · OI =
4πr R2 − 2Rr;
II. K = π(2r)2 − π · 02 = 4πr2 ;
III. K = π(OI 2 2 B C
√ + r) − π(OI − r) = √
2
4πr · OI = 4πr R − 2Rr. Cum ı̂n cazul II avem R = ( 2 + 1)r şi constatăm uşor

că R2 − 2Rr √
= r, urmează că ı̂n toate cazurile aria coroanei este dată de
R=( 2+1)r
formula p
K = 4πr R2 − 2Rr.
Notă. Pentru aria coroanei K ′ determinată
√ de cercurile cu centrul ı̂n I şi tan-
gentele la C(O, R) obţinem formula K ′ = 4πR R2 − 2Rr (ı̂ntr-adevăr, K ′ = π(R +
OI)2 − π(R − OI)2 = 4πR · OI etc.).

63
Clasa a X-a Å ã Å ã
10 5
X.171. Demonstraţi că numărul A = arccos √ : arccos √ este natural.
7 7 2 7
Ionel Tudor, Călugăreni
5 10
Soluţie. Notăm a = arccos √ şi b = arccos √ ; atunci a, b ∈ (0, π), cos a =
2 7 7 7
5 10 5 1 π π
√ şi cos b = √ . Cum √ > = cos , rezultă că a < . În plus, cos 3a =
2 7 7 7 2 7 2 3 3
10
4 cos3 a − 3 cos a = √ = cos b, cu 3a, b ∈ (0, π). Deducem că 3a = b, aşadar
7 7
A = b : a = 3 ∈ N.
X.172. Fie a, x, y, z ∈ (0, 1) sau a, x, y, z ∈ (1, ∞) astfel ı̂ncât loga x + loga y +
loga z + logx a + logy a + logz a = 10. Arătaţi că min{a, a9 } ≤ xyz ≤ max{a, a9 }.
Dan Popescu, Suceava
Soluţie. Dacă α = loga x + loga y + loga z şi β = logx a + logy a + logz a, atunci
aβ ≥ 9 (inegalitatea mediilor M H ≤ M A), de unde α(10 − α) ≥ 9, prin urmare
α ∈ [1, 9]. Rezultă că 1 ≤ loga xyz ≤ 9, adică tocmai cerinţa problemei.
X.173. Punctul M este situat pe ipotenuza BC a triunghiului dreptunghic isoscel
ABC. Pe perpendiculara ı̂n M pe AM se consideră punctul P astfel ı̂ncât AM = M P
şi P se află ı̂n semiplanul determinat de dreapta AM şi punctul C. Punctul Q este
simetricul lui C faţă de M, iar perpendiculara ı̂n Q pe BC intersectează AB ı̂n R.
Demonstraţi că dreapta RP trece printr-un punct fix, indiferent care ar fi poziţia lui
M pe segmentul BC.
Claudiu-Ştefan Popa, Iaşi
Soluţie. Raportăm planul la un reper cartezian având dreapta BC ca axă Ox,
mediatoarea lui BC drept axă Oy şi unitatea celor
două axe egală cu OA. Avem că A(0, 1), B(−1, 0),
C(1, 0) şi M (a, 0), unde a ∈ (−1, 1). Deducem uşor
că P (1 + a, a), Q(2a − 1, 0) şi R(2a − 1, 2a). Ecuaţia
dreptei P R este y(a−2) = ax−3a şi această dreaptă
trece prin punctul fix S(3, 0), care este simetricul lui
B fată de C.
X.174. Numerele complexe nenule a, b, c sunt
astfel ı̂ncât |a| ≤ 1, |b| ≤ 1, |c| ≤ 1 şi |a + b + c| ≤ 1. Arătaţi că |z − a| + |z − b| +
|z − c| + |z + a + b + c| ≥ |a + b|2 + |b + c|2 + |a + c|2 , ∀z ∈ C.
Tidor Pricope, elev, Botoşani
Soluţie. Notăm d = −a − b − c; atunci a + b + c + d = 0 şi |d| ≤ 1. Suma P din
membrul stâng al inegalităţii din enunţ devine |z−a|+|z−b|+|z−c|+|z−d| = |z−a|.
Avem:
X X X X
|z − a| ≥ |a| · |z − a| = |a| · |z − a| = |za − |a|2 | ≥
≥ |z(a + b + c + d) − |a|2 − |b|2 − |c|2 − |d|2 | =
X
= |a|2 = |a + b|2 + |b + c|2 + |c + d|2 ,

64
ultima egalitate rezultând din identitatea lui Hlawka.
X.175. Fie α, β ∈ R astfel ı̂ncât |α + β| ≥ 2. Dacă z1 , z2 ∈ C, |z1 | = |z2 |,
demonstraţi că |z1 + z2 | ≤ |αz1 + βz2 |.
Ovidiu Pop, Satu-Mare
Soluţia 1. Dacă z1 = 0, atunci z2 = 0 şi concluzia
este
adevărată. Dacă z1 6= 0,
z2 z2 z2
atunci z2 6= 0 şi avem de dovedit că 1 + ≤ α + β · . Fie = x + yi, x, y ∈ R
z 1 z 1 z 1
z2
şi x2 + y 2 = = 1; inegalitatea devine 1 + 2x + x2 + y 2 ≤ α2 + 2αβx + β 2 x2 + β 2 y 2 ,
z1
sau α2 + β 2 + 2αβx − 2x − 2 ≥ 0, unde x ∈ [−1, 1] şi |α + β| ≥ 2. Pentru a demonstra
această ultimă inegalitate, considerăm funcţia f : [−1, 1] → R, f (x) = 2(αβ − 1)x +
(α2 + β 2 − 2). Cum f (1) = (α + β)2 − 4 ≥ 0 şi f (−1) = (α − β)2 ≥ 0, rezultă că
f (x) ≥ 0, ∀x ∈ [−1, 1] şi, cu aceasta, soluţia este completă.
Soluţia 2 (Dan Dumitrescu, elev, Rm. Vâlcea). Punctele A(z1 ) şi B(z2 ) se
z1 + z2 
află pe cercul cu centrul ı̂n origine şi de rază egală cu |z1 | = |z2 |. Fie D
2
mijlocul coardei AB. Avem OD ⊥ AB.
Evident,
este suficient
să demonstrăm
inegalitatea pentru α + β ≥ 2. Să o scriem
z1 + z2 αz1 + βz2
ı̂n forma ≤
2 2 .
Å ã
αz1 + βz2
Dacă α+β = 2, atunci punctul M este pe dreapta AB şi inegalitatea
2
revine la inegalitatea geometrică OD ≤ OM, adevărată. Mai mult, avem egalitate
dacă şi numai dacă α = β = 1.
Dacă α + β > 2, atunci punctul M se află de partea dreptei AB ce nu conţine
αz1 + βz2 αz1 + (β − γ)z2 γ
originea, după cum rezultă din egalitatea = + z2 , cu γ >
2 2 2
αz1 + (β − γ)z2
0, şi faptul că ∈ AB. Ca urmare, ı̂n acest caz |z1 + z2 | < |αz1 + βz2 |.
2

Clasa a XI-a
XI.171. Fie a, b, c, d numere complexe astfel ı̂ncât a(b + c) = d(b + c) = 1 şi
b2 + ad = c2 + ad = 0. Demonstraţi că b(a + d) = c(a + d) = 1 şi a2 + bc = d2 + bc = 0.
Lucian Tuţescu şiÅIonuţã Å
Ivănescu,
ã Craiova
Å ã
a b c d 1 0
Soluţie. Condiţiile din enunţ revin la faptul că = .
c d a b 0 1
Å ãÅ ã
c d a b
Atunci cele două matrice din stânga sunt inverse una celeilalte, deci =
a b c d
Å ã
1 0
şi, de aici, rezultă cerinţele problemei.
0 1
XI.172. Fie A, B ∈ M3 (C) două matrice astfel ı̂ncât A2 B + BA2 = AB 2 + B 2 A
şi A − B este inversabilă. Demonstraţi că A şi B nu pot fi simultan inversabile.
Dumitru Crăciun, Fălticeni
Soluţie. Relaţia din enunţ se scrie sub forma A(A − B)B = −B(A − B)A.
Cum matricele sunt de ordin impar, prin trecere la determinanţi obţinem că det A ·

65
det (A − B) · det B = −det B · det (A − B) · det A. Avem că det (A − B) 6= 0, prin
urmare 2det A · det B = 0 şi, de aici, rezultă concluzia problemei.
XI.173. Fie A, B ∈ M2 (R) astfel ı̂ncât A2 +B 2 +I2 = AB. Arătaţi că AB = BA.
Gheorghe Iurea, Iaşi
Soluţie. Deoarece A2 = aA − xI2 , B 2 = bB − yI2 , de unde a = Tr A, x = det A,
b = Tr B, y = det B, egalitatea din enunţ se rescrie sub forma (A − bI2 )(B − aI2 ) =
(ab − x − y + 1)I2 .
Dacă ab − x − y + 1 6= 0, matricele A − bI2 şi B − aI2 sunt inversabile, iar
(B − aI2 )(A − bI2 ) = (ab − x − y + 1)I2 . Desfăcând parantezele, obţinem că BA =
A2 + B 2 + I2 = AB.
Dacă ab − x − y + 1 = 0, atunci (A − bI2 )(B − aI2 ) = O2 , de unde det (A − bI2 ) = 0
sau det(B − aI2 ) = 0. Dacă det (A − BI2 ) 6= 0, rezultă că B − aI2 = O2 , deci
B = aI2 , prin urmare AB = BA; la fel se procedează dacă det (B − aI2 ) 6= 0. Dacă
det (A − bI2 ) = det (B − aI2 ) = 0, obţinem că x − ab + b2 = y − ab + a2 = 0; atunci
condiţia ab − x − y + 1 = 0 devine a2 − ab + b2 + 1 = 0, imposibil pentru a, b ∈ R.
Å În concluzie,
ã AB = BA. Remarcăm că problema are obiect: matricele A = B =
1 1
satisfac ipoteza.
−2 −1
xn
XI.174. Calculaţi derivata de ordin n a funcţiei f : R\{a} → R, f (x) = ,
x−a
unde n ∈ N∗ este dat.
Lucian Tuţescu şi Teodora Rădulescu, Craiova
xn − an + an
Soluţie. Avem că f (x) = = (xn−1 +xn−2 ·a+. . .+x·an−2 +an−1 )+
x−a Å ã(n)
an (n) n 1
. Derivata de ordin n a parantezei este nulă, deci f (x) = a · =
x−a x−a
n n
a · (−1) · n!
.
(x − a)n+1
f (x) f (x)
XI.175. Fie f : R→R o funcţie continuă, astfel ı̂ncât lim = lim =a
x→+∞ x x→−∞ x
şi lim [f (x)− ax] = lim [f (x)− ax] = b, cu a, b ∈ R. Atunci, pentru orice d ∈ R∗+ ,
x→+∞ x→−∞
f (x2 ) − f (x1 )
există x1 , x2 ∈ R astfel ı̂ncât x2 −x1 = d şi = a. (O extindere a teoremei
x2 − x1
de medie a lui Lagrange.)
Temistocle Bı̂rsan, Iaşi
1
Soluţie. Considerăm funcţia g : R → R definită prin g(x) = [f (x+d)−f (x)]−a.
d
Evident, g este continuă.
Dacă există x0 ∈ R astfel ı̂ncât g(x0 ) = 0, luăm x1 = x0 , x2 = x0 + d şi constatăm
f (x2 ) − f (x1 )
uşor că = a.
x2 − x2
Arătăm că un astfel de punct x0 există, raţionând prin reducere la absurd. Să
presupunem, aşadar, că g(x) 6= 0, ∀x ∈ R. Din continuitatea funcţiei g rezultă că
g(x) > 0, ∀x ∈ R sau g(x) < 0, ∀x ∈ R. Să presupunem că suntem ı̂n cazul g(x) > 0,

66
∀x ∈ R; ı̂n al doilea caz se procedează la fel. Prin urmare, avem

f (x + d) − f (x) > ad, ∀x ∈ R,

de unde f (x) < f (x + d) − ad şi f (x + d) > f (x) + ad, ∀x ∈ R. Utilizând ı̂n mod
repetat aceste ultime inegalităţi, obţinem:

f (−nd)+nad < . . . < f (−d)+ad < f (0) < f (d)−ad < f (2d)−2ad < . . . < f (nd)−nad,

de unde, trecând la limită pentru n → ∞, obţinem b < f (0) < b, ceea ce este absurd.
Afirmaţia din enunţ este complet demonstrată.

Notă. În loc să se impună ca lungimea proiecţiei coardei să fie d, se poate cere
ca lungimea coardei să fie d (proiecţia va avea lungimea d| cos α|).

Clasa a XII-a
Z 5
ln2 x ln 2
XII.171. Dacă a ∈ R∗ , arătaţi că 2 2
dx < .
4 x +a 2
Aurel Chiriţă, Slatina
√ ln2 x x
Soluţie. Se arată uşor că ln x < x, ∀x ∈ (4, ∞); atunci 2 2
< 2 . In-
Z 5 x 2+ a x + a2
ln x 1 25 + a2
tegrând această inegalitate pe intervalul [4, 5], obţinem că 2 2
dx < ln
4 x +a 2 16 + a2
2
25 + a
şi este evident că < 2.
16 + a2
XII.172. Fie f : [0, 1] → R o funcţie de două ori derivabilă, cu derivata a doua
continuă. Demonstraţi că
Z 1

lim n(f (1) + f (1) − n(f (1) − n xn f (x)dx)) = f (1) + 3f ′ (1) + f ′′ (1).
n→∞ 0

Z 1Marian Tetiva, Bârlad


1
Soluţie. Integrând de două ori prin părţi, obţinem că xn f (x)dx = f (1)−
Ç å 0 n+1
Z 1
1
f ′ (1) − xn f ′′ (x)dx . Şirul a cărui limită dorim să o calculăm
(n + 1)(n + 2) 0
capătă forma
Z 1
n n(3n + 2) ′ n2
f (1) + f (1) + ·n xn f ′′ (x)dx.
n+1 (n + 1)(n + 2) (n + 1)(n + 2) 0

67
Trecând la limită şi folosind faptul că, dacă h : [0, 1] → R este o funcţie continuă,
Z 1
atunci lim n xn h(x)dx = h(1), obţinem concluzia dorită.
n→∞ 0
Z n 2
1 x + n2
XII.173. Calculaţi lim 3 · −x + 1
dx.
n→∞ n 0 2
D.M. Batineţu-Giurgiu, Bucureşti şi Neculai Stanciu, Buzău
Soluţie. Mai general, vom demonstra că:
Dacă f : [0, ∞) → R şi g : [0, 1] → R sunt funcţii caontinue şi există L =
lim f (x) ∈ R, atunci
x→∞

Z n x Z 1
1
lim · f (x)g dx = L · g(x)dx.
n→∞ n 0 n 0
Z x
1 n
Considerăm funcţia h : [0, ∞) → R, h(x) = f (x)−L; atunci f (x)g dx =
Z n Z n   n 0 n
1  x  L x x
h(x)g dx + g dx. Cu substituţia = t, al doilea termen este
n 0 n n 0 n n
Z 1
egal cu L · g(t)dt. Arătăm acum că primul termen tinde spre 0 când n → ∞.
0
Deoarece g este continuă pe [0, 1], există M > 0 astfel ı̂ncât |g(x)| ≤ M, ∀x ∈ [0, 1].
Dacă H este o primitivă a funcţiei |h|, avem:
Z n  x  Z
1 1 n
0 ≤ lim h(x)g dx ≤ M · lim |h(x)|dx =
n→∞ n 0 n n→∞ n 0
Z n+1
H(n) H(n + 1) − H(n)
= M · lim = M lim =M· |h(x)|dx = 0,
n→∞ n n→∞ (n + 1) − n n

pentru că lim h(x) = 0.


x→∞
1
Particularizând f (x) = , g(x) = 1 + x2 , cum L = lim f (x) = 1 şi
Z 1 2−x
+ 1 x→∞
4 4
g(x)dx = , obţinem că limita cerută ı̂n enunţ este egală cu .
0 3 3
XII.174. Determinaţi polinoamele f = X 3 + aX + b ∈ C[X] care au cel puţin
două rădăcini ı̂ntregi.
Gheorghe Iurea, Iaşi
Soluţie. Dacă x1 , x2 , x3 sunt rădăcinile lui f şi x1 , x2 ∈ Z, cum x1 + x2 + x3 = 0,
rezultă că x3 ∈ Z. Atunci a = x1 x2 + x1 x3 + x2 x3 ∈ Z şi b = −x1 x2 x3 ∈ Z.
Renotăm x1 = n ∈ Z; din f (n) = 0 obţinem că b = −n3 − an, iar f = X 3 + aX −
n − an = (X − n)(X 2 + nX + a + n2 ). Ecuaţia x2 + nx + a + n2 = 0 va avea rădăcinile
3
1
ı̂ntregi, prin urmare ∆ = −4a − 3n2 = p2 , cu p ∈ Z. Deducem că a = − (3n2 + p2 ),
4
unde condiţia necesară şi suficientă pentru ca a sa fie număr ı̂ntreg este ca n şi p să
−n ± p
aibă aceeaşi paritate; atunci x2,3 = ∈ Z.
2

68
1 1
În concluzie, polinoamele căutate sunt cele de forma X 3 − (3n2 + p2 )X + (np2 −
4 4
n3 ), unde p, n ∈ Z sunt numere cu aceeaşi paritate.
XII.175. Arătaţi că, oricare ar fi n ∈ N∗ , există k ∈ N∗ astfel ı̂ncât 125|2k + 3n .
Marian Cucoaneş,
Å ã Mărăşeşti
1
Soluţie. În inelul Z125 , avem: |U (Z125 )| = ϕ(125) = 125 1 − = 100. Vom
5
arăta că ordinul elementului b 2 ∈ U (Z125 ) este 100: divizorii naturali ai lui 100 sunt
1, 2, 4, 5, 10, 20, 25, , 50, 100 şi b
21 = b2 6= b1, b
22 = b4 6= b1, b24 = c 16 6= b1, b 25 = c 32 6= b1,
b10 c b b 20 c b b 25 c b b50
2 = 24 6= 1, 2 = 76 6= 1, 2 = 57 6= 1, 2 = 124 6= 1, 2 d b b 100
= 1. Rezultă că b
b 2 este
generator al grupului multiplicativ (U (Z125 ), ·), deci U (Z125 ) = {b 2, b
22 , . . . , b
2100 }.
Fie n ∈ N ; cum (3, 5 ) = 1, deducem că (−3 , 5 ) = 1, prin urmare −b
∗ 3 n 3
3n ∈
b k bn
U (Z125 ) şi atunci există k ∈ {1, 2, . . . , 100} astfel ı̂ncât 2 = −3 . De aici, urmează
concluzia problemei.

Soluţiile problemelor pentru pregătirea


concursurilor propuse ı̂n nr. 2/2016
A. Nivel gimnazial
Å ãÅ ãÅ ã
1 1 1 1
G306. Arătaţi că ecuaţia 1 + 1+ 1+ = 1+ are soluţii ı̂n
x y z 2016
N × N × N.
Gheorghe Iurea, Iaşi
1 a+1 3a + 3 3a + 3 3a + 2
Soluţie. Notăm a = 2016; atunci 1 + = = = · ·
Å ãÅ ã Å 2016 ã a 3a 3a + 2 3a + 1
3a + 1 1 1 1
= 1+ 1+ 1+ , deci putem considera x = 3a, y =
3a 3a + 2 3a + 1 3a
3a + 1, z = 3a + 2.
G307. Determinaţi numerele naturale n pentru care a = 2n + 4n + 8n + 107n este
pătrat pefect.
Ioan Viorel Codreanu, Satulung (Maramureş)
Soluţie. Dacă n = 0, atunci a = 22 ; dacă n = 1, atunci a = 112 . Vom arăta că
n = 0 şi n = 1 sunt singurele soluţii.
Dacă n ≥ 3 este impar, atunci a = M4 + M4 + M4 + (M4 + 3) = M4 + 3, număr
care nu poate fi pătrat perfect. Dacă n ≥ 2 este par, n = 2k, k ∈ N∗ , arătăm că
(107k )2 < a < (107k + 1)2 . Inegalitatea din stânga este evidentă, iar cea din dreapta
revine la 22k + 42k + 82k < 2 · 107k + 1. Avem: 2 · 107k + 1 > 84k = (64 + 16 + 4)k >
64k + 16k + 4k = 82k + 42k + 22k . Cum a este cuprins ı̂ntre două pătrate perfecte
consecutive, rezultă că a nu este pătrat perfect.
G308. Scrieţi numărul 30643064 ca sumă de cuburi perfecte cu număr minim de
termeni.
Ioan Viorel Codreanu, Satulung (Maramureş)
Soluţie. Avem: 30643064 = 30643063 · 3064 = 30643063(103 + 103 + 103 + 43 ) =
(30641021 ·10)3 +(30641021 ·10)3 +(30641021 ·10)3 +(30641021 ·4)3 . Dovedim că nu există

69
scrieri cu mai puţin de patru termeni. Un cub perfect dă, la ı̂mpărţirea prin 9, unul
dintre resturile 0, 1 sau 8. Adunând cel mult trei cuburi perfecte, suma nu poate da
rest 4 la ı̂mpărţirea cu 9. Pe de altă parte, 30643064 ≡ 43064 ≡ 4 · (43 )1021 ≡ 4(mod 9).
Aşadar, numărul minim de termeni din sumă este 4, iar o scriere cu patru termeni a
fost găsită.
G309. Rezolvaţi ı̂n numere naturale ecuaţia 3a + 4b = 5c .
Andrei George Turcu, elev, Craiova
Soluţie. Fie (a, b, c) soluţie cu a 6= 0; reducând ecuaţia modulo 3, obţinem că
(−1)c = 1, deci c = 2x, x ∈ N. Ecuaţia devine (5x − 2b )(5x + 2b ) = 3a şi, de aici,
5x − 2b = 3α , 5x + 2b = 3β , α < β, α + β = a. Prin adunare, 2 · 5x = 3α (1 + 3β−α ),
adică α = 0 şi atunci 5x − 2b = 1, 5x + 2b = 3a . Scăzând aceste egalităţi deducem că
2b+1 = 3a − 1. Dacă b = 0, am avea a = 1, fals. Dacă b ≥ 1, reducând ecuaţia modulo
4, rezultă că a = 2y. Ultima relaţie devine 32y − 1 = 2b+1 ⇔ (3y − 1)(3y + 1) = 2b+1 ,
de unde 3y − 1 = 2m , 3y + 1 = 2n , cu m < n, m + n = b + 1. Scăzând, deducem că
2n − 2m = 2 ⇔ 2m (2n−m − 1) = 2, prin urmare m = 1, n = 2, y = 1. Obţinem soluţia
a = b = c = 2.
Dacă a = 0, ecuaţia devine 1 + 4b = 5c , deci c 6= 0. Dacă c = 1, atunci b = 1. Dacă
c ≥ 2, atunci (5 − 1)(5c−1 + . . . + 5 + 1) = 4b , aşadar 5c−1 + 5c−2 + . . . + 5 + 1 = 4b−1 .
Pentru c impar, obţinem imediat o contradicţie. Dacă c este par, c = 2z, z ∈ N∗ ,
ecuaţia devine 1 + 4b = 52z ⇔ (5z − 1)(5z + 1) = 4b . Ca mai sus, 5z − 1 = 2u ,
5z + 1 = 2v , cu u < v, u + v = 2b; prin scădere, 2 = 2v − 2u , de unde v = 2, u = 1 şi
2b = 3, imposibil.
În concluzie, soluţiile ecuaţiei sunt (2, 2, 2) şi (0, 1, 1).
G310. Dacă pătratul numărului natural nenul x se poate scrie sub forma x2 =
3y + 3y + 1, y ∈ N∗ , atunci x este sumă de două pătrate perfecte consecutive.
2

Marius Drăgan, Bucureşti şi Neculai Stanciu, Buzău


Soluţie. Relaţia din enunţ se poate scrie sub forma 3(2x − 1)(2x + 1) = (6y + 3)2 .
Numerele 2x − 1 şi 2x + 1 fiind relativ prime, rezultă că există m, n ∈ N pentru care
(i) 2x + 1 = m2 , 2x − 1 = 3n2 sau (ii) 2x + 1 = 3m2 , 2x − 1 = n2 . Cazul (i) conduce
la m2 − 3n2 = 2, egalitate imposibilă modulo 3; rămân adevărate (ii), prin urmare
Å ã Å ã
n2 + 1 n2 + 1 n−1 2 n+1 2
x= , cu n impar (pentru că x ∈ N). Deoarece = + ,
2 2 2 2
rezultă concluzia problemei.
Observăm că problema are obiect: de exemplu, x = 13 are proprietatea din enunţ
(luând y = 7).
G311. Fie x, y, z numere reale pozitive astfel ı̂ncât xyz = x+y+z+2. Demonstraţi
că
x+1 y+1 z+1 √
3+ + + ≥ 3 3 xyz.
y+1 z+1 x+1
Marian Tetiva, Bârlad
1 1 1
Soluţie. Condiţia din ipoteză poate fi scrisă sub forma + + = 1,
x+1 y+1 z+1
1 1 z x+1 z(x + 1)
de unde + = , prin urmare 1 + = . Două relaţii
x+1 y+1 z+1 y+1 z+1

70
similare se mai pot obţine; adunându-le pe toate trei şi aplicând inegalitatea mediilor
pentru membrul drept, rezultă chiar inegalitatea cerută.
G312. Dacă a, b, c sunt lungimile laturilor unui triunghi, demonstraţi că

a+b−c (b + c − a)(c + a − b)
≥ .
3ab − bc − ac abc
Răzvan Morariu, elev, Iaşi
Soluţie. Scriem inegalitatea din enunţ sub forma

(a + b − c)2 3ab − bc − ac
≥ . (∗)
(a + b − c)(b + c − a)(c + a − b) abc

Avem: (a + b − c)2 = (a2 + b2 + c2 − ab − bc − ca) + (3ab − bc − ca) ≥ 3ab − bc − ca,


Q
deoarece prima paranteză este nenegativă. Vom mai arăta că (a + b − c) ≤ abc şi
atunci (∗) este adevărată. Cu notaţiile a = x + y, b = x + z, c = y + z,
Y √ √ √ Y
(a + b − c) = 8xyz = 2 xy · 2 yz · 2 zx ≤ (x + y) = abc

şi, cu aceasta, soluţia este completă. Egalitatea are loc dacă şi numai dacă a = b = c.
G313. Punctele L şi K aparţin ipotenuzei (BC) a triunghiului dreptunghic isos-
cel ABC astfel ı̂ncât L ∈ (BK). Demonstraţi că segmentele (LK) şi (BL), (KC)
pot să fie ipotenuza, respectiv catetele unui triunghi dreptunghic dacă şi numai dacă
’ = 45◦ .
m(LAK)
Claudiu-Ştefan Popa şi Doru Buzac, Iaşi
Soluţia 1. Dacă m(∡LAK) = 45◦ , luăm ı̂n considerare △ABK şi △LCA.
A
Deoarece m(∡ABK) = m(∡LCA) = 45◦

şi m(∡BKA) = m(∡CAL) = 45 +
m(∡CAK), rezultă că cele două triunghiuri
AB BK
sunt asemenea, de unde = ⇒
CL AC
2
AB · AC = CL · BK ⇒ AB = CL · BK.
În △ABC avem relaţiile: 2 · AB 2 = BC 2
şi BC = BL + LK + KC, care conduc la
2 · CL · BK = (BL + LK + KC)2 ⇔ 2(CK + B
L K C
LK) · (BL + LK) = (BL + LK + KC)2 , de
unde, după efectuarea calculelor, se obţine
LK 2 = BL2 + KC 2 . Reciproc, din relaţiile D
LK 2 = BL2 + KC 2 şi 2 · AB 2 = (BL + LK + KC)2 , rezultă AB 2 = CL · BK ⇔
AB BK
AB · AC = CL · BK, deci = şi, cum m(∡ABK) = m(∡LCA) = 45◦ ,
CL AC
rezultă △ABK ∼ △LCA, aşadar m(∡AKB) = m(∡LAC). Dar m(∡AKB) =
45◦ + m(∡CAK) iar m(∡LAC) = m(∡LAK) + m(∡CAK), de unde rezultă că
m(∡LAK) = 45◦ .
Soluţia 2. Dacă m(∡LAK) = 45◦ considerăm segmentul [AD], simetricul seg-
mentului [AB] faţă de dreapta AL. Deoarece m(∡BAL) + m(∡CAK) = 45◦ , atunci

71
m(∡BAL) < 45◦ , m(∡CAK) < 45◦ , deci m(∡BAL) < m(∡LAK) şi, cum m(∡BAL)
= m(∡DAL), atunci m(∡DAL) < m(∡LAK), de unde (AD ⊂ int (∡LAK). Pentru
că m(∡LAK) = 45◦ , rezultă că 2[m(∡DAL) + m(∡DAK)] = 90◦ , deci m(∡DAK) =
m(∡CAK), adică [AD] este şi simetricul lui [AC] faţă de dreapta AK. Observăm
acum că △ABL ≡ △ADL şi △ACK ≡ △ADK, de unde [DL] şi [BL] sunt congru-
ente şi [DK] şi [CK] sunt, de asemenea, congruente iar m(∡LDK) = m(∡LDA) +
m(∡KDA) = m(∡ABL) + m(∡ACK) = 90◦ .
Reciproc, presupunem că LK 2 = BL2 + CK 2 , dar m(∡LAK) 6= 45◦ . Din LK 2 =
BL2 + CK 2 , rezultă că LK > BL, LK > CK. Dacă m(∡CAL) ≤ 45◦ , atunci LK <
BC
≤ BL, contradicţie cu LK > BL. Prin urmare m(∡CAL) > 45◦ şi există M ∈
2
(LC)\{K} astfel ı̂ncât m(∡LAM ) = 45◦ şi, deci, are loc relaţia LM 2 = BL2 + M C 2 ,
de unde BL2 = LM 2 −M C 2 = LK 2 −CK 2 . Se obţine LM 2 +CK 2 = LK 2 +M C 2 (∗).
Dacă M ∈ (LK), atunci LK 2 +M C 2 = (LM +M K)2 +(M K +CK)2 > LM 2 +CK 2 .
Dacă M ∈ (KC), atunci LM 2 +CK 2 = (LK +KM )2 +(CM +KM )2 > LK 2 +CM 2 .
Ambele cazuri sunt ı̂n contradicţie cu (*). Rămâne adevărat că m(∡LAK) = 45◦ .
Notă. Cu unele mici modificări la Soluţia 1, se poate arăta că rămâne adevărată
concluzia şi dacă unul dintre punctele L, K aparţine lui (BC) şi celălalt este situat
pe prelungirea ipotenuzei, cu ordinea L, B, K şi C ori B, L, C şi K.
G314. În rombul ABCD, punctele E şi F se află pe laturile AB şi BC astfel
ı̂ncât ∡ECF = ∡ABD. Să se demonstreze că (CD + DF )(CB + BE) = BD2 .
Titu Zvonaru, Comăneşti
Soluţie. Considerăm latura rombului egală cu 1. Notăm cu 2a = BD, x = EB,
CE CF 2a
y = DF. Din asemănări, obţinem uşor că CP = , CQ = , BQ = ,
x+1 y+1 y+1
2a
PD = . Deoarece ∡P CF = ∡P DF , ∡ECQ = ∡QBE, patrulaterele DCP F şi
x+1
BCQE sunt inscriptibile. Din aceste patrulatere şi din faptul că BD este mediatoarea
diagonalei AC, deducem că F P = P C = P A, EQ = QC = QA.
Folosind teorema lui Ptolemeu, avem:

y · CE CE 2a · CF CE 2a
DF · CP + F P · DC = DP · CF ⇔ + = ⇒ = ;
x+1 x+1 x+1 CF y+1
x · CF CF 2a · CE CF 2a
BE · QC + BC · QE = CE · BQ ⇔ + = ⇒ = .
y+1 y+1 y+1 CE x+1

De aici, obţinem că (x + 1)(y + 1) = 4a2 ⇔ (CD + DF )(CB + BE) = BD2 .


G315. Se consideră drepunghiul ABCD cu AB = 35, BC = 14 şi punctul M
pe latura AB astfel ı̂ncât AM =21. Despre un punct X de pe laturile dreptunghiului
(X6=C) vom spune că este legat de M dacă AC, perpendiculara ı̂n M pe CM şi
perpendiculara ı̂n X pe CX sunt trei drepte concurente. Determinaţi punctele legate
de M .
Gabriel Popa, Iaşi
Soluţie. Notăm cu S intersecţia dreptei AC cu perpendiculara ı̂n M pe CM .

72
Un punct X este legat de M dacă şi numai dacă CX ⊥
SX, deci dacă şi numai dacă X aparţine cercului de di-
ametru CS.
Dacă O este mijlocul lui CS, triunghiurile BOM
şi BOC sunt congruente (L.L.L.), prin urmare OE =
d(O, AB) = d(O, BC) = OF. Deducem că OEBF este pătrat, cu OF = BF = a.
CF OF 14 − a a
Din △COF ∼ △CAB rezultă că = , adică = , de unde a = 10.
CB AB √ 14 35 √
2
Apoi, raza cercului de diametru CS este CO = CF + OF = 2 29. Avem: 2

d(O, AB) = d(O, BC) < r, d(O, CD) < r iar d(O, DA) > r. Cercul de diametru CS
va tăia AB ı̂n două puncte (M şi N ), BC ı̂n două puncte (P şi C), CD ı̂n două
puncte (Q şi C) şi nu va intersecta AD.
În concluzie, există trei puncte legate de M , anume N, P şi Q. Se obţine imediat
că BN = BP = 6, iar CQ = 20.

B. Nivel liceal
Notă. Domnul Vasile Jiglău observă că soluţia problemei L289 publicată ı̂n
RecMat 1/2016 este una parţială, fiind valabilă doar pentru cazul particular ı̂n care
există două triunghiuri consecutive din şirul definit care sunt congruente. Republicăm
această problemă, ı̂nsoţită de soluţia autorului.

L289. Construim şirul de triunghiuri An Bn Cn , n ∈ N, astfel: △A0 B0 C0 este


arbitrar ales; vârfurile △Ak+1 Bk+1 Ck+1 sunt punctele ı̂n care medianele △Ak Bk Ck
intersectează cercul circumscris acestuia, oricare ar fi k ∈ N. Dacă ı̂n şirul astfel
definit există două triunghiuri congruente, arătaţi că △A0 B0 C0 este echilateral.
Vasile Jiglău, Arad
Soluţie. Demonstrăm ı̂ntâi următoarea
Lemă. Fie ABC un triunghi oarecare A′ , B ′ , C ′ punctele ı̂n care medianele ce por-
nesc din A, B, C intersectează cercul circumscris al acestuia. Notăm cu ω, ω ′ unghiu-
rile Brocard ale triunghiurilor ABC, respectiv A′ B ′ C ′ . Atunci ω ′ ≥ ω, cu egalitate
când triunghiul ABC este echilateral.
Demonstraţie. Fie M mijlocul segmentului BC; cu notaţiile uzuale, avem
÷ S
sin BAM = . În triunghiul BAA′ aplicăm teorema sinusurilor şi rezultă BA′ =
cma
S abc bc ac
2R ⇒ BA′ = = şi, asemănător, CA′ = . Din prima teoremă a
cma 2cma 2ma 2ma
lui Ptolemeu aplicată ı̂n patrulaterul ABA′ C obţinem:

ac ab b 2 + c2
a · AA′ = c +b ⇒ AA′ = .
2ma 2ma 2mb

Din prima teoremă a lui Ptolemeu aplicată ı̂n patrulaterul ABA′ B ′ rezultă că AA′ ·

73
BB ′ = A′ B ′ · c + AB ′ · BA′ . De aici,
Å ã
1 b 2 + c2 a2 + c2 ab ba (a2 + c2 )(b2 + c2 ) − a2 b2
A′ B ′ = · − · = =
c 2ma 2mb 2ma 2mb 2cma mb
P
c( a2 ) 4ma mb mc
= ⇒ A′ B ′ · P 2 = 2cmc ⇒ A′ B ′ 2 · k = c2 (2a2 + 2b2 − c2 ),
2ma mb ( a )
Å ã
4ma mb mc 2
unde k = P 2 . Scriind expresiile analoage pentru B ′ C ′ şi A′ C ′ şi ı̂nsumân-
aP P P 2 P ′ ′2 P 2 2
du-le, obţinem: k A′ B ′ 2 = 4 a2 cP 2
− a4 şi kP A B ·P B′C ′ 2 = P (2a c +
2b c − c )(2a b + 2a c − a ) = 8 a b c + 5 a b − 2 a b − 2 a6 b2 =
2 2 4 2 2 2 2 4 2 2 4 4 4 2 6
P P 4 4 P 2 2 4 P 2 6 P 6 2 P P
10 a2 b2 c4 +5 P 2 a2 b2 −2 Pa b2 c2 −2 P 4 a b −2 P 2 a2 b P = 5( a2 b2 )2P −2 a2 b2 (a4 +
4 4 2 2 4
b + c ) = 5( a b ) − 2( a b )( a ) = ( a b )[5( a b ) − 2( a )].
Utilizăm următoarea formulă pentru cosinusul unghiului lui Brocard: cos ω =
P
a2
pP (cf. T. Lalescu - Geometria triunghiului). Obţinem
2 a2 b 2
P P
′ 4 a2 b 2 − a4
cos ω = p P P P .
2 ( a2 b2 )(5 a2 b2 − 2 a4 )
Să demonstrăm că cos ω ′ ≤ cos ω. Aceasta este echivalentă cu
P P P 2
4 a2 b 2 − a4 a
pP P 2 2 P 4 ≤ pP 2 2 ⇔
2 2
2 ( a b )(5 a b − 2 a ) 2 a b
X X X q X X
⇔4 a2 b 2 − a4 ≤ ( a2 ) (5 a2 b 2 − 2 a4 ).
P P P P P
Deoarece ( a2 )2 = ( a4 ) + 2( a2 b2 ) şi 2 a2 b2 − a4 = 16S 2 este strict mai
mare decât zero, prin ridicare la pătrat se păstrează echivalenţa, astfel că după ridicare
la pătrat ultima egalitate devine echivalentă cu:
X X X X
16( a2 b2 )2 + ( a4 ) − 8( a2 b2 )( a4 ) ≤
X X X X
≤ [( a4 )2 + 2( a2 b2 )][5( a2 b2 ) − 2( a4 )] ⇔
X X X X
⇔ 16( a2 b2 )2 + ( a4 )2 − 8( a2 b2 )( a4 ) ≤
X X X X X X
≤ 5( a2 b2 )( a4 ) − 2( a4 )2 + 10( a2 b2 )2 − 4( a2 b2 )( a4 ) ⇔
X X X X
⇔ 3( a4 )2 − 9( a4 )( a2 b2 ) + 6( a2 b2 )2 ≤ 0 ⇔
X X X X
⇔ 3[( a4 ) − ( a2 b2 )][( a4 ) − 2( a2 b2 )] ≤ 0,
P P P P
care este adevărată deoarece Pa4 ≥ Pa2 b2 şi 2 a2 b2 ≥ a4 (formula lui Heron).
Egalitatea se realizează când a4 = a2 b2 , deci când a = b = c, adică atunci când
ABC este echilateral.
π
În consecinţă, cos ω ′ ≤ cos ω. Cum ω, ω ′ ∈ [0, ] şi cum ı̂n acest interval funcţia
6
cos este descrescătoare, rezultă că ω ′ ≥ ω.

74
Din formulele A′ B ′ ·k = cmc şi analoagele rezultă că, dacă A′ B ′ C ′ este echilateral,
atunci şi ABC este echilateral.
Revenim acum la enunţul problemei. Notăm cu ωk unghiul lui Brocard al tri-
unghiului Ak Bk Ck din şirul construit conform enunţului. Din Lemă, ca şi din modul
ı̂n care a fost definit şirul de triunghiuri, rezultă că ω0 ≤ ω1 ≤ . . . ≤ ωk ≤ ωk+1 ≤ . . .,
deci că şirul (ωk )k∈N este monoton crescător. Presupunem că există m, n ∈ N, m strict
mai mare decât n, astfel ı̂ncât triunghiurile Am Bm Cm şi An Bn Cn să fie congruente.
Rezultă că ωn = ωm , prin urmare ωn = ωn+1 = . . . = ωm−1 = ωm . Conform punctu-
lui b) al lemei, rezultă că triunghiurile An Bn Cn , An+1 Bn+1 Cn+1 , . . . , Am Bm Cm sunt
echilaterale.
Avem: An Bn Cn echilateral ⇒ An−1 Bn−1 Cn−1 echilateral ⇒ An−2 Bn−2 Cn−2
echilateral ⇒ . . .. După n paşi, obţinem A0 B0 C0 este echilateral, adică chiar concluzia
problemei.

Notă. În nr. 2/2016, p. 167, este dată o soluţie greşită la problema L297.
Republicăm această problemă, ı̂nsoţită de o soluţie corectă.
L297. În triunghiul ABC notăm cu La , Lb , Lc picioarele bisectoarelor şi cu Sa , Sb ,
Sc picioarele simedianelor (La , Sa ∈ BC etc.). Ce condiţie trebuie să ı̂ndeplinească
triunghiul ABC pentru ca triunghiurile La Lb Lc şi Sa Sb Sc să fie ı̂nscrise ı̂n acelaşi
cerc?
Temistocle Bı̂rsan, Iaşi
Soluţie. Presupunem că △La Lb Lc şi △Sa Sb Sc se pot ı̂nscrie ı̂n acelaşi cerc.
Puterile punctelor B şi C faţă de acest cerc conduc la sistemul: BLa ·BSa = BLc ·BSc
şi CLa · CSa = CLb · CSB . Utilizând formulele pentru segmentele determinate pe
ac ac2
laturi de bisectoare şi simediane (BLa = , BSa = 2 etc.), obţinem, după
b+c b + c2
calcule, următorul sistem: (a−c)[ac(a+c)+abc−b3 ] = 0, (a−b)[ab(a+b)+abc−c3 ] = 0.
Combinând, avem de considerat patru cazuri, care conduc la două√soluţii posibile ale
problemei: △ABC este echilateral sau este isoscel de forma (l, l, ( 2 − 1)l).
Ambele soluţii sunt valabile. Dacă △ABC este echilateral, afirmaţia este evidentă:
picioarele bisectoarelor şi simedianelor coincid două câte două şi se află pe cercul
R
C(O, ).
2
În cazul triunghiului isoscel, putem proceda astfel: calculăm lungimile laturilor
△La Lb Lc (cu teorema cosinusului), calculăm raza cercului circumscris (La , Lb , Lc )
l » √
(cu formula 4RS = abc) şi verificăm că pe cercul C(O′ , √ 2(2 2 − 1) se află şi
4 7
picioarele simedianelor, unde centrul O′ este punctul de pe mediatoarea bazei tri-
unghiului la distanţă egală cu raza faţă de bază.

L306. Fie triunghiul ABC ı̂nscris ı̂n cercul C, C1 un cerc tangent cercului C ı̂n
N1 şi laturii BC ı̂n M1 , C2 un cerc tangent cercului C ı̂n N2 şi laturii BC ı̂n M2 şi
A1 mijlocul segmentului M1 M2 . Arătaţi că AA1 este axa radicală a cercurilor C1 şi
÷1 ≡ A
C2 dacă şi numai dacă BAA ÷ 1 AC.
Neculai Roman, profesor, Mirceşti (Iaşi)

75
Soluţie. Fie O, O1 şi O2 centrele cercurilor C, C1 şi C2 , {M } = M1 N1 ∩ M2 N2 ,
{P } = M1 N1 ∩ C şi {Q} = M2 N2 ∩ C. Din A N2
triunghiurile isoscele O1 M1 N1 şi OP N1 rezultă
∠O1 M1 N1 ≡ ∠OP N1 , deci OP kO1 M1 şi, prin
N O2
urmare, OP ⊥ BC. Analog, OQ ⊥ BC. De- 1 O1
ducem că P şi Q coincid cu M , mijlocul arcului
1 O
BC. Acum avem: m(∠M M1 M2 ) = [m(M ¯ C) +
2 B M2 C
M1 A1
m(BN¯ 1 )] = 1 [m(M ¯ B)+m(BN ¯ 1 )] = 1 m(M ¯ N 1) =
2 2
m(∠M N2 N1 ), deci △M M1 M2 ∼ △M N2 N1 , de
M M1 M M2
unde = ⇔ M M 1 · M N1 = M M 2 ·
M N2 M N1
M N2 ; obţinem că M este pe axa radicală a cer- M
curilor C1 şi C2 .
Să presupunem că AA1 este axa radicală a cercurilor C1 şi C2 şi să arătăm că
∠BAA1 ≡ ∠A1 AC. Din M ∈ AA1 şi ∠BAM ≡ ∠M AC rezultă ∠BAA1 ≡ ∠A1 AC,
ceea ce era de demonstrat.
Reciproc, să presupunem că ∠BAA1 ≡ ∠A1 AC şi să arătăm că AA1 este axa
radicală a cercurilor C1 şi C2 . Din ∠BAA1 ≡ ∠A1 AC şi ∠BAM ≡ ∠M AC, rezultă
că M ∈ AA1 . Apoi, cum M este pe axa radicală a cercurilor C1 şi C2 şi din A1 M1 =
A1 M2 , rezultă că A1 este pe axa radicală a cercurilor C1 şi C2 ; obţinem că AA1 este
axa radicală a cercurilor C1 şi C2 .
L307. Se consideră patrulaterul convex ABCD şi punctele M ∈ (AB) şi N ∈
(CD). Intersecţiile diagonalelor pentru fiecare dintre patrulaterele ABCD, AM N D
şi BCN M se notează cu O, O1 respectiv O2 . Demonstraţi că punctele O1 , O şi O2
sunt coliniare.
Claudiu-Ştefan Popa, Iaşi
Notă. Ioan Pop, Iaşi, Marius Olteanu, Rm. Vâlcea şi elevul Dan Du-
mitrescu, Rm. Valcea observă că problema este, de fapt, o reformulare a teoremei
lui Pappus; pentru demonstraţia acesteia se poate consulta, de exemplu,
www.mathpages.com/home/kmath542/kmath542.htm
L308. Fie ABC un triunghi dreptunghic ı̂n A şi D proiecţia lui A pe BC. Notăm
cu I1 centrul cercului ı̂nscris ı̂n triunghiul ABD şi cu I2 centrul cercului ı̂nscris ı̂n
triunghiul ADC. Să se demonstreze că raza cercului circumscris triunghiului AI1 I2
este egală cu raza cercului ı̂nscris ı̂n triunghiul ABC.
Titu Zvonaru, Comăneşti
Soluţie (dată de Corneliu Mănescu-Avram, Nicuşor Zlota, precum şi de
autorul problemei). Notăm cu r, r1 , r2 razele cercurilor ı̂nscrise ı̂n triunghiurile ABC,
ABD respectiv ADC. Fie R1 raza cercului circumscris triunghiului AI1 I2 . Este uşor
de observat că ∠I1 AI2 = 45◦ , ∠I1 DI2 = 90◦ .
r1 c cr
Deoarece triunghiurile ABD şi ABC sunt asemenea, avem că = ⇒ r1 =
r a a
br √ √
şi, analog, r2 = . Deoarece DI1 = r1 2, DI2 = r2 2, cu teorema lui Pitagora ı̂n
a

76
triunghiul dreptunghic DI1 I2 obţinem:
 
p c2 r 2 b2 r 2 √
I1 I2 = I1 D2 + I2 D2 = 2 2
+ 2 2
= r 2.
a a
Aplicând acum teorema sinusurilor ı̂n triunghiul AI1 I2 , rezultă:

I1 I2 r 2
R1 = = = r.
2 sin ∠I1 AI2 2 sin 45◦
Notă. Domnii Neculai Roman şi Marius Olteanu remarcă faptul că pro-
blema este un caz particular al L286 din RecMat 2/2015 (autor Neculai Roman).
Menţionăm că problema L308 a fost trimisă revistei din anul 2014, deci responsabi-
litatea publicării sale aparţine redacţiei.
L309. Fie ABC un triunghi ascuţitunghic, a, b, c lungimile laturilor sale, G cen-
trul de greutate iar X, Y, Z puncte arbitrare pe dreptele BC, CA respectiv AB. Arătaţi
că
AX 2 + GX 2 BY 2 + GY 2 CZ 2 + GZ 2 5
2
+ 2
+ ≥ .
a b c2 2
D.M. Bătineţu-Giurgiu, Bucureşti şi Neculai Stanciu, Buzău
Soluţie (dată de Neculai Roman, precum şi de autorii problemei). Notăm cu
ha , hb , hc ı̂nălţimile triunghiului şi cu da , db , dc distanţele de la G la dreptele BC,
ha
CA respectiv AB; evident că da = ≤ GX, ha ≤ AX şi analoagele. Din a =
3
ha 1
ha (ctg B + ctg C) obţinem că = , prin urmare
a ctg B + ctg C

X AX 2 + GX 2 X h2a + h2a Å ã2 X
9 10 ha 1
≥ = = .
a2 a 9 a (ctg A + ctg B)2
Å ã
P P 1 9
Ţinând cont de inegalitatea ( xy) · 2
≥ , ∀x, y, z > 0 (Olimpiada de
(x + y) 4
Matematică,
P Iran, 1996) unde luăm x = ctg A, y = ctg B, z = ctg C şi folosind faptul
că ctg A ctg B = 1, deducem că
X 1 9
2
≥ ,
(ctg A + ctg B) 4

de unde cerinţa problemei. Egalitatea are loc atunci când △ABC este echilateral iar
X, Y, Z sunt mijloacele laturilor sale.
Notă. În enunţul problemei, publicat ı̂n RecMat 1/2016, menţiunea că △ABC
este ascuţitunghic lipseşte. Domnul Marius Olteanu construieşte efectiv un triunghi
obtuzunghic pentru care inegalitatea din enunţ este falsă.
L310. Pentru n ∈ N, n ≥ 2, arătaţi că sunt adevărate inegalităţile:
3n + 5 1 P n 1 3n + 6
a) < · < ;
6 n k=1 ln(1 + k1 ) 6

77
2n + 3 1 1 P
n 1 2n + 4 1
b) + < · < + .
6 9n + 9 n(n − 1) k=1 ln2 (1 + k1 ) 6 4n + 4
(Enunţ corectat)
Mihàly Bencze, Braşov
Soluţie. Demonstrăm că este adevărată dubla egalitate
Å ã 1 Å ã 1
1 n+ 3 1 n+ 2
(∗) 1+ <e< 1+ , ∀n ∈ N∗ .
n n
1
Considerăm funcţiile f, g, h : (0, ∞) → R, f (x) = ex , g(x) = 1 + x · x şi h(x) =
1− 2
1 2
1+x· x . Pentru x ı̂ntr-o vecinătate a lui x0 = , dezvoltând ı̂n serie de
1− 3 2n
Å +1 ã
P∞ xn P∞ xn 1 1
puteri, putem scrie că f (x) = şi g(x) = 1 + x n
. Cum < n−1 ,
n=0 n! n=0 2 n! 2
2 1
∀n ∈ N, n ≥ 2, obţinem că f (x) < g(x), prin urmare f (x0 ) < g(x0 ) ⇔ e 2n+1 < 1+ ,
n
adică a doua parte a inegalităţii (∗). Pentru prima parte, se procedează asemănător,
folosind funcţiile f şi h.
1 1 1
a) Din (*) rezultă k + < 1 < k + , ∀k ∈ N∗ . Sumând după k ∈
3 ln(1 + k ) 2
n(3n + 5) Pn 1 n(n + 2)
{1, 2, . . . , n}, deducem că < 1 < , de unde concluzia
6 k=1 ln(1 + k ) 2
dorită.
2k 1 1 1
b) Din (*) rezultă k 2 + + < 2 1
< k 2 + k + , ∀k ∈ N∗ . Dând valori
3 9 ln (1 + k ) 2
lui k şi sumând, obţinem cerinţa.
Notă (Marius Olteanu, Râmnicu Vâlcea). În enunţul iniţial, publicat ı̂n Rec-
Mat 2/2016,
" cerinţele problemei
# ï erau:ò
Xn
1 1 3n + 5
a) · 1 = , ∀n ∈ N∗ ;
n ln(1 + k ) 6
" k=1
n
# ï ò
1 X 1 2n + 3 1
b) · = + , ∀n ∈ N, n ≥ 2.
n(n − 1)
k=1
ln2 (1 + k1 ) 6 9n + 9
Egalitatea de la punctul a) este adevărată, ı̂n timp ce egalitatea de la b) nu are
ı̂ntotdeauna loc!
n+1 n+1 1
Într-adevăr, din cele de mai sus rezultă că <A< + , ∀n ∈ N∗ , unde
2 2 2
1 P n 1 ∗ 1
A= · . Dacă n = 2p, p ∈ N , atunci p + < A < p + 1, deci [A] = p
n k=1 ln(1 + k1 ) 2
ï ò ï ò ï ò
3n + 5 5 3n + 5
şi = p + , prin urmare [A] = . Dacă n = 2p + 1, p ∈ N∗ , se
6 6 ï ò 6
3n + 5
obţine analog că [A] = = p + 1.
6
1 Pn 1
Notând B = · , n ≥ 2 considerând resturile modulo 3
n(n − 1) k=1 ln2 (1 + k1 )

78
ï ò
2n + 3 1
ale lui n, observăm că egalitatea [B] = + are loc dacă n = 3q şi
6 9n + 9

n = 3q + 2, q ∈ N , ı̂nsă nu este adevărată pentru n = 3q + 1, q ≥ 3.
Pn x2i
L311. Fie x1 , x2 , . . . , xn ∈ R, n ≥ 2, astfel ı̂ncât 2 = 1. Arătaţi că
i=1 1 + xi
1
|x1 x2 . . . xn | ≤ √ . În ce caz avem egalitate?
( n − 1)n
Lucian Tuţescu, Craiova şi Aurel Chiriţă, Slatina
Soluţie. Avem
 
1 x21 x22 x2n n−1 x22 · . . . · x2n
2 =1− 2 = 2 + ... + ≥ (n − 1)
1 + x1 1 + x1 1 + x2 1 + xn2 (1 + x22 ) · . . . · (1 + x2n )
şi analoagele. Prin ı̂nmulţirea celor n inegalităţi deducem că
1 x21 x22 · . . . · x2n
≥ (n − 1)n ,
(1 + x21 )(1 + x22 ) · . . . · (1 + x2n ) (1 + x1 )(1 + x22 ) · . . . · (1
2 + x2n )
de unde obţinem cerinţa.
Pentru n = 2, din relaţia de condiţie obţinem (x1 x2 )2 = 1, deci avem egalitate
pentru orice x1 , x2 . Pentru n ≥ 3, avem egalitate dacă şi numai dacă
1
|x1 | = |x2 | = . . . = |xn | = √ .
n−1
x2i P
n
Este uşor de observat că inegalitatea este adevărată ı̂n ipoteza 2 ≤ 1.
i=1 1 + xi
Notă. Am primit soluţii corecte din partea domnilor Titu Zvonaru, Marius
Olteanu, Ioan Viorel Codreanu şi Dan Dumitrescu.
L312. Fie a, b, c numere reale pozitive astfel ı̂ncât abc = 1. Arătaţi că a3 + b3 +
2ab 2bc 2ca
c3 + 2 + 2 + 2 ≥ 6.
a + b2 b + c2 c + a2
Titu Zvonaru, Comăneşti şi Bogdan Ioniţă, Bucureşti
Soluţia 1 (a autorilor). Avem:
2ab 2bc 2ca X
a3 + b 3 + c3 + 2 2
+ 2 2
+ 2 2
≥6⇔ a3 − 3abc ≥
a +b b +c c +a
XÅ 2ab
ã X (a + b + c)(a − b)2 X (a − b)2
≥ 1− 2 ⇔ ≥ .
a + b2 2 a2 + b 2
2(a − b)2
Este suficient să demonstrăm că (a + b + c)(a − b)2 ≥ . Dacă a = b avem
a2 + b 2
egalitate: dacă nu, rămâne de arătat că (a+b+c)(a +b ) ≥ 2 ⇔ (a+b)(a2 +b2 )+c(a2 +
2 2

b2 ) ≥ 2. Această inegalitate rezultă ţinând cont de faptul că c(a2 + b2 ) ≥ 2abc = 2.


Soluţia 2 (Nicuşor Zlota şi Marius Olteanu). Folosind inegalitatea lui Berg-
ström şi faptul că x3 + y 3 ≥ x2 y + xy 2 , ∀x, y ∈ (0, ∞), avem:
X ab X 12 (1 + 1 + 1)2 9
2 2
= 2 2
≥ P 2 2
≥ P 3 .
a +b c(a + b ) (a b + ab ) (a + b3 )

79
9
Rămâne să demonstrăm că a3 + b3 + c3 + ≥ 6, fapt care revine la ine-
a3 + b 3 + c3
galitatea evidentă (a3 + b3 + c3 − 3)2 ≥ 0.
Soluţia 3 (Ioan Viorel Codreanu şi Marian Cucoaneş). Pentru a, b numere
a3 + b 3 (a + b)(a2 + b2 )
reale pozitive are loc inegalitatea ≥ (este echivalentă cu
2 4
2
(a − b) ≥ 0). Folosind inegalitatea mediilor, avem:

(a + b)(a2 + b2 ) 2ab √ »
+ 2 ≥ 2 · ab(a + b),
4 a + b2
a3 + b 3 2ab √ p
prin urmare + 2 2
≥ 2 · ab(a + b) şi ı̂ncă două inegalităţi similare.
2 a +b Q Q
Ţinând cont, din nou, de inegalitatea mediilor şi de binecunoscuta (a + b) ≥ 8 a,
obţinem:
XÅ 2ab
ã √ X» √ q Y
3 6 2 b 2 c2
a + 2 ≥ 2 · ab(a + b) ≥ 3 2 · a (a + b) ≥ 6abc = 6.
a + b2

Notă. Am mai primit soluţii corecte de la elevii Vlad-Mihai Ciuperceanu,


Craiova şi Dan Dumitrescu, Rm. Vâlcea.
L313. Fie a, b, c, d numere reale pozitive astfel ı̂ncât abc + bcd + cda + dab = 4.
Demonstraţi că

(a12 − a8 + 4)(b10 − b6 + 4)(c8 − c4 + 4)(d6 − d2 + 4) ≥ 256.

Nicuşor Zlota, Focşani


Soluţie. Dacă a = b = c = d = 1, cerinţa are loc cu egalitate. Considerăm
funcţiile fi : R∗+ → R, f1 (x) = x12 − x8 + 4, f2 (x) = x10 − x6 + 4, f3 (x) = x8 − x4 + 4,
f4 (x) = x6 − x2 + 4 şi gi : R → R, gi (x) = ki x4 + mi , i = 1, 4, unde numerele ki
şi mi sunt alese astfel ı̂ncât fi (1) = gi (1) şi fi′ (1) = gi′ (1), i = 1, 4. Cum fi (1) =
fi′ (1) = 3, vom avea, de fapt, gi (x) = x4 + 3, i = 1, 4. Din x6 (x2 − 1)2 (x2 + 1) ≥ 0
şi (x2 − 1)2 (x2 + 1) ≥ 0, ∀x ∈ R∗+ , rezultă că f1 (x) ≥ f2 (x) ≥ f3 (x) ≥ f4 (x) ≥ gi (x),
∀x ∈ R∗+ .
În continuare, vom aplica inegalitatea lui Hölder, precum şi cunoscuta

(a + b + c + d)3 ≥ 16(abc + bcd + cda + dab),


1 a+b+c+d
care reprezintă inegalitatea Maclaurin p1 ≥ p33 , unde p1 = şi p3 =
4
abc + bcd + cda + dab
. Avem:
4
f1 (a) · f2 (b) · f3 (c) · f4 (d) ≥ g1 (a) · g2 (b) · g3 (c) · g4 (d) =
Y
= (a4 + 14 + 14 + 14 ) ≥ (a + b + c + d)4 ≥
»
≥ 3 (16(abc + bcd + cda + dab))4 = 256.

80
Notă. Soluţii ı̂n aceeaşi manieră am primit din partea domnului Marian Cu-
coaneş, Mărăşeşti, precum şi din partea elevului Dan Dumitrescu, Rm. Vâlcea.

2 2 2
L314. Fie√ a,
√b, c numere reale astfel ı̂ncât a + b + c = 9 şi a + b + c = 3s,
unde
√ s√ ∈ [− 3, 3]. Notăm Ms = max(2a + 2b + 2c − abc). Determinaţi min{Ms |s ∈
[− 3, 3]}.
Leonard Giugiuc, Drobeta Tr. Severin şi Marian Cucoaneş, Mărăşeşti
p2 − q 2
Soluţie. Dacă p = a + b + c, = ab + bc + ca (q ≥ 0) şi r = abc, are loc
3
inegalitatea Vo Quoc Ba Can:
1
(∗) r≥ (p + q)2 (p − 2q).
27

√ √ 27 − p2
În condiţiile problemei, p = 3s ∈ [−3 3, 3 3] şi q = , prin urmare 2p − r ≤
p 2
1 7p 5p3 (27 − p2 ) 27 − p2
2p − (p + q)2 (p − 2q) = − + √ . Cum egalitatea ı̂n (*) este
27 2 54 27 2 √ √
efectiv atinsă, obţinem că Ms √= f (p), unde f : [−3 3, 3 3] → R este definită prin
7x 5x3 (27 − x2 ) 27 − x2
f (x) = − + √ . Funcţia f este derivabilă, având derivata
2 54 27 2
1 √
f ′ (x) = (63 − 5x2 − x 54 − 2x2 ) şi tabelul de variaţie:
18

√ √ 37
În concluzie, min{Ms |s ∈ [− 3, 3]} = min f = − √ .
3 3
L315. Se consideră expresia E(z) = |az 2 + bz + c| + |bz 2 + cz + a| + |cz 2 + az + b|,
unde z ∈ C este variabil şi a, b, c ∈ R∗ sunt fixate. Determinaţi max{E(z)||z| = 1}.
Marcel Chiriţă
Soluţie (Gheorghe Iurea, Iaşi). Cum (α+β+γ)2 ≤ 3(α2 +β 2 +γ 2 ), ∀α, β, γ ∈ R,
avem:
X X
E 2 (z) ≤ 3 |az 2 + bz + c|2 = 3 (az 2 + bz + c)(az 2 + bz + c).

Deoarece |z| = 1, avem că zz = 1 şi z = cos t + i sin t, t ∈ [0, 2π]. Efectuând calculele,
relaţia de mai sus devine

E 2 (z) ≤ 9(a2 + b2 + c2 ) + 3(ab + bc + ca)(4 cos2 t + 4 cos t − 2).

Observăm că 4 cos2 t + 4 cos t − 2 = (2 cos t + 1)2 − 3 ∈ [−3, 6], ∀t ∈ [0, 2π].

81
Dacă ab+bc+ca ≥ 0, atunci E 2 (z) ≤ 9(a2 +b2 +c2 )+3(ab+bc+ca)·6 = 9(a+b+c)2 ,
prin urmare E(z) ≤ 3|a + b + c|. Cum E(1) = 3|a + b + c| rezultă că, ı̂n acest caz,
Emax = 3|a + b + c|.
Dacă ab + bc + ca < 0, atunci E 2Ç(z) ≤ 9(a 2
√+
2 2
åb + c ) + 3(ab + bc + ca)(−3) =
1 3 √
9(a2 + b2 + c2 − ab − bc − ca). Cum E − + i = 3 a2 + b2 + c2 − ab − bc − ca,
2 2

rezultă că, ı̂n acest caz, Emax = 3 a2 + b2 + c2 − ab − bc − ca.

IMPORTANT
ˆ În scopul unei legături rapide cu redacţia revistei, pot fi utilizate următoarele
adrese e-mail: t birsan@yahoo.com şi profgpopa@yahoo.co.uk . Pe
această cale colaboratorii pot purta cu redacţia un dialog privitor la ma-
terialele trimise acesteia, procurarea numerelor revistei etc. Sugerăm cola-
boratorilor care trimit probleme originale pentru publicare să le numeroteze
şi să-şi reţină o copie xerox a lor pentru a putea purta cu uşurinţă o discuţie
prin e-mail asupra acceptării/neacceptării acestora de către redacţia revistei.

ˆ La problemele de tip L se primesc soluţii de la orice iubitor de matematici


elementare (indiferent de preocupare profesională sau vârstă). Fiecare dintre
soluţiile acestor probleme - ce sunt publicate ı̂n revistă după jumătate de
an - va fi urmată de numele tuturor celor care au rezolvat-o.

ˆ Adresăm cu insistenţă rugămintea ca materialele trimise revistei


să nu fie (să nu fi fost) trimise şi altor publicaţii.

ˆ Rugăm ca materialele tehnoredactate să fie trimise pe adresa redacţiei


ı̂nsoţite de fişierele lor (de preferinţă ı̂n LATEX).

ˆ Pentru a facilita comunicarea redacţiei cu colaboratorii ei, autorii materi-


alelor sunt rugaţi să indice adresa e-mail.

Vizitaţi pagina web a revistei Recreaţii Matematice:

http://www.recreatiimatematice.ro

82
Probleme propuse1
Clasele primare
P371. Compară lungimea catedrei cu lăţimea uşii din clasa ta, măsurându-le cu
palma.
(Clasa pregătitoare) Georgiana Viţel, elevă, Iaşi
P372. Un copil are patru bancnote de 1 leu, trei bancnote de 5 lei şi o bancnotă
de 10 lei. Câte bancnote va avea copilul dacă schimbă bancnotele de 5 lei şi 10 lei ı̂n
bancnote de 1 leu?
(Clasa pregătitoare) Marian Chiriac, elev, Iaşi
P373. Un elev a călătorit cu trenul de pe data de 4.01.2017, la amiază, până pe
data de 6.01.2017, seara. Alt elev a călătorit cu autobuzul patru jumătăţi de zi şi
ı̂ncă două jumătăţi de oră. Care elev a călătorit mai multe ore?
(Clasa I ) Alexandru Chiriac, Hodora, Iaşi
P374. Într-o clasă, şase elevi au ziua de naştere ı̂n aceeaşi săptămână, dar niciunul
sâmbătă sau duminică. Arătaţi că, măcar ı̂ntr-o zi a săptămânii, cel puţin doi elevi
ı̂şi sărbătoresc ziua de naştere.
(Clasa I ) Victoria Ursu, elevă, Iaşi
P375. Ordonaţi crescător numerele ab, cd, ef , gh ştiind că sunt ı̂ndeplinite condi-
ţiile: a > g, g = c, c > e, d < h.
(Clasa I ) Beatrice Spı̂nu, elevă, Iaşi
P376. În câte moduri se poate achita suma de 52 lei cu bancnote de 1 leu, 5 lei
şi 10 lei?
(Clasa a II-a) Cătălin Petrişor, elev, Iaşi
P377. Figura alăturată conţine un pătrat mare şi patru pătrate mici
egale, aşezate la fel pe laturile pătratului mare. Câte axe de simetrie are
această configuraţie geometrică?
(Clasa a II-a) Nicolae Vieru, elev, Iaşi
P378. În câte cazuri un număr de forma a1b este mai mic decât un
număr de forma 8c2?
(Clasa a II-a) Irina Simon, elevă, Iaşi
P379. Află perechile de numere naturale (a, b) care satisfac simultan egalităţile
a + a + a + a = b şi a · a = b.
(Clasa a III-a) Andreea Munteanu, elevă, Iaşi
P380. Refaceţi ı̂mpărţirea alăturată, ı̂nlocuind steluţele cu
cifre potrivite.
(Clasa a III-a) Alexandra Mădălina Ciobanu, Iaşi
P381. Un elev scrie numerele de la 100 la 999
fără să le despartă prin virgulă, după cum urmează:
100101102 . . . 111 112 . . . 997998999. Câte secvenţe de câte cinci
cifre vecine egale apar ı̂n şir? (Un exemplu de astfel de secvenţă este cea subliniată).
(Clasa a III-a) Adelin Nicolae Bechet, elev, Iaşi
1 Se primesc soluţii până la data de 10 iunie 2017.

83
P382. Se consideră şirul de numere: 1, 2, 2, 3, 3, 3, 1, 2, 2, 3, 3, 3, . . . Aflaţi suma
numerelor de pe locurile 301, 482 şi 2017.
(Clasa a III-a) Bianca Ţugui, elevă, Iaşi
P383. Pe o tablă este scris de 13 ori numărul 22 şi de 15 ori numărul 25. Câte
numere trebuie şterse pentru ca suma numerelor rămase să fie 398?
(Clasa a IV-a) Petru Asaftei, Iaşi
P384. Se consideră numărul A = abcd + bcda + cdab + dabc. Aflaţi numărul abcd
ştiind că a < b < c < d şi A = 10 · 11 · 101.
(Clasa a IV-a) Andreea Bı̂zdı̂gă, studentă, Iaşi
1 1 1 1 1
P385. Dintr-o bară se taie cinci bucăţi egale cu , , , şi din bară. Ce
2 4 8 32 64
fracţie din bară rămâne?
(Clasa a IV-a) Monica Maftei, elevă, Iaşi
P386. Suma a şapte numere pare şi nenule este 26. Calculaţi produsul tuturor
diferenţelor de câte două numere dintre cele şapte.
(Clasa a IV-a) Denisa Apetrei, elevă, Iaşi
Clasa a V-a
V.214. Determinaţi numerele naturale a, b şi c, ştiind că c este prim iar a + b =
a − b + c = 2017.
Tinuţa Bejan, Iaşi
V.215. Adăugând o cifră la stânga şi o cifră la dreapta numărului 2017, obţinem
un număr de şase cifre care este divizibil cu 198. Determinaţi acest număr de şase
cifre.
Vlad Mihai Ciuperceanu, elev, Craiova
V.216. Adunăm un număr natural cu dublul sumei cifrelor sale. Arătaţi că
numărul obţinut este divizibil cu 3.
Marian Ciuperceanu, Craiova
V.217. Există numere naturale de n cifre (n ∈ N∗ ) a1 a2 . . . an şi b1 b2 . . . bn astfel
ı̂ncât a1 a2 . . . an · b1 b2 . . . bn = a1 a2 . . . an b1 b2 . . . bn ?
Ioan Viorel Codreanu, Satulung (Maramureş)
V.218. Determinaţi numerele naturale nenule a şi b cu proprietatea că a! + b2 =
20172 + 1. (Am notat a! = 1 · 2 · . . . a.)
Cosmin Aştefanei, elev, Iaşi
V.219. Determinaţi numerele naturale a, b şi c cu proprietatea că a2 + b2 + c2 =
22017 .
Cătălin Budeanu, Iaşi
V.220. Câte numere de două cifre se scriu sub forma ab − a, unde a, b sunt cifre,
a 6= 0?
Gheorghe Iurea, Iaşi
Clasa a VI-a
a b c
VI.214. Determinaţi numerele naturale nenule a, b şi c pentru care + + = 4.
2 3 5
Ionuţ-Florin Voinea, elev, Bucureşti

84
x
VI.215. Determinaţi numerele naturale x şi y pentru care 92 − 243y = 6560.
Ioan Viorel Codreanu, Satulung (Maramureş)
VI.216. Determinaţi numerele prime p şi q astfel ı̂ncât numărul (p + 9)q să fie
pătrat perfect.
Lucian Tuţescu şi Carmen Terheci, Craiova
VI.217. Determinaţi numerele naturale a, b, c, cel puţin egale cu 2, pentru care
5ab − 7
este număr natural nenul.
abc + 2
Mihaela Berindeanu, Bucureşti
VI.218. Determinaţi numărul A = mn+5 · nm+4 , m, n ∈ N\{0, 1}, care are exact
63 de divizori naturali.
Ionel Tudor, Călugăreni
VI.219. Dat triunghiul ABC, notăm cu A′ simetricul lui A faţă de C şi cu C ′
simetricul lui C faţă de B. Paralela prin B la AC intersectează dreapta A′ C ′ ı̂n M
iar B ′ este simetricul lui B faţă de M . Demonstraţi că BA′ = B ′ C ′ .
Valeriu Iovan, Craiova
VI.220. Se consideră triunghiul isoscel ABC cu AB = AC şi m(A) b < 40◦ .
Ştiind că există E ∈ (AB) şi F ∈ (AC) astfel ı̂ncât AE = CF = BC şi AF = F E,
determinaţi măsura unghiului A. b
Vasile Chiriac, Bacău
Clasa a VII-a
VII.214. Pentru n ∈ Z, notăm E(n) = (n − 1)(n − 2) . . . (n − 2016).
a) Demonstraţi că E(n) ≥ 0, oricare ar fi n ∈ Z.
b) Determinaţi cel mai mic număr ı̂ntreg m astfel ı̂ncât E(n) > 0, oricare ar fi
n ∈ Z, n ≥ m.
Alecu Orlando şi Mihaela Roşioru, Roşiorii de Vede
VII.215. Determinaţi numerele ı̂ntregi m şi n pentru care m2 −6mn+5n2 = 2017.
Ionel Tudor, Călugăreni şi Viorica Dogaru, Giurgiu
√ VII.216.
√ Determinaţi numerele naturale abc cu cifrele a, b, c nenule, ştiind că
abc + bc este număr natural.
Nicolae Ivăşchescu, Craiova
VII.217. Determinaţi cea mai mică valoare a numărului natural n pentru care
13n + n se divide cu 61.
Titu Zvonaru, Comăneşti
VII.218. Se consideră triunghiul isoscel ABC cu AB = AC = 15 şi BC = 24.
Pe semidreapta opusă bisectoarei unghiului A b se consideră punctul H astfel ı̂ncât
AH = 7. Arătaţi că H este ortocentrul triunghiului ABC.
Viorica Momiţă, Iaşi
VII.219. Fie M, N, P mijloacele laturilor DA, AB şi BC ale patrulaterului convex
ABCD iar Q un punct pe latura CD, diferit de mijlocul acesteia. Arătaţi că ABkCD
1
dacă şi numai dacă AMN P Q = AABCD .
2
Ovidiu Pop, Satu Mare

85
VII.220. Se consideră trapezul ABCD (ABkCD) şi punctele M ∈ (AD) şi
N ∈ (BC) astfel ı̂ncât M N kAB. Arătaţi că patrulaterele AM CN şi BM DN sunt
sau nu simultan trapeze.
Ioan Pop, Iaşi
Clasa a VIII-a
VIII.214. a) Dacă un triunghi are aria egală cu 2, atunci cel puţin două dintre
laturile sale au lungimile cel puţin egale cu 2.
b) Arătaţi că există triunghiuri care să aibă aria egală cu 2 şi exact două laturi
mai mari decât 2.
Maria Rusu, Târgu Frumos
VIII.215. Fie I centrul sferei ı̂nscrisă ı̂n tetraedrul echifacial ABCD. O dreaptă d
care trece prin I intersectează planele (BCD), (ACD), (ABD) şi (ABC) ı̂n A′ , B ′ , C ′
respectiv D′ . Dacă punctul L, interior tetreaedrului, se află pe dreapta d, arătaţi că
A′ L B ′ L C ′ L D′ L
+ ′ + ′ + ′ = 4.
A′ I BI CI DI
Constantin Petrea, Paşcani
VIII.216. Fie a, b, c, d, e numere ı̂ntregi astfel ı̂ncât a3 + b3 + c3 + d3 + e3 = 1001.
Demonstraţi că cel puţin unul dintre cele cinci numere este divizibil cu 7.
Roxana Vasile şi Ileana Dragomir, Craiova
VIII.217. Fie p un număr natural prim. Vom spune că numărul ı̂ntreg a este
x py
p-admisibil dacă există x, y ∈ Z∗ , x 6= y astfel ı̂ncât + = a. Determinaţi numerele
y x
2017-admisibile.
Dan Popescu, Suceava
VIII.218. Demonstraţi că există n ∈ N∗ astfel ı̂ncât ultimele 2017 cifre ale
numărului 2017n să fie 00 | .{z
. . 00} 1.
2016 de 0
Marian Voinea şi Laurenţiu Moldovan, Bucureşti
VIII.219. Demonstraţi că există o infinitate de perechi (x, y) de numere ı̂ntregi
pentru care x3 − 30y = 56.
Ioan Viorel Codreanu, Satulung (Maramureş)
VIII.220. Determinaţi n ∈ N∗ astfel ı̂ncât numărul A = 2n + n18 să fie prim.
Dan Lucian Grigorie şi Constantina Prunaru, Craiova
Clasa a IX-a
IX.176. Dacă a, b, c sunt numere reale pozitive, arătaţi că
X Å ã
1 1 1 1 1
≤ √ √ + √ + √ .
(a + b)2 + (a + c)2 8 abc a b c
Ovidiu Pop, Satu Mare
IX.177. Fie (an )n≥1 o progresie aritmetică cu a1 > 0 şi r ∈ N∗ . Ştiind că există
Å ã
∗ a1 + a2 + . . . + ak k+1
k ∈ N astfel ı̂ncât = a21 rk−1 , arătaţi că an ∈ N∗ , ∀n ∈ N∗ .
1 + 2 + ... + k
Cătălin Cristea, Craiova

86
IX.178. Fie √ x, y ∈ R astfel ı̂ncât 9x2 + 25y 2 − 6x + 30y + 1 = 0. Arătaţi că
|3x − 5y − 4| ≤ 3 2.
Constantin Dragomir, Piteşti
IX.179. În triunghiul ABC (cu notaţiile uzuale) are loc inegalitatea

a2 · ra + b2 · rb + c2 · rc ≥ 108r3 .

D.M. Bătineţu-Giurgiu, Bucureşti şi Neculai Stanciu, Buzău


IX.180. Presupunem că ı̂n interiorul triunghiului isoscel ABC (AB = AC) există

un punct P astfel ı̂ncât m(P BC) = 15◦ , m(P ’CB) = 30◦ şi P A ⊥ P B. Demonstraţi
că triunghiul ABC este echilateral.
Titu Zvonaru, Comăneşti

Clasa a X-a
X.176. Dacă a, b, c, d ∈ (0, 1) sau a, b, c, d ∈ (1, ∞), arătaţi că

logabc bd2 + logabd ac2 + logbcd ca2 + logacd db2 ≥ 4.

Petru Asaftei, Iaşi


X.177. Determinaţi z ∈ C, ştiind că 5z 2017 − |z| = 4 şi z 2015 + 2|z| = 3.
Oana Preda şi Tatiana Cristea, Craiova
X.178. Determinaţi x, y ∈ R pentru care x3 − 3xy 2 = −46 şi y 3 − 3x2 y = −9.
Marian Cucoaneş, Mărăşeşti
X.179. Numerele complexe x, y, z au module egale cu 1 şi xyz = x + y + z + 2.
Arătaţi că două dintre numere sunt egale cu −1.
Marian Tetiva, Bârlad
X.180. Determinaţi x ∈ (1, ∞) pentru care xlog3 2 + 1 = (x − 1)log2 3 .
Cosmin Manea şi Dragoş Petrică, Piteşti

Clasa a XI-a
XI.176. Dacă A ∈ Mn (R) şi B = A · At , arătaţi că B 6 + B 4 + In 6= B 3 + B.
Bogdan-Petre Posa, Bucureşti
XI.177. Fie f : R → R o funcţie concavă cu proprietatea
Å ã că f (x2 + x + 1) ≤
1
f (x + 1), ∀x ∈ R. Demonstraţi că f (x2 + x + 1) ≤ f , oricare ar fi x ∈ R.
2
Dumitru Crăciun, Fălticeni
XI.178. Fie (λn )n≥1 un şir cu termeni pozitivi astfel ı̂ncât şirul (λn+1 − λn )n≥1
este mărginit. Calculaţi limita şirului (xn )n≥1 definit prin
Ç å n1
1 1 1
xn = √ +√ + ...+ p + λn .
n+1 n+2 (n + 1)2

Gheorghe Costovici, Iaşi

87
x0 ≥ 2, xn+1 = x4n − 4x2n + 2, ∀n ∈
XI.179. Considerăm şirul (xn )n≥0 definit prin …
q » q »
√ √
N. Notăm an = 2 + 2 + . . . + 2 + xn şi bn = −2 + 2 + 2 + . . . + 2 + xn ,
| {z } | {z }
3n radicali 3n−2 radicali
n ∈ N∗ . Determinaţi limita şirului cn = 2n (an + bn − 2), n ∈ N∗ .
Florin Stănescu, Găeşti
p √
XI.180. Fie (Ln )n≥1 şirul lui Traian Lalescu, Ln = n+1 (n + 1)! − n n!, n ∈ N∗
Pn 1 p √
şi xn = , n ∈ N∗ . Calculaţi limita şirului Bn = exn+1 · n+1 Ln+1 − exn · n Ln .
k=1 k
D.M. Bătineţu-Giurgiu, Bucureşti şi Neculai Stanciu, Buzău

Clasa a XII-a
XII.176. Fie f : R → R o funcţie continuă şi impară. Calculaţi
Z e
1
dx.
1 (x2 + 1)(1 + xef (ln x) )
e

D.M. Bătineţu-Giurgiu, Bucureşti şi Neculai Stanciu, Buzău


Z 1
x2
XII.177. Calculaţi ln((1 + x)(1 + x9 )) · dx.
0 1 + x3
Lucian Tuţescu şi Ionuţ Ivănescu, Craiova
Z 1
π
XII.178. Fie f : [0, 1] → R o funcţie continuă astfel ı̂ncât f (x)dx = √ .
0 3 3
1 1
Demonstraţi că există c ∈ (0, 1) cu proprietatea că < f (c) < .
1 + 2c 3c
Mihai Haivas, Iaşi
XII.179. Considerăm funcţiile continue f : (0, ∞) → R şi g : (0, ∞) → (0, ∞)
astfel ı̂ncât lim f (x) = 0 şi g(x + 1) ≤ f (x) ≤ g(x), ∀x ∈ (0, ∞). Arătaţi că şirul
Z n+1 x→∞
xn = g(x)dx, n ≥ 1, este convergent şi determinaţi limita sa.
n
Cosmin Manea şi Dragoş Petrică, Piteşti
XII.180. Fie p un număr natural prim, p ≡ 1 (mod 5). Arătaţi că ecuaţia x2 = b
5
are soluţii ı̂n Zp .
Marian Cucoaneş, Mărăşeşti

Probleme pentru pregătirea concursurilor


A. Nivel gimnazial
G316. Se consideră a pungi numerotate 1, 2, . . . , a, fiecare conţinând câte b mo-
nede, b > a. Masele tuturor monedelor se exprimă prin numere ı̂ntregi şi, cu excepţia
unei pungi, toate monedele au aceeaşi masă. În acea pungă se află numai monede
false, fiecare având masa cu p mai mică decât masa unei monede adevărate, unde p

88
nu este multiplu de a − 1. Folosind un cântar cu afişaj, determinaţi din trei cântăriri
care este numărul pungii cu monede false.
Geanina Hăvârneanu, Iaşi
G317. Aflaţi numerele naturale n pentru care n! + (n + 1)! + (n + 3)! este cub
perfect.
Ioan Viorel Codreanu, Satulung (Maramureş)
G318. Fie x, y, z numerele reale pozitive astfel ı̂ncât xyz = x + y + z + 2.
Demonstraţi că au loc inegalităţile:
X X 1 X 1
x≥2 √ ≥6 √ ≥ 6.
xy − 1 xy − xy + 1

Marian Tetiva, Bârlad


G319. Dacă a, b, c ∈ (0, 1), demonstraţi că
X (a + b)2 X X X
> 3( a2 ) − 2( ab) + 3( a).
2
Cosmin Manea şi Dragoş Petrică, Piteşti
G320. a) Dacă a, b, c sunt lungimile laturilor unui triunghi, arătaţi că 0 ≤
|a − b| |b − c| |c − a|
+ + < 2.
a+b b+a c+a
b) Pentru orice t ∈ [0, 2), există un triunghi ale cărui laturi a, b, c au proprietatea
|a − b| |b − c| |c − a|
că 0 ≤ + + ≤ t.
a+b b+c c+a
Constantin Dragomir, Piteşti
G321. În interiorul triunghiului dreptunghic isoscel ABC, AB = AC, se consideră
punctul M astfel ı̂ncât m(M÷ CA) = 15◦ . Dacă Q este proiecţia lui A pe M C şi
AQ 1 ÷
= , determinaţi măsura unghiului M BC.
MC 2
Mihai Berindeanu, Bucureşti
G322. Fie ABCD un patrulater convex, {O} = AC ∩ BD, iar P este simetricul
lui O faţă de mijlocul segmentului AD. Demonstraţi că ABkCD dacă şi numai dacă
aria patrulaterului AP DO este media armonică a ariilor triunghiurilor ABD, ABC,
ACD şi BCD.
Claudiu-Ştefan Popa, Iaşi
G323. Fie ABC un triunghi oarecare, D, E, F mijloacele laturilor BC, CA re-
spectiv AB şi M, N, P proiecţiile centrului de greutate pe laturile BC, CA respectiv
AB. Dacă △M N P ∼ △DEF , arătaţi că triunghiul ABC este echilateral.
Temistocle Bı̂rsan, Iaşi
G324. Fie ABCD un patrulater inscriptibil cu CD = AD + BC. Demonstraţi că
“ se intersectează ı̂ntr-un punct situat pe
b şi B
bisectoarele interioare ale unghiurilor A
latura CD.
Neculai Roman, Mirceşti (Iaşi)

89
G325. Se consideră tetraedrul ABCD şi punctele M, N, P şi Q pe segmentele
MA NB PC QD
AB, BC, CD respectiv DA astfel ı̂ncât = = = = k > 1. Fie
MB NC PD QA
{X} = AC ∩ M N, {Y } = BD ∩ N P, {Z} = AC ∩ P Q şi {T } = BD ∩ M Q.
a) Arătaţi că tetraedrele ABCD şi XY ZT au acelaşi centru de greutate.
Å 2 ã2
VXY ZT k +1
b) Demonstraţi că = .
VABCD k2 − 1
Marius Olteanu, Râmnicu Vâlcea

B. Nivel liceal

L316. Fie ABC un triunghi dreptunghic, m(A) b = 90◦ şi punctele P, Q pe cateta
AC, P ı̂ntre A şi Q, astfel ı̂ncât m(P ’ ’ = m(C)
BA) = m(QBC) “ ≤ 30◦ .
a) Arătaţi că centrul cercului circumscris triunghiului BCP este simetricul punc-
tului Q faţă de BC.
b)
 πDeterminaţi valoarea
 π i maximă a produsului P A·QA şi deduceţi că tg α·ctg 2α ≤
α
tg2 − , ∀α ∈ 0, .
4 2 6
Mihail Frăsilă şi Constantin Petrea, Paşcani
L317. Fie O, H centrul cercului circumscris, respectiv ortocentrul triunghiului
ascuţitunghic ABC. Considerăm punctul D pe latura AB şi punctul E pe latura AC
astfel ı̂ncât A este centrul cercului exı̂nscris triunghiului ODE. Demonstraţi că DE
este mediatoarea segmentului AH.
Titu Zvonaru, Comăneşti şi Bogdan Ioniţă, Bucureşti
L318. Fie ABC un triunghi dreptunghic ı̂nscris ı̂n cercul C de rază R şi D piciorul
ı̂nălţimii din vârful unghiului drept A. Cercurile C1 şi C2 de centre O1 şi O2 tangente
interior cercului C mai sunt tangente şi semidreptelor [AB şi [AD şi respectiv [AC
8Rr2
şi [AD. Arătaţi că aria(△AO1 O2 ) = , unde r este raza cercului ı̂nscris ı̂n
2R + r
triunghiul ABC.
Neculai Roman, Mirceşti (Iaşi)
L319. Fie I centrul cercului ı̂nscris ı̂n triunghiul ABC. Notăm cu O1 , O2 şi O3
centrele cercurilor circumscrise tringhiurilor BIC, CIA respectiv AIB. Demonstraţi
că
1 1 1 1 1 1
+ + ≥ + + .
AB · O3 O1 BC · O1 O2 CA · O2 O3 AB · BC BC · CA CA · AB

Florin Stănescu, Găeşti


L320. Într-un plan se dau două puncte B şi C şi o dreaptă ∆ paralelă cu BC.
Un punct A este mobil pe ∆. Să se detemrine locul geometric al circumcentrului
triunghiului median al triunghiului ABC.
Temistocle Bı̂rsan, Iaşi

90
L321. Notăm cu [x] şi {x} partea ı̂ntreagă respectiv partea fracţionară ale
numărului real x. Demonstraţi că, pentru orice număr real pozitiv x, are loc ine-
galitatea
[x]2 {x}2 11
2 2
+ ≤ .
7[x] + (2{x} + [x]) 7{x} + (2[x] + {x})2
2 72
Nicuşor Zlota, Focşani

L322. Fie a, b, c ∈ (0, ∞) cu a + b + c = 3. Demonstraţi că
√ √
3 a b c 3 3
≤ 2 + + ≤ .
4 a + 1 b 2 + 1 c2 + 1 4
Generalizare!
Tidor Pricope, elev, Botoşani
L323. Demonstraţi că, pentru oricare numere a, b, c din intervalul (−1, 1), are loc
inegalitatea

(1 − abc)3 (1 − a3 )(1 − b3 )(1 − c3 ) ≤ (1 − a2 b)(1 − ab2 )(1 − a2 c)(1 − ac2 )(1 − b2 c)(1 − bc2 ).

Marian Tetiva, Bârlad


L324. Dacă a, b, c ∈ (0, ∞), demonstraţi inegalităţile:
X 2a2 a2 + b 2 + c2
a) ≥3 ;
b+c a+b+c
X a3 3 a2 + b 2 + c2
b) 2 2
≥ .
b +c 2 a+b+c
Neculai Stanciu, Buzău şi Titu Zvonaru, Comăneşti
L325. Dacă x, y, z ∈ (0, ∞), demonstraţi că

x2 y2 z2 » 2 3(x2 + y 2 + z 2 − xy − yz − zx)
+ + ≥ 3(x + y 2 + z 2 ) + .
y z x x+y+z
Leonard Giugiuc, Drobeta-Tr. Severin şi Marian Cucoaneş, Mărăşeşti

Training problems for mathematical contests

A. Junior highschool level


G316. A number of a bags, numbered 1, 2, . . . , a , are considered, such that
each of them contains b coins, b > a. The masses of all the b coins in each bag are
expressed by integer numbers and − except one bag − all the coins [coin bags] have
the same mass. That bag contains false coins only, each coin having its mass less
by p than the mass of a true coin, such that p is not a multiple of a − 1. Using a
scale with digital display, determine the number of the bag with false coins by three
weighings only.
Geanina Hăvârneanu, Iaşi

91
G317. Find the natural numbers n such that n! + (n + 1)! + (n + 3)! is a perfect
cube.
Ioan Viorel Codreanu, Satulung (Maramureş)
G318. Let x, y, z be positive real numbers such that x y z = x + y + z + 2. Prove
that the following inequalities hold :
X X 1 X 1
x≥2 √ ≥6 √ ≥ 6.
xy − 1 xy − xy + 1

Marian Tetiva, Bârlad


G319. If a, b, c ∈ (0, 1), prove that
X (a + b)2 X X X
> 3( a2 ) − 2( a b) + 3( a).
2
Cosmin Manea şi Dragoş Petrică, Piteşti
G320. a) If a, b, c are the side lengths of a triangle, show that
|a − b| |b − c| |c − a|
0≤ + + < 2.
a+b b+a c+a
b) For any t ∈ [0, 2), there exists a triangle whose sides a, b, c have the property
|a − b| |b − c| |c − a|
that 0 ≤ + + ≤ t.
a+b b+c c+a
Constantin Dragomir, Piteşti
G321. It is considered, in the interior of the right-angled isosceles triangle ABC,
AB = AC, the point M such that m(M ÷ CA) = 15◦ . If Q is the projection of A on
AQ 1 ÷
M C and = , determine the measure of the angle M BC.
MC 2
Mihai Berindeanu, Bucureşti
G322. Let ABCD be a convex quadrilateral, {O} = AC ∩ BD, and is the
symmetric of O with respect to the midpoint of the segment AD. Prove that ABkCD
if and only if the area of the quadrilateral AP DO is the harmonic mean of the areas
of triangles ABD, ABC, ACD and BCD.
Claudiu-Ştefan Popa, Iaşi
G323. Let ABC be an arbitrary triangle, D, E, F the midpoints of the sides
BC, CA and AB respectively and let M, N, P be the projections of the gravity
center on the sides BC, CA and AB respectively. If △ M N P ∼ △ DEF , show that
the triangle ABC is equilateral.
Temistocle Bı̂rsan, Iaşi
G324. Let ABCD be an inscribable quadrilateral with CD = AD + BC. Prove
“ intersect each other at a point
b and B
that the interior angle bisectors of angles A
situated on the side CD.
Neculai Roman, Mirceşti (Iaşi)
G325. It is considered the tetrahedron ABCD and the points M, N, P and Q
MA NB PC
on the segments AB, BC, CD and AD respectively such that = = =
MB NC PD

92
QD
= k > 1. Let {X} = AC ∩ M N, {Y } = BD ∩ N P, {Z} = AC ∩ P Q and
QA
{T } = BD ∩ M Q.
a) Show that the tetrahedrons ABCD and XY ZT share the same gravity center.
Å 2 ã2
VXY ZT k +1
b) Prove that = .
VABCD k2 − 1
Marius Olteanu, Râmnicu Vâlcea

B. Highschool Level

L316. Let ABC be a right-angled triangle with m(A) b = 90◦ and the points P, Q
situated on the smaller side AC, with P between A and Q, such that m(P ’ BA) =
’ = m(C)
m(QBC) “ ≤ 30 .◦

a) Show that the centre of the circumcircle of the triangle BCP is the symmetric
of point Q with respect to BC.
b) Determine
 π theαmaximum
  value of the product P A · QA and deduce that tan α ·
πi
cot 2α ≤ tan2 − , ∀α ∈ 0, .
4 2 6
Mihail Frăsilă şi Constantin Petrea, Paşcani
L317. Let O, H be the center of the circumcircle and the orthocenter of the
acute-angled triangle ABC. Let us consider the point D on the side AB and point
E on the side AC such that A is the centre of the escribed circle to the triangle
ODE. Prove that DE is the mid-perpendicular of the line segment AH.
Titu Zvonaru, Comăneşti şi Bogdan Ioniţă, Bucureşti
L318. Let ABC be a right-angled triangle inscribed in the circle C of radius R
and D the foot of the altitude from the vertex of the right angle A “ . The circles C1
and C2 of centres O1 and O2 , that are inner-tangent to the circle C are also tangent
to the half-lines [AB & [AD , and [AC & [AD respectively. Show that the area
8Rr2
(△ AO1 O2 ) = , where r is the radius of the inscribed circle (incircle) in the
2R + r
triangle ABC.
Neculai Roman, Mirceşti (Iaşi)
L319. Let I be the center of the circle inscribed in triangle ABC . We denote
by O1 , O2 and O3 the circumcentres of the circumcircles to the triangles BIC, CIA
and AIB respectively. Prove that

1 1 1 1 1 1
+ + ≥ + + .
AB · O3 O1 BC · O1 O2 CA · O2 O3 AB · BC BC · CA CA · AB

Florin Stănescu, Găeşti


L320. In a plane there are given two points B and C , and a straight line ∆
which is parallel to BC. A point A is mobile on ∆. Determine the geometric locus
of the circumcentre of the median triangle to triangle ABC.
Temistocle Bı̂rsan, Iaşi

93
L321. We denote by [x] and {x} the integer part and decimal part of the real
number x, respectively. Prove that, for any positive real number x , the following
inequality holds :

[x]2 {x}2 11
+ ≤ .
7[x]2 + (2{x} + [x])2 7{x}2 + (2[x] + {x})2 72
Nicuşor Zlota, Focşani

L322. Let a, b, c ∈ (0, ∞) with a + b + c = 3. Prove that
√ √
3 a b c 3 3
≤ 2 + + ≤ .
4 a + 1 b 2 + 1 c2 + 1 4
Generalization required !
Tudor Pricope, elev, Botoşani
L323. Prove that, for any three numbers a, b, c in the interval (−1, 1), the
following inequality holds :

(1−a b c)3 (1−a3 )(1−b3 )(1−c3 ) ≤ (1−a2 b)(1−a b2 )(1−a2 c)(1−a c2 )(1−b2 c)(1−b c2).

Marian Tetiva, Bârlad


L324. If a, b, c ∈ (0, ∞), prove the inequalities
X 2a2 a2 + b 2 + c2
a) ≥3 ;
b+c a+b+c
X a3 3 a2 + b 2 + c2
b) 2 2
≥ .
b +c 2 a+b+c
Neculai Stanciu, Buzău şi Titu Zvonaru, Comăneşti
L325. If x, y, z ∈ (0, ∞), prove that

x2 y2 z2 » 2 3(x2 + y 2 + z 2 − xy − yz − zx)
+ + ≥ 3(x + y 2 + z 2 ) + .
y z x x+y+z
Leonard Giugiuc, Drobeta Tr. Severin şi Marian Cucoaneş, Mărăşeşti

Vizitaţi pagina web a revistei Recreaţii Matematice:

http://www.recreatiimatematice.ro

94
Pagina rezolvitorilor

BÂRLAD
Colegiul Naţional ,,Gh. Roşca Codreanu”. Clasa a V-a (prof. DOBRIN Bogdan).
HUIBAN Damian: V(207-210,213).
BRAŞOV
Şcoala Gimnazială nr. 8. Clasa pregătitoare (ı̂nv. VRÎNCEANU Simona).
PESTREA Andra Gabriela: P(355-357,359).
CRAIOVA
Şcoala Gimnazială ,,Gheorghe Ţiţeica”. Clasa I (prof. BANU Carmen). CO-
TOLAN Lucian Iustin: P(355-359).
Palatul Copiilor, Craiova. Clasa a V-a (prof. IOVAN Valeriu). COTOLAN
Alexandra Luminiţa: V(207-210,213). Clasa a VIII-a (prof. IOVAN Valeriu). DU-
MITRU Mihaela Gabriela: VIII(207-211).
Şcoala Gimnazială ,,Mircea Eliade”. Clasa a VI-a (prof. MATEI Monica).
GAVRILĂ Elena Georgiana: V(207,208,210,211,213), VI(207,208,211), VII(208,209).
Colegiul Naţional ,,Fraţii Buzeşti”. Clasa a VI-a (prof. IONESCU Maria). CIU-
PERCEANU Vlad Mihai: VI(207,208,210), VII(207-209,213), VIII(207,210), G311,
L312. Clasa a IX-a (prof. TUŢESCU Lucian). DRĂGHIA Denisa Iulia: VIII(207,
210,211), IX.171, XI.174, G(310,311),L312. Clasa a XI-a (prof. PĂTRAŞCU ION).
BEŢIU Pavel: IX(171-175), X(171-173), XI.174.
IAŞI
Şcoala nr. 26 ,,George Coşbuc”. Clasa a II-a (prof.ı̂nv.primar PÎNZARIU
Petronela). DAROCZI Lara-Daniela: P(357-362); IACOB Antonia-Călina: P(357-
362); HRIŢCU Delia-Gabriela: P(357-362); HUŢAN Andreea-Diana: P(357-362);
MANOLE Şerban-Gabriel: P(357-361). Clasa a IV-a (prof.ı̂nv.primar RACU Maria).
ALUCĂI Denisa: P(355-359); HUŢAN Adela: P(356-359,362,363); NĂSTASE Adrian:
P(355-359,362,363,366,367); PICHIU Alexia: P(358,360-362; 364,367); POPESCU
Alexandra: P(364-370); SCOBAN Dana-Ilinca: P(356,358,360,362,363,367); SAM-
SON David-Alexandru: P(355,356,358,360,362,363); SZTANKAI-CRISTOF Erika:
P(360-362,363,364,366,368). Clasa a VI-a (prof. BEJAN Tinuţa). BĂSU Delia:
P(369,370), V(207-209,211,213), VI.213, G(307,309); CARP Ştefana: P(369,370),
V(207-209,211,213), VI.213, G(307,309); CERNĂUŢEANU Beatrice: P(369,370),
V(207-209,211,213), VI.213, G(307,309); CÎŞLARIU Diana: P(369,370), V(207-209,
211,213), VI.213, G(307,309); MERCAŞ Ioana: P(369,370), V(207-209,211,213),
VI.213, G(307,309); NIŢĂ Amalia: P(369,370), V(207-209,211,213), VI.213, G(307,
309); PÂRĂU Crina: P(369,370), V(207-209,211,213), VI.213, G(307,309); SÎRBU
Bianca: P(369,370), V(207-209,211,213), VI.213, G(307,309).
Liceul Economic ,,Virgil Madgearu”. Clasa a IX-a (prof. OLENIUC Claudia).
AGAFIŢEI Mihaela: VII(207-209), VIII(208,210); AMARANDEI Ioana Alexandra:
VII(207-209), VIII(208,210); CIOBANU Iulian Bogdan: VII(207-209), VIII(208,210);
CHILCOŞ Elena-Sabina: VII(207-209), VIII(208,210); DÎSCĂ Bogdan-Cristian:
VII(207-209), VIII(208,210); GELIP Teodor Daniel: VII(207-209), VIII(208,210);

95
FRUNZĂ Andrei: VII(207-209), VIII(208,210); GROSU Emilia: VII(207-209),
VIII(208,210); HORLEANU Maria-Diana: VII(207-209), VIII(208,210); PAVĂL Edu-
ard: VII(207-209), VIII(208,210); NEDELCU Cosmin: VII(207-209), VIII(208,210).
Colegiul Naţional ,,Emil Racoviţă”. Clasa a IX-a (prof. PIŢU Leon). OLENIUC
Iulian: VII(207-209), VIII(208-210).
Colegiul Naţional Iaşi. Clasa a VI-a (prof. POPA Gabriel). CIORNEI Ştefan:
P(367,370), V(208-210); DUMINICĂ Bogdan: P(367-370), V(207-210,213); ILIE
Alexandru: P367,V(207-209,213); IOŢOVICI Cosmin: V(207-209,211,213); LEHACI
Miruna: P(368-370), V(207-209); LISĂ Răzvan: P(368,370), V(207-209); ROZMA-
LIN Smaranda: P(367-369), V(207,208). Clasa a VII-a (prof. POPA Gabriel). CIO-
COIU Alexandru Boris: VII(207-213); PAVEL-ALEXANDRU Ingrid: V.208, VI.213,
VII(208,210,211); PLEŞCAN Matei-Bogdan: V.208, VI(207,210,213), VII.210; NIMI-
GEANU Ioana-ANDREEA: V(208,210), VI(207,210), VII.210; STRUGARIU Ştefan:
VII(207-211). Clasa a XI-a (prof. POPA Gabriel). ANUŞCĂ Armand: IX.171,
X.171, XI(171-173); AŞTEFANEI Cosmin: IX.171, X.171, XI(171-173); OBADĂ
Ştefan Alexandru: IX.171, X.171, XI(171-173); STRUGARU Irina Mălina: IX.171,
X.171, XI(171-173).
RÂMNICU VÂLCEA
Colegiul Naţional ,,Alexandru Lahovari”. Clasa a X-a (prof. GORGOTĂ Vasile).
DUMITRESCU Dan: X(171-173,175), G311, L(307,311-313).
ROŞIORI (BACĂU)
Şcoala Gimnazială nr. 1. Clasa a V-a (prof. CICEU Nela). ALEXESCU Dragoş:
V(207-210,212), VI.207. Clasa a VIII-a (prof. CICEU Nela). DUBEI Steluţa:
VI.210, VII.208, VIII(207-211,213); PLOŞNIŢĂ Daniel-Cătălin: VI.208, VII.213,
VIII(20/-211,213); ROMAN Vasile: VI(207,213), VII.209, VIII(207-211,213).

Elevi rezolvitori premiaţi

Colegiul Naţional ,,Fraţii Buzeşti”, Craiova


DRĂGHIA Denisa Iulia (cl. a IX-a): 1/2016(10pb), 2/2016(5pb),1/2017(8pb).
Palatul Copiilor, Craiova
DUMITRU Mihaela-Gabriela (cl. a VIII-a): 1/2016(11pb), 2/2016(5pb), 1/2017(5pb).
Colegiul Naţional din Iaşi
CIOCOIU Alexandru Boris (cl. a VI-a): 2/2015(7pb), 1/2016(6pb), 2/2016(6pb).
Şcoala Gimnazială nr. 1, Roşiori (Bacău)
DUBEI Steluţa (cl. a VIII-a): 1/2016(8pb), 2/2016(9pb), 1/2017(7pb).
ROMAN Vasile (cl. a VIII-a): 1/2016(11pb), 2/2016(10pb), 1/2017(9pb).

96
Revista semestrială RECREAŢII MATEMATICE este editată de ASOCIAŢIA
„RECREAŢII MATEMATICE”. Apare la datele de 1 martie şi 1 septembrie şi se
adresează elevilor, profesorilor, studenţilor şi tuturor celor pasionaţi de matematica
elementară.
În atenţia tuturor colaboratorilor
Materialele trimise redacţiei spre publicare (note şi articole, chestiuni de metodică,
probleme propuse etc.) trebuie prezentate îngrijit, clar şi concis; ele trebuie să prezinte
interes pentru un cerc cât mai larg de cititori. Se recomandă ca textele să nu depăşească
patru pagini. Evident, ele trebuie să fie originale şi să nu fi apărut sau să fi fost
trimise spre publicare altor reviste. Rugăm ca materialele tehnoredactate să fie
însoţite de fişierele lor.
Problemele destinate rubricilor: Probleme propuse şi Probleme pentru
pregătirea concursurilor vor fi redactate pe foi separate cu enunţ şi demonstraţie/
rezolvare (câte una pe fiecare foaie) şi vor fi însoţite de numele autorului, şcoala şi
localitatea unde lucrează/învaţă.
Redacţia va decide asupra oportunităţii publicării materialelor primite.
În atenţia elevilor
Numele elevilor ce vor trimite redacţiei soluţii corecte la problemele din rubricile
de Probleme propuse şi Probleme pentru pregătirea concursurilor vor fi menţionate
în Pagina rezolvitorilor. Elevii menţionaţi de trei ori vor primi o diplomă şi un premiu
în cărţi. Elevii rezolvitori vor ţine seama de regulile:
1. Pot trimite soluţii la minimum cinci probleme propuse în numărul prezent
al revistei (pe o foaie va fi redactată o singură problemă).
2. Elevii din clasele VI-XII au dreptul să trimită soluţii la problemele propuse
pentru clasa lor, pentru orice clasă mai mare, din două clase mai mici şi imediat
anterioare. Cei din clasa a V-a pot trimite soluţii la problemele propuse pentru clasele a
IV-a, a V-a şi orice clasă mai mare, iar elevii claselor I-IV pot trimite soluţii la
problemele propuse pentru oricare din clasele primare şi orice clasă mai mare. Orice
elev poate trimite soluţii la problemele de concurs (tip G şi L).
3. Vor fi menţionate următoarele date personale: numele şi prenumele, clasa,
şcoala şi localitatea, precum şi numele profesorului cu care învaţă.
4. Plicul cu probleme rezolvate se va trimite prin poştă (sau va fi adus direct) la
adresa Redacţiei:
Prof. dr. Temistocle Bîrsan
Str. Aurora, nr. 3, sc. D, ap. 6,
700 474, Iaşi
Jud. IAŞI
E-mail: t_birsan@yahoo.com
CUPRINS
Doctor PAUL TANCO (1843-1916) – personalitate a epocii sale
(primul român doctor în matematici) ........................1
Profesorul VASILE OPROIU – 75 de ani de viaţă şi activitate ................................................4
Ioan V. Maftei (1941-2016) ......................................................................................................6

ARTICOLE ŞI NOTE
M. DRĂGAN, N. STANCIU – A new method to solve inequalities.........................................7
V. JIGLĂU – An inequality on the medians of a triangle........................................................13
Gh. IUREA – An application of Pell’s equation ......................................................................17
M. CHIRCIU – În legătură cu o identitate în triunghi din revista Curierul Matematic............21

NOTA ELEVULUI
t. DOMINTE – Conice şi cubice în probleme elementare de loc geometric.........................25
V. TUCHILU , R.A. MORARIU, R. ANTOHI – Aspecte geometrice ale unei rozete
asociate unui triunghi .....................31

CORESPONDENŢE
A. REISNER – Un problème de la combinatoire des ensembles .............................................38

CUM CONCEPEM ... CUM REZOLVĂM


T. BÎRSAN – O privire asupra unei probleme arabe ...............................................................41

ŞCOLI ŞI DASCĂLI
A. RADU – Academia Mihăileană (1835-1864) ..................................................................44

CONCURSURI ŞI EXAMENE
Concursul interjudeţean „Speranţe Olimpice”, ed. a XVI-a, Paşcani, 2016 ..............................50

PROBLEME ŞI SOLUŢII
Soluţiile problemelor propuse în nr. 2/2016 ..............................................................................53
Soluţiile problemelor pentru pregătirea concursurilor propuse în nr. 2/2016 ............................69
Probleme propuse ......................................................................................................................83
Probleme pentru pregătirea concursurilor .................................................................................88
Training Problems for Mathematical Contests ..........................................................................91
Pagina rezolvitorilor ...............................................................................................................95
Elevi rezolvitori premiaţi ....................................................................................................... 96
ISSN 1582 – 1765 10 lei

S-ar putea să vă placă și